Sie sind auf Seite 1von 150

Case 1 – Adult Male Health Maintenance

Cardiovascular Diseases
 Hypertension
o Men 18-39 – Office BP measurement every 3-5 years if within normal range
o Men 40+ – Office BP measurement every year if within normal range
o If abnormal range – confirm with Ambulatory or Home Blood Pressure Measurement
 Lipid Disorders
o Men 20-34 – Fasting HDL/Total Cholesterol or Fasting HDL/LDL/Lipid panel if increased risk
o Men 35-45 – Fasting HDL/Total Cholesterol or Fasting HDL/LDL/Lipid panel every year
 Abdominal Aortic Aneurysm
o Men 65-75 & Ever Smoked – one-time abdominal ultrasound to rule in/out
o Men 65-75 & Never Smoked – may consider abdominal ultrasound, but not necessary
 Coronary Artery Disease
o Screening in asymptomatic adults with EKG, CT, or stress testing NOT recommended
 Peripheral Artery Disease
o Screening in asymptomatic adults NOT recommended
Cancer
 Colorectal Cancer
o Men 50-75 – colonoscopy every 10yr –OR– FOBT or Flexible Sigmoidoscopy every 3-5yr
o Men 76-85 – if never screened or if clinically indicated, pt may benefit
 Prostate Cancer
o Men Any Age - may be benefit for detection with Digital Exam or PSA, however, outcomes are
not shown to be consistently improved, thus screening is NOT recommended
 Lung Cancer
o Men 55-80 with 30+ smoking Hx OR quit within last 15yr – annual screening with low dose CT
scan of chest/abdomen/pelvis
 Bladder, Testicular, or pancreatic cancers
o Insufficient or not recommended to screen for any of these in an asymptomatic adult
Other Health Concerns
 Obesity
o All Adults BMI 30+ - intensive counselling/behavioral intervention to promote weight loss
 Type II DM
o All adults 40-70 overweight or obese – blood glucose screening at annual visits
o Initiate earlier screening if: HTN, hyperlipidemia, family Hx DM, Hx of gestational DM, Hx of
PCOS, or if Black/Hispanic/Pacific Islander/Native American
 Depression
o Use of a standard depression questionnaire in any patients who have not been screened or
suspected of depression.
 Tobacco Use
o Clinical History that shows any tobacco use warrants counselling and treatment for cessation
 Alcohol Use
o Use of standard alcohol abuse screenings (AUDIT or AUDIT-C) who have not been screened or if
suspected of alcohol abuse
Adult Immunizations/Lifestyle Interventions (from CDC recommendations)
 Influenza – every year around October regardless of health status
 Td/Tdap – Td booster every 10 years with a single Tdap in adult life; pregnant women need Tdap with
every pregnancy to protect the baby regarless of health status
 Varicella (Shingles) – one dose at age 60+, even if you’ve had the shingles before
o Contraindicated in pregnancy, immunodeficiency, or HIV with CD4 <200
 Pneumococcal
o PSV13 – single dose at age 65+ or under certain medical conditions (weak immune system, if
HIV+ regardless of CD4+, poor kidney function, or asplenia)
o PPSV23 – single dose at age 65+ or under certain medical conditions (weak immune system, if
HIV+ regardless of CD4+, poor kidney function, asplenia, chronic liver/heart/lung disease,
alcoholism, or diabetes I or II)
o Pts with risk factor diseases should receive at age 50+ and then either again 5 yrs later or at
age 65yr, whichever is later
 Meningococcal Disease – recommended for high risk groups (college dorm or military recruits),
asplenia, or certain compliment deficiencies
 MMR – <60yr and not previously received
o Contraindicated in pregnancy, immunodeficiency, or HIV with CD4 <200
 HPV – women age 19-26 OR men age 19-21 who haven’t gotten it already
 Varicella (Chickenpox) – any age if not previously received
o Contraindicated in pregnancy, immunodeficiency, or HIV with CD4 <200
 Hepatitis A – give if not received as a child
o Give if pt has chronic liver disease
 Hepatitis B – give if not received as a child
o Give if pt has chronic liver disease, HIV+, poor kidney function, or Diabetes I or II
 Haemophilus influenza (Hib) – give if pt asplenic, sickle cell disease, or had a bone marrow transplant
Lifestyle Interventions
 Exercise, Healthy Diet, Safe Sex, and Seat Belt Use should be encouraged at all visits however mixed
results have been shown as to the level of motivation counselling confers
Case 2 – Chronic Obstructive Pulmonary Disease (COPD)
COPD is a spectrum of disease encompassing for Chronic Bronchitis and Emphysema
 Chronic Bronchitis – cough/sputum production most days for at least 3mo during 2yrs consecutively
 Emphysema – SOB caused by enlargement of bronchioles and alveoli due to lung tissue destruction
The most common etiology of COPD is cigarette smoke exposure (90%; primary, second-hand smoke, or
environmental) but young/non-smokers may contract it due to rare a1-antitrypsin deficiency
 Pathologic changes: Mucous gland hypertrophy/hypersecretion, ciliary dysfunction/destruction, lung
tissue destruction, and airway remodeling
 Pathologic consequences: fixed airway obstruction, poor mucous clearance, cough, wheezing, dyspnea
 Presentation: cough (intermittent  constant), thick/white mucus production, dyspnea, “barrel
chest”, distant heart sounds, expiratory wheezes or distant breath sounds, accessory muscle use in
respiration, signs of cyanosis
o CXR: normal until advanced stage disease; may show lung hyperinflation, increased AP
diameter, flattening of diaphragm, and lung bullae formation
o Spirometry: FVC/FEV1 are both decreased and ratio is <0.7 without albuterol reversibility (gain
of 12% function or 200mL) and decreasing FEV1 with progression
Dx: clinical symptoms and spirometry
Tx: depends on severity of disease
 Stage 0 – early with cough/sputum but normal spirometry  smoking cessation (reduce rate of
pulmonary function decline to non-smoker rate), vaccination to pneumococcus and influenza (prevent
exacerbations)
 Stage 1 – FEV/FVC <0.7 | FEV1 >80% | +/- symptoms  add SABA (albuterol or ipratropium)
 Stage 2 – FEV/FVC <0.7 | FEV1 50-80% | +/- symptoms add LABA (salmetrol, tiotropium)
 Stage 3 – FEV/FVC <0.7 | FEV1 30-50% | +/- symptoms  add Inhaled corticosteroid
(fluticasone/triamcinolone) will aid in decreasing exacerbations
 Stage 4 – FEV/FVC <0.7 | FEV1 <30% or <50% with symptoms  add O2 supplementation 15hr/day

Case 2.5 – Uworld Tips about COPD


4665: COPD Physiology
 The hyperinflation and other signs of COPD occur due to two mechanisms:
o Static alveolar destruction: decreased elasticity and destruction of lung parenchyma physically
makes floppy, larger spaces for air fill up. Thus overall you have higher lung volume at baseline.
o Dynamic “air stacking”: elastic recoil of the alveoli in the lungs is largely responsible for
expiration. Because COPD pts have decreased elasticity, this expiration is slow. Should a COPD
patient need to breath faster (they go for a run) they’ll start their next inhalation before
exhalation is complete, causing air to “stack up” in the lungs causing hyperinflation
 The hyperinflation compresses the diaphragm and gives it less room to contract. Because the
diaphragm is the single most important entity for inhalation, a flattened diaphragm in COPD results in
increased effort on inspiration.

4645: Remember that vital capacity in COPD decreases due to air trapping and obstruction of expiration.
 Vital capacity = amount of air a person can expire with a normal expiration
 COPDers trap air in the lungs due to obstruction/destruction of alveoli, thus they have a harder time
getting the air out! Thus vital capacity deceases greatly
4718: Oxygen Induced CO2 retention in COPD
 In a severe chronic COPD-er, their body is accustomed to a state of hypoxia, which can be concerning
for hospital folks monitoring their O2 levels. However, rapid resolution of low O2 levels though oxygen
supplementation can result in acute CO2 retention through 3 mechanisms
o V/Q mismatch – normally, COPD causes VQ mismatch through alveolar destruction. However,
this is offset through selective vasoconstriction in the lung to only perfuse well ventilated areas.
With the addition of O2, even poorly ventilated areas have acceptable O2 levels for the body,
thus selective vasoconstriction does not occur AND V/Q mismatch worsens.
o Haldane effect – in hypoxemia, deoxyhemoglobin (dHb) serves as a buffer for H+ ions. With
additional oxygen, these normally dHb will become oxyhemoglobin oHb, releasing those H+
ions, resulting in [H + HCO3  H2O + CO2], raising CO2 levels
o Decreased respiration – the supplemental oxygen will cause increased oxygenation in the body,
which tells the brain to slow down breathing, as it considered O2 levels to be OK. However, this
also will retain CO2.
 These three effects together cause increased CO2 retention and acidosis  neural vasodilatation
(reflex typically used to increase bloodflow, but here causes underperfusion) and neurotransmitter
changes  seizures
 As a goal, O2 levels should only be cautiously raised to about 90-93% for COPD pts

4535: CO2 Narcosis is altered mental status in pts with PaCO2 >60mmHg. Classically this will occur in the
setting of COPD, where CO2 retention is commonplace. It may also contribute to a respiratory acidosis
(increased pH, decreased bicarb)

8905: Distinguishing Asthma from COPD


 Clinically they can both look similar, especially in an adult patient, but there are two major hallmarks
that can set asthma off from COPD:
o Reversibility: FEV1 increase by >12% demonstrates reversibility  asthma. COPD may be
partially reversible in early stages, but never to this degree.
o DLCO: normal or increased DLCO  asthma. COPD may be normal in early stages, but never
increased.
 Note that adult onset asthma is harder to treat (more limited response), but is usually the situation
where you’re trying to distinguish it from COPD

4730/4039/4569/4667: Acute COPD exacerbation


 Presentation: wheezing, tachypnea, prolonged expiration, and accessory muscle use. Difficulty
breathing NOT resolved with albuterol inhaler. Air trapping with hypoxia, hypercapnia. Classically
change in sputum from whiteyellow/green
o Recent upper respiratory infection is the most common cause
o Distant heart sounds: common finding as heart is displaced downward from lung hyperinflation
 Dx: hyperinflation on CXR & hypoxia/CO2 retention with changes in 1 or more of the following:
o Increased cough severity or frequency
o Increased volume or change in character of sputum
o Increased level of dyspnea
 Tx: supplemental oxygen, inhaled bronchodilators, systemic corticosteroids (oral prednisone or IV
methylprednisone), or muscarinic antagonists are standard treatment
o Antibiotics if suspected bacterial infection
o Oseltamivir is influenza suspected
o Mechanical ventilation if failure to resolve event with ventilatorary failure

2919/3031: Alpha-1 Anti-trypsin Deficiency


 Presentation:
o Lung: COPD (cough, dyspnea, wheezing, recurrent pulmonary infections, mucoid sputum)
 <45 years or with minimal/no smoking history
 Basilar predominant (smoke rises causing more damage in upper lungs)  decreased
lung sounds at the base with bilateral lucency of lung bases
o Liver: mildly abnormal LFTs, hepatocellular carcinoma, neonatal hepatitis
 Family history/person history of unexplained liver disease
o Skin: panniculitis (inflammation of subcutaneous fat)
 Dx: PFTs + serum AAT levels
 Tx: human AAT supplementation + bronchodilators + corticosteroids
o Lung/liver transplantation for severe disease is the definitive treatment

4520: Spontaneous Pneumothorax from COPD


 Presentation: sudden onset chest/back pain, SOB, reduced breath sounds on affected side, and hyper-
resonance
o Occurs due to chronic alveolar sac destruction forming blebs (seen as enlarged air sacs on
chest CT) that can rupture into the pleural space, compressing the lungs.
 Dx: clinical presentation with Chest CT evidence
 Tx: no complete concensus. Based on clinical scenario and risk stratification.

3716/3042: The best thing to do to lower mortality in COPD patients is stop smoking. Long-term oxygen
therapy (LTOT) is the only other thing that has been show to decrease mortality.
 Criteria for initiating LTOT are as follows:
o PaO2 <55mmHg or Sat <88%
o PaO2 <59 or Sat <89% in pt with cor pulmonale, evidence of HF, or hematocrit >55%

4593/4297: Cor pulmonale: process of increased lung-vascular pressure. Untreated, causes right heart failure.
 Etiology: COPD (most common), idiopathic pulmonary HTN, interstitial lung disease, obstructive sleep
apnea. Note that LVF causing RVF is NOT cor pulmonale.
 Dx:
o Sym: Dyspnea, syncope, or angina on exertion
o Sign: Peripheral edema, increased JVD, hepatojugular reflex, pulsatile liver, edema, ascities, etc.
o Imaging: EKG or echo can be used but right heart catheterization with elevated pulmonary
artery systolic pressure >25mmHg is confirmatory
 Tx: vasodilators/anti-HTN drugs
Case 3 – Joint Pain
If a patient presents with a non-traumatic mono-articular joint swelling, it’s important to examine the joint for
swelling, redness, warmth, or other signs of disease. Should these signs be present aspiration of the joint with
analysis of the joint fluid is the first step. Possible etiologies include:
 Septic Joint – very limited ROM (pain), joint effusion, fever. The ROM is very telling as cellulitis, bursitis,
and osteomyelitis will often still retain joint mobility
o Monoarticular joint infection is 90% of the time bacterial
o Chronic joint infection may be fungal or mycobacterial
o Acute polyarticular infection may be due to seeding from endocarditis or H.gonorrhea
o Aspiration will often have a positive culture, classically with S.aureus and WBCs around
100,000 (mainly neutrophils); in fact, these are more common in patients with rheumatoid
arthritis as the chronic inflammation and use of steroids predispose to infection
o HIV(+) pts may have N.gonorrhea, Salmonella, or H.flu infections
o IVDU pts may have odd organsisms like Strep spp., Staph spp., Gram(-) bugs, or Pseudomonas
o Tx: drainage and IV antibiotic treatment of infection
 Gouty Arthritis – classically of the first metatarsophalangeal joint (podagral) but can happen at any
joint. Often monoarticular.
o Aspiration yields Monosodium Urate crystals (needles, negative birefringence) with high WBC
content (2000 – 60,000)
o Classically occurs with trauma, surgery, heavy meals (wine/cheese; high purine foods), and
thiazide diuretic use (reduces renal uric acid excretion)
o Tx: NSAIDs with avoidance of triggers; possibly others if recurrent
 Pseudogout – similar to gout but aspiration will yield calcium oxalate, positively birefringent,
rhomboid crystals. Classically this can occur in pts with kidney disease (poor oxalate excretion)
o Tx: NSAIDs with treatment of underlying condition
 Osteoarthritis – classically in older adults due to repetitive use of joint, trauma, or obesity causing
cartilage destruction and bone-to-bone contact/ligament damage
o Pain gradually onsets with dull/deep ache that worsens at the end of day/improves with rest
and may become constant at later stages.
o Crepitus, joint instability, effusion  joint deformity and limited ROM
o Typically, X-rays are normal at first with development of bony sclerosis, subchondral cysts, and
osteophytes occuring as disease progresses
o Tx: NSAIDs with physical therapy and possibly joint injections
 Rheumatoid Arthritis – classically affecting women in their 30-50s, it’s an autoimmune disease that
attacks the joints, causing damage and destruction
o Often worse in the morning and improves with movement; with classic deformities to the
hands/fingers and possibly signs of other autoimmune disease
o (+)Rheumatoid factor & (+)anti-cyclic citrullinated protein (CCP) are classic immunologic
correlates; other autoimmune diseases may be positive (more common if you have one!)
o Elevated ESR, CRP, anemia, thrombocytosis, and low albumin can all be signs of chronic
inflammation
o Tx: Physical therapy + DMARD (Sulfasalazine/Methotrexate/Infliximab/Etanercept)
Case 4 – Prenatal Care
[Ch.6 Pre-conception and Antepartum Care – Topic 9 & 10]
All health encounters during reproductive years should include counseling about proper medical care to have
the best possible pregnancy outcomes. This is especially emphasized with pre-conception care! It is known
that good preconception counselling and antepartum care increase the likelihood of a healthy baby!

Preconception Counselling
Vaccinations
 All women should receive these if planning a pregnancy: rubella, varicella zoster, pertussis, and
hepatitis B vaccines should be given, unless already immune. An HIV test should also be given unless
the patient specifically declines it.
o Pregnancy should be delayed 1 month after receiving any live vaccine
 If woman is at risk, screen for: STD, TB (Mantoux test), any other worrisome disease
 Some genetic testing can be offered based on race: sickle cell (AA), B-thalassemia (Mediterranean,
southeast Asians), a-thalassemia (Mediterranean, southeast Asians), Tay-Sachs (Ashkenazi Jews),
Canavan disease (Ashkenazi Jews), Cystic Fibrosis (White people and Ashkenazi Jews), anything
indicated with family history
Health Maintenance
 Make sure pre-existing conditions are well controlled (HTN, diabetes, thyroid disorders, seizures, etc.)
 Folate supplementation (0.4mg normally or 4mg in moms with Hx of NTDs)
 Lifestyle adjustments: Work to achieve proper weight, abstaining from alcohol/tobacco/drugs,
advising on certain foods to avoid (soft cheese, deli meats, etc.), exercise, etc.

Diagnosis of Pregnancy
 Presentation: one/more missed periods following sexual activity without proper contraception
o Symptoms: Breast tenderness, fatigue, nausea/vomiting are common
o Early Signs: Uterine enlargement (apparent around 6wk), Chadwick sign (bluish vaginal
discoloration), Hegar sign (softening of the cervix)
o Late signs: quickening (perception of fetal movement, 16wk), hyperpigmentation of skin/linea
alba (increased a-MSH)
 Dx: urine b-hCG detection/serum b-hCG quantitative testing/ultrasonographic detection of fetal heart
o Best performed early in the morning when hCG levels are highest.
o Home pregnancy tests are designed for specificity (low false positive rate, high false negative)
o B-hCG levels should double every 48hrs, but at minimum must increase 53% in that time.

Initial Prenatal Visit


 Full history/physical with a pelvic exam to assess 1dimentions of the bony pelvis, 2cervical length/
consistency/shape/mobility/dilation
 History/physical should focus on risks that they can give to the pregnancy (smoking, alcohol, seat belt
use, etc.), then focused testing or intervention to minimize these risks should be undertaken

Assessment of Gestational Age/Estimated Date of Delivery


 Gestational age – number of weeks elapsed since the first day of LMP and the date of delivery
o Naegele rule (LMP + 7 days – 3 months) is used to quickly calculate estimated date of delivery
(EDD) from LMP
o These are useful but can be flawed, as an exact date of the LMP and a normal uterine cycle
must be present for it to work
o Ultrasound measurement gestational age is much more accurate; best at 18-20wk gestation
Subsequent Antenatal Visits
 Typically, you schedule a visit [once every 4 weeks from week 1-28], then [once every 2 weeks from
week 29-36], then [weekly visits thereafter]
 More frequent visits may be scheduled for patients at risk for complications!
 In these visits you:
o Ask about problems (bleeding, nausea/vomiting, dysuria, vaginal discharge)
o Ask if mom continues to feel regular fetal movements
o Measure blood pressure, weight, fundal height, and fetal heart tones
 Fundal height approximates the #of weeks gestation until 36 weeks; after 36, the baby’s
head engages and the uterus begins to shrink as it moves into the pelvis
 Fetal heart rate should be approx. 110-160bpm; deviation should be assessed
o Palpate the uterus for presentation (95% are ultimately ‘vertex’)
 Note that recommended maternal weight gain is adjusted for starting weight:
o Underweight – gain 28-40lbs (1lb per week)
o Normal weight – gain 25-35lbs (1lb per week)
o Overweight – gain 15-25lbs (0.6lb per week)
o Obese – gain 11-20lbs (0.5lb per week)
Common Screening Tests
 First Trimester (10-13wk) – pregnancy-associated plasma protein A (PAPP-A), B-hCG, and ultrasound
assessment of nuchal translucency
 Second Trimester (15-20wk) – triple (MS-AFP, estriol, B-hCG) or quad (triple + inhibin A) screening or
other genetic screenings (cell-free DNA), without an ultrasound exam for signs of trisomies/NTDs
 Third Trimester (20+wk) - glucose challenge test (24-28wk normally, first trimester if obese or
diabetic) with subsequent glucose tolerance test if abnormal. GBS swab (35-37wk) is also done
universally. Hemoglobin, hematocrit, HIV, and Rh-factor repeat are also done.

Fetal Assessment Techniques


 Fetal growth is best charted via ultrasound
o Early: crown-rump length correlates closely with gestational age
o Later: skull circumference/diameter, abdominal circumference, femur length, and cerebellar
diameter are used to assess gestational age and fetal weight
 Fetal Well-being is assessed via many techniques
o Kick Counting – common, indirect measurement that mom can do! Mom should feel
approximately 10 fetal movements every 2 hours
o Non-stress test (NST) – mom’s intrauterine pressure and baby’s heart rate are monitored via
external monitoring. Baby’s line hopefully shows variability, accelerations, or late decelerations
in correspondence with increased maternal pressure. Non-reactive, early decelerations,
variable decelerations, or other abnormal findings should be followed up.
o Contraction or Oxytocin Stress Test – basically an NST but baby’s heart rate is looked at during
a contraction or mom is given oxytocin for a contraction to see how it reacts. High false positive
rate, thus multiple tests must be interpreted collectively to give a proper picture.
o Biophysical Profile (BPP) – typically these are undertaken when non-reassuring NST is seen. It is
an ultrasonographic evaluation of 5 parameters, lasting 30 minutes. 8/10 or 10/10 is good.
 Non-stress test – if reactive, +2
 Fetal Breathing – >1 episode of diaphragm movement for >30sec, +2
 Fetal Movement – >3 discrete arm/leg/back/finger movements, +2
 Fetal Tone – >1 extremity/back/hand extension with return to flexion, +2
 Amniotic Fluid volume – a single pocket >1x2cm, +2
o Modified BPP – only a non-stress test (reactive is good) and amniotic fluid index (AFI;
assessment of 4 quadrants of amniotic fluid; >5 is good). Typically used as it’s easier and
similarly predictive as the full BPP
o Ultrasonography of the umbilical artery bloodflow – a technique to assess forward flow
through the artery, typically using a systolic/diastolic ratio to assess how well flow occurs.
Placental insufficiency can lead to significant morbidity/fetal death, and if the placenta is
“pushing back” less diastolic flow will occur. Thus increased sys/diastolic ratio quantifies
worsening placental blood flow and can even forewarn of impending fetal demise. Typically
end systolic reverse flow is an indication for induction/C-section.
 Fetal Maturity is assessed through lung development, as it’s one of the last systems to fully mature
o Surfactant/albumin ratio (via amniocentesis) – high means higher surfactant production
o Lecithin/Sphingomyelin ratio – higher means more lecithin (a component of surfactant)
o Phosphatidylglycerol – present with complete fetal lung maturity, approx. 35wk
o These techniques are important in assessment, as immature lung function increases the risk for
respiratory distress syndrome (collapse of airway due to lack of surfactant)
 Should a neonate need to delivered early, balancing risk of RDS vs continuing pregnancy
must be assessed
 Antenatal corticosteroids, post-natal ventilation/close monitoring are mainstays
Antepartum Patient Education
 Employment – you can keep working; look into your job’s maternity leave policies
 Exercise – 30min of moderate exercise per day is good. Avoid supine exercises. Don’t go too hard.
Don’t get into a sauna or hot tub afterwards. Remember your balance will change, increased fall risk
 Nutrition/Weight gain – iron and folate supplementation is good. You may get pica (sign of anemia).
Follow the BMI-based guidelines on how much weight you should gain. You’re not eating for two.
 Breastfeeding – it’s really good for baby (immunity, good nutrients, bonding) and you (weight loss,
bonding), just make sure to supplement Vit.D (or sunshine) and don’t deny the Vit.K shot.
 Sexual activity – OK unless contraindication (placenta previa, PPROM, or preterm labour).
 Travel – OK. Make sure your properly immunized for where you’re going. Get a record of your gyn care
in case you go into labour. Don’t sit around too much (increased risk of DVT, walk every 1-2hr). It’s
recommended NOT to travel after 36wk…you may go into labour away from your normal healthcare.
Wear a seatbelt every time.
 Teratogens – timing, dose, and susceptibility will determine the effects of a teratogen
o Medications: Short list of teratogens. Refer to pg. 75-76 for good summaries
o Ionizing radiation: <5 rads is fine. Usually concern with therapeutic range radiation doses (think
chemotherapy). Most imaging techniques are specialized in pregnancy to give a low rad dose.
o Methyl mercury: Large fish (tuna, shark, king mackerel) have higher mercury content because
they eat smaller fish, and should be avoided in pregnancy. Shrimp, canned light tuna (not white
albacore) salmon, pollock, and catfish are all low and are fine to eat (approx. 2x week)
o Herbal remedies: don’t take these. They don’t work and could cause harm
o Alcohol – don’t drink it. The #1 most common preventable cause of mental retardation,
developmental delay, and birth defects.
o Smoking/Drugs – don’t do it.
Common Pregnancy symptoms
 Headache (normal; if not relieved by Tylenol then call the doc); Edema (normal; should resolve with
sitting down/going to bed); Nausea/Vomiting (normal in 1st trimester, hyperemesis gravidarum is rare
but will be much worse); Heartburn; Constipation (physiologic; docusate/psyllium can help); Fatigue
(typically restricted to 1st trimester) Leg Cramps/Back Pain/Round Ligament Pain (all from bodily
adjustments you’re making to accommodate baby); Varicose Veins/Hemorrhoids; Vaginal Discharge
Risk Factors for Genetic Disorders
 Advanced maternal age - increases the risk for trisomy 21/all chromosomal abnormalities (increasing
risk with increasing age), however the majority of tri-21 cases occur in women younger than 35
 Previous chromosomal abnormality – basically if you’ve had one previously, it increases the chance
(1.6-8.2 x maternal age risk) of having another depending on various factors)
o Type of trisomy, spontaneous abortion or no, maternal age at previous affected pregnancies,
current maternal age
 Hx of early pregnancy loss – 50% of first-trimester losses are due to chromosomal abnormalities. The
most common is monosomy X, however others all can have this effect
 Advanced paternal age – fathers >50yrs significantly increase the risk of X-linked recessive and
autosomal dominant disorders due to increased risk of mutation
 Ethnicity – blacks (sickle cell), whites (cystic fibrosis), and Ashkenazi Jews (Tay-Sachs, Gaucher,
Niemann-Pick diseases), Mediterraneans (B-thalassemia), and Asians (a-thalassemia) are mentioned in
the text, however this is simply to highlight that certain races have certain predilections for disease

Prenatal Screening (finding someone at heightened-risk in a low-risk population)


 First-Trimester Screening (1-13wk) – Trisomy 21, Trisomy 18, Trisomy 13
o hCG, PAPP-A are both measured in the first trimester
o Advantage is that decisions about continuation of pregnancy can be made at this time
o Trisomy 21
 Associated with elevated hCG and decreased PAPP-A
 Combined with ultrasonographic imaging for nuchal transparency (fluid collection at the
fetal neck; an early sign of trisomy 21)
 When noting these markers, genetic council, 2nd trimester CVS/amnio are offered
o Any major/minor structural malformations increase the risk of aneuploidy and should warrant a
karyotyping of mom/genetic testing of fetus
 Second Trimester Screening (14-27wk)
o Triple screen (AFP, B-hCG, unconjugated estriol) or quadruple screen (+ inhibin A) are used in
detection of Trisomy 21, as well as other chromosomal disorders
 Trisomy 21
 Decreased – AFP, unconjugated estriol
 Increased – B-hCG, Inhibin A
 Trisomy 18 – all levels decreased
o Ultrasound – cardiac defects and “soft markers” (nuchal fold, ventriculomegaly, echogenic
bowel, shortened femur/humerus, absent nasal bone, pyelectasis) increase risk of Down syn.
 Finding abnormalities on ultrasound  genetic counseling and further testing
o Screening for Neural Tube Defects (NTDs) – elevated maternal serum AFP indicated increased
risk for NTDs. If a mom has elevated AFP  ultrasound exam for abnormalities should be done.
 All women should have 0.4mg folic acid prior to conception as most women will not
know their child has a NTD until it is too late to correct it.
 Women with a previous NTD pregnancy should have 4mg daily folic acid
 Integrated Screening
o Using both first-trimester AND second-trimester testing together, gives a 85-88% detection rate
of serum-detectable abnormalities
Questions from This Section
 The most accurate week for ultrasound is between week 14.0 – 15.6
 While PAPP-A/Nuchal translucency/free B-hCG is a fine first trimester screening test for Down’s
Syndrome, the most accurate test is the maternal cell-free DNA test. It will detect 99% of Down’s
syndrome cases, and can be used as early at 9wks.
 When a pregnant patient fails with 1-hr and 3-hr oral glucose tolerance test the first change is to
educate/initiate glucose monitoring + diabetic diet. If this fails to put person in normal glucose ranges,
then insulin or an oral hypoglycemic agent may be initiated. Further oral glucose challenges are likely
not necessary.
 Shoulder dystocia, metabolic disturbances, preeclampsia, polyhydramnios and fetal macrosomia are all
associated risks of gestational diabetes. Apparently intrauterine growth restriction (IUGR) is a
complication associated with pre-existing diabetes in pregnancy.
 According to ACOG, the recommended dose for patients not at risk for NTDs is at least 0.6 mg/day.
However, other organizations may suggest 0.4mg/day. Regardless, women at increased risk should
take 4mg/day, switching to 0.6 mg/day after the first trimester.
 Valproic acid use during pregnancy is associated with a 1 to 2% incidence of neural tube defects,
specifically lumbar meningomyelocele. Fetal ultrasound examination at approximately 16 to 18 weeks
gestation is recommended to detect neural tube defects.
 In an apparently malnourished patient, folic acid supplementation, as well as evaluation for
deficiencies in her iron, protein and other nutrient stores should occur. In general, a patient needs
approximately 70 grams of protein a day, along with her other nutrients. Pts should never lose weight
in pregnancy, but should adjust weight gain based on BMI.
 Incorrect dating, specifically under-estimation of gestational age, is the most common explanation
for an elevated MSAFP. 90-95% of cases of elevated MSAFP are caused by conditions other than NTDs
including under-estimation of gestational age, fetal demise, multiple gestation, ventral wall defects and
a tumor or liver disease in the patient. Next appropriate step in the management of this patient is to
obtain an ultrasound to assess the gestational age, viability, rule out multiple gestation as well as a
fetal structural abnormality.
 While Cell-free DNA screening can detect 99% of Down’s syndrome cases, it should not be used as a
rule-out for chromosomal abnormalities. Amniocentesis or Chorionic Villus Sampling would be better
choices (although the question doesn’t explain why). The fetus in this case had a thickened NT, this
patient should be scheduled to have a detailed fetal ultrasound and echocardiogram at 18-20 weeks to
rule out anomalies. However, it is not possible to diagnose a chromosomal abnormality with an
ultrasound. Similarly, although genetic sonograms (targeted sonogram) focus on markers associated
with Down syndrome, they are not diagnostic.
 Ibuprofen is safe to take until around 32 weeks gestation, when premature closure of the ductus
arteriosis is a risk. While heparin is safe during pregnancy, warfarin has known teratogenic effects and
should not be given. If continued anticoagulation is necessary, low molecular weight heparin is the
drug of choice.
[Ch.7 Genetics and Genetic Disorders in OBGYN – Topic 9 and Topic 32]
Prenatal Diagnosis of Genetic Disorders
 Essentially you have someone in a high risk group (Ashkanazi Jews) or you’ve identified someone via
pre-natal screening tests and diagnosis is being sought
 Carrier testing
o Testing to see if mom or dad carry a certain disease; typically one parent is tested first, then the
other if their test comes back positive
o Direct testing – blood/saliva sample is taken and the DNA in those cells is tested fro specific
mutations or sequences that have been shown to cause disease. This will only work with
diseases that have an already-identified genetic basis
o Indirect testing – same process, but the test is looking for DNA sequences that are often linked
to mutations that cause disease called restriction fragment length polymorphisms. This is used
in diseases that do not have a specific mutation to test for yet.
 Fetal Diagnostic Procedures
o Amniocentesis – taking some fluid under ultrasound guided needle extraction for gene analysis
 Early (11-13wk) – heightened risk for complications/fetal demise, not recommended
 Midtrimester (15-20wk) – >99% diagnostic accuracy and a <1% risk for complications
o Chorionic Villus Sampling (CVS) – using ultrasound guided needle to sample part of the chorion
(immature placenta)
 Before 10 weeks – increased risk of limb/oro-facial abnormalities, not recommended
 Transabdominal CVS After 10 weeks – similar risk profile to midtrimester amniocentesis,
thus thought to be relatively safe, and able to be used earlier
o Percutaneous Umbilical Blood Sampling (PUBS; “cordocentesis”) – sampling of the fetal blood
via the umbilical cord after 20 weeks gestation
 Benefit of rapid karyotyping (18-24hrs), but not as useful nowadays
 Genetic Counseling – typically referral to a medical geneticist who will council the patient using the key
elements of accurate diagnosis, communication, and non-directive presentation of options
o The whole point is to inform the patients as much as possible for them to make an informed
decision as to the options that are available
Questions form This Section
 Serum Screening Tests in order of sensitivity
o Triple Screen (69%), Quad Screen (81%), 1st trimester combined screen (PAPP-A, hCG, nuchal
translucency; 85%), Integrated screen (if nuchal translucency not obtained, 85-88%),
Sequential Screen (1st trimester combined + 2nd trimester quad; 93%), and Cell-Free DNA
(>99%)
Case 4.5 – Antepartum Care Tips from UWorld
2404: Routine Prenatal Visit Labs
 First visit: Blood Type/Rh-D type/antibody screen (Lewis, Duffy, Kell); Hb/Hct/MCV, diseases (HIV,
VRDL/RPR, HBsAg, Chlamydia PCR), Rubella/varicella immunity, urine culture, urine protein
 24-28wk: Hb/Hct, Antibody screen with RhoGAM admin (If RhD-), and 50g 1-hour glucose challenge
 35-37wk: GBS swab and culture

10441: If there’s any question that wants you to do screening for the baby and ‘cell free DNA testing’ is an
answer choice…just choose that one. Although it isn’t diagnostic (you’ll need a CVS or amniocentesis for that)
you can get great screening results to guide further testing.
2569: 2nd trimester Quadruple Screening Classic Findings:
 Trisomy 21: MS-AFP low; B-hCG low; Estriol low; normal inhibin A
 Trisomy 18: MS-AFP low; B-hCG high; estriol low; inhibin A high
 NTDs or abdominal wall defect: MS-AFP high; all other normal
o The most likely cause of an elevated MS-AFP is fetal abdominal wall defect (gastroschesis or
omphalocele); but NTDs and multiple gestations can do it as well
o AFP is like the albumin of the fetus, thus if the gut is open to the amniotic fluid, AFP rises
o Note that elevated AFP warrants careful ultrasound evaluation for structural abnormalities

2568: In a patient with abnormal triple/quad screening results; the next best step for assessment is an
ultrasound. While you may be temped to go straight to CVS/amniocentesis, the ultrasound can be used to
check for anatomic abnormalities associated with congenital defects as well as guide you for the best place to
do the amniocentesis. CVS/amnio will get done, they’ll simply need an ultrasound first.

Case 5 – Well-Child Exam


Pediatric History
 Prenatal/Perinatal History – Duration of pregnancy, complications, birth weight, type of birth,
medications given, and any records from the hospital pertaining to the child’s birth
 Family History – any diseases, problems, or abnormal growth patterns
 Social history – parental education, religion, smoking/drinking/drug use, etc.
Growth
 Height/weight should be taken and plotted against a standard growth chart
 Head circumference should be taken in children <3yr of age
 Blood pressure taken in children starting at 3yr of age
Development – typically plotted against standard things that a child should do at that age
Age Motor Language Social Other
1 mo Reacts to pain Responds to noise Eye contact/notes
the human face
2 mo Eyes follow object Vocalizes Social smile/
to midline/head up recognizes parent
prone
4 mo Eye follow object Laugh/squeal Note hands
past midline/rolls
over
6 mo Sits unsupported/ Babbles Recognizes
transfers objects strangers
between hands /
rolls prone to
supine
9 mo Pincer grasp/ “Mama”, “Dada”, Explores
crawls/ cruises “bye-bye”
1 yr Walks/ throws 1-3 words; follows Stranger anxiety/ “1 word at 1”
objects 1 step commands separation anxiety
2 yr Traverses stairs/ 2-3 word phrases, Parallel play
copies a line/ runs/ refers to self as
kicks ball name, pronouns
3 yr Copies a circle, Sentences, Group play, simple Tricycle, 3 cubes, 3
pedals a tricycle, recognizes 3 colors games, knows colors, 3 kids in a
bridge of 3 cubes, gender, knows group
repeats 3 numbers 1st/last name
4 yr Knows body parts, Speech is Plays with kids,
copies a cross, understood by all, social interaction
Copies a square, tells a story
hops on one foot,
throws overhand
5 yr Copies a triangle, Writes name,
catches a ball, counts 10 objects
partially dresses
self
6 yr Draws a person Knows right form
with 6 parts, ties left
shows, skips

Screening Tests
 2 weeks or younger – PKU, congenital hypothyroidism, hemoglobinopathies, galactosemia, and in-born
errors of metabolism are all screened for in early life
 Lead poisoning – more rare nowadays, but 1yr and 2yr should be screened
 Iron deficiency – usually taken between 6mo – 1yr, often iron deficiency is the most common cause of
anemia in children. Often empiric treatment with iron supplementation is done
 Hearing/Vision screening – often done in the newborn nursery.
o Hearing typically done by snapping finger and noting a response
o Vision tested by looking for the red reflex (present is NORMAL)
o Strabismus should be referred to pediatric opthamology as soon as noted
 TB in high risk children (often in immigrants or with family that has known Hx of TB)
 Lipid panel between ages 2-10yr to rule out congenital lipid disorder

Guidance
 Includes guidance on injury prevention, nutrition, discipline, exercise, mental health, and healthcare
 Accident/injuries are the leading cause of death in children >1yr old
 Car seats should be a mainstay
o <20lbs – rear facing car-seat
o 20-40lbs – forward facing car seat
o >40lbs – booster seat until about 4’9” (can sit with back against the seat rest with legs bent)
 Sudden Infant Death Syndrome (SIDS) – sudden, unanticipated death of a child during sleep. Thought
to be due to shutting down of the brainstem during REM and suffocation. Parents should have children
sleep on their backs on a firm mattress in the crib, with nothing else in the crib with them
 Food
o Breast feeding is the best initial nutrition for the infant
o Cereal/other baby food/water introduced at 4-6mo of age
o Whole cow’s milk introduced at 12mo and continued until 2yrs, then switch to skim
Immunizations from Age 0-18yr
Immunization Dose 1 Dose 2 Dose 3 Dose 4
Hepatitis B Birth 1-2 months 6-18 months
Rotavirus 2 months 4 months
DTaP 2 months 4 months 6 months 15 month + 4-6yr
H.flu (Hib) 2 months 4 months 12-15 months 12-15 months
Pneumonia 2 months 4 months 6 months 12-15 months
(PCV13)
Polio (Inactivated) 2 months 4 months 6-18 months 4-6yr
Influenza Start at 6months
(annual Vaccine)
MMR 12-15 months 4-6yr
Varicella 12-15 months 4-6yr
Hepatitis A 12-23 months 12-23 months
Meningococcal 11-12 yr
TDap 11-12 yr
HPV >11yr >11yr >11yr
Meningococcus B 16-18yr
Pneumonia PPSV23 High-risk only

By age:
 0mo – Hep B
 2mo – HepB, Rota, DTaP, H.flu, PCV13, Polio
 4mo – Rota, DTaP, H.flu, PCV13, Polio
 6mo – Hep B, DtaP, PCV13, polio, Flu
 12mo – H.flu, PCV13, MMR, Varicella, HepA
 6yr – DtaP, MMR, Varicella
 12yr – Meningococcal, TDaP, HPV
 18yr – Meningococcus B
Case 6 – Allergic Disorders
Allergic Rhinitis – inflammation of the nasal mucous membranes due to allergy exposure
 Presentation: sneezing, nasal congestion/itching, rhinorrhea, anosmia, post-nasal drip, headache, ear-
ache, watery/itchy/red eyes, or drowsiness. Sinuses are often inflamed with blue-grey coloration
(venodilation).
o The result of IgE-response to allergen, which allow activation of mast-cell degranulation when
allergen is present in the body after sensitization
o Mast cell degranulation results in histamine, tryptase, chymase, kinase, leukotriene, and
prostaglandin D2 release to mediate inflammation, mucous production, and nerve sensitization
o Neutrophil/eosinophil/leukocyte/macrophage recruitment occurs to continue inflammation
and may result in systemic effects (malaise, fatigue, sleepiness)
 Physical Exam: allergic shiners (dark circles under eyes), small horizontal crease on nose bride (“allergic
crease”) caused by rubbing the nose (“allergic salute”); boggy, pale blue-grey sinuses producing clear,
thin mucus.
o Nose: palpation of sinuses, Inspection for nasal polyps, tumors, septal deviation
o Ear: tympanic membrane retraction, air-fluid levels, or bubbles behind the eardrum or loss of
tympanic mobility may all occur with Eustachian tube involvement
o Eyes: injection/swelling/tear production; sometimes with circles/lines
o Throat: erythema, “cobblestoning”, or tonsillar hypertrophy
 Dx: presentation with search for possible triggers
 Tx:
o Education and avoidance of allergen are primary modalities
o 1st gen antihistamines (diphenhydramine) – cheap, but often sedating & anticholinergic effects
(dry mouth, blurry vision, urinary retention, esp in elderly)
o 2nd gen antihistamines (loratidine/cetirizine) – more expensive, but non-sedating due to poor
CNS penetration and less anti-cholinergic size effect. Often onset 15-30min after taking, thus
best used intermittently for flare-ups
o Decongestants (pseudoephedrine) – a-agonist given intranasal that causes vasoconstriction.
Best used very short term for administration of other drugs as rebound-hyperemia will ensure
that the pt will never stop using the spray.
o Corticosteroid Spray – very effective for long-term control or persistent allergies; often working
within 2-4wk. Nosebleeds, pharyngitis, and URIs can all occur more often.
o Leukotriene inhibitors (montelukast or zileuton) – used best in pts with allergies + asthma as
they can treat both conditions; however, not more effective than corticosteroids
o Oral Corticosteroids – only to be used with severe, unresponsive allergies
o Desensitization therapy – used in pts who fail to respond to treatments
 1. Spot testing to ID specific allergens
 2. Injections of highly diluted allergens with progressive increasing increments,
eventually reducing antigen inflammatory response (weekly/biweekly)
 note that anaphylactic reactions may occur

Anaphylaxis – widespread, severe allergic reaction due to systemic response to allergen


 Presentation: rapid onset dyspnea/bronchospasm, viscera edema, hypotension, GI/uterine
contraction, urticarial and angioedema after allergen exposure
 Dx: clinical
 Tx: immediate administration of 1:1000 dose aqueous epinephrine (epi-pin) with repeated injections
every 15-30min as necessary. If hospitalized, IV fluids should be started and intubation should be
undertaken if necessary with monitoring for the next 24hr
Conjunctivitis – infection of the conjunctiva (the most common eye disease)
 Presentation: redness, discomfort, and sometimes crusting of one/both eyes, photophobia; NO
blurring or loss in vision should occur in this disease
 Dx/Tx: based on swab/culture for bacterial or virus analysis
o Bacterial – Staph, Strep, Haemophilus, Moraxella, or Pseudomonas could all occur; often
unilateral but may become bilateral; sulfonamide eye drops should clear the infection
o Viral – most commonly Adnovirus; often less crusting and more watery discharge and often
bilateral; weak topical steroids used to control inflammation with resolution in approx. 2wk
o Allergic – will occur with other allergy symptoms and resolve with treatment
o Chemical – result of damage to eye; immediate flushing and normal care should resolve

Case 7 – Tobacco Cessation


Tobacco use is the single greatest cause of preventable death and it’s been shown that physician intervention
at patient encounters increases tobacco cessation rate. Discussion should involve the Five As:
 Asking about current level of tobacco use
 Advising to quit through specific messages for the patient
 Assess willingness to quit and proceed when patient would like to proceed
 Assist Quitting via help groups, encouragement, and pharmacologic intervention
 Arrange for follow-up and support!
Drug Therapy
 Nicotine Replacement – gives the addictive substance with gentle decrease in amount until no nicotine
is being used. May result in replacing smoking with a new nicotine habit (inhaler with highest
potential)
o All Category D drugs; thus cannot be used in pregnancy
o Nicotine gum should be chewed then placed in cheek (like chewing tobacco); continuous
chewing actually decreases efficacy
 Zyban (long acting Bupropion) – NE/DA blocker which is thought to remove the “reward” gain by
smoking, thus making smoking less enjoyable, thus easier to quit
o Begin 1-2wk before quit date with 150mg for 3 days, then 150mg BID for 4 days for 7-12wk;
may be used in conjunction with nicotine replacement therapy
o Adverse effects – dry mouth, insomnia
o Contraindication – Eating disorders, on MAO-inhibitors within last 2wks, or seizure disorders
o Category C-drug, thus not known what it does in pregnancy, but relatively safe
 Chantix (Varenicline) – partial nicotine agonist reducing cravings, withdrawal symptoms, and may
decrease smoking’s “reward” via blocking endogenous nicotine agonism
o Begin 1wk before quit ate with 0.5mg for 3 days, then 0.5mg BID for 4days, then 1mg/day for
the rest of the 6mo treatment; may be used with nicotine replacement therapy
o Adverse effects – nausea, insomnia, vivid dreams, psychiatric effects (behavior changes,
agitation, depression, suicidal ideation)
o Drug must be used in reduced dosages in pt with renal clearance <30 mL/min
o Category C-drug, thus not known what it does in pregnancy, but relatively safe
 Women who smoke should be encouraged to quit without the use of pharmacotherapy; but Zyban and
Chantix are NOT contraindicated. While benefits are best if smoking ceases before; occurring with
cessation at ANY time during pregnancy.
Case 8 – Ethics in Medicine
This case addresses a teenager with an issue that they want to keep private from their parents; while
autonomy, beneficence, nonmaleficence, and justice should all be followed, there are situations where a
patient’s wishes may not be followed:
 Suicidal or homicidal ideation – pt is a danger to themselves or others and this must be reported
 Serious chemical/drug dependence – pt is in danger of death due to overdose
 Abuse – pt is in danger from their abuser and must be protected
Some situations may happen where a minor is considered able to receive care without parental consent:
 Judicial bypass – a state law that can allow a minor to be deemed “mature” thus allowing them to
undergo medical treatments under their own will
 Emancipation – a minor is of certain age, lives apart from parents, and is self-sufficient. May also occur
if patient is married, pregnant/has children, in military, or simply declared emancipated by the law.
 Type of healthcare sought – maternity services, contraception, STI dx/tx, reportable disease dx/tx,
treatment of drug/alcohol related problems, sexual assault treatment, or mental health treatment
Reportable diseases: confidentiality is often requested, but some diseases are legally required to be reported
 This varies by state but often includes STIs, HIV, and other serious infectious diseases
 Pt’s partners and parents may be informed through the state department
 It’s best to inform the patient that their disease MUST be reported, and the health department will
inform partners/parents without the patient’s consent if they do not go themselves.
Pregnancy/abortion services availability to teenagers vary by state law. It’s best to simply look up where your
state laws stand with these situations.

Case 9 – Anemia in the Geriatric Population


Presentation: weakness, fatigue, conjunctival/buccal/palmar pallor with additional signs pointing to cause of
the anemia
 Microcytic/normocytic – iron deficiency, thalassemias, anemia of chronic disease, anemia of elderly,
sideroblastic (B6 deficiency), acute or chronic hemorrhage, RBC hemolysis
 Macrocytic – folate deficiency, b12 deficiency

Case 9.5 – Anemia Tips From UWorld


4358/2877: Iron Study Characteristics in Microcytic Anemias
 Iron Deficiency: not much iron around, thus the body is looking to take up as much as possible
o Decreased: MCV, serum iron, ferritin, transferrin saturation (iron/TIBC)
o Increased: TIBC
 Thalassemia: poor production of hemoglobin leaves iron out in the blood
o Decreased: MCV, TIBC
o Increased: iron, ferritin, transferrin saturation
 Anemia of Chronic Disease: the body takes inflammation to mean “bacterial infection” and sequesters
iron in hopes that it’ll limit bacterial reproduction.
o Decreased: MCV, iron, TIBC, transferrin saturation
o Increased: ferritin
o Typically, a chronic inflammatory disease (autoimmune is classic!) can be noted from vignette.
Treatment is as follows:
 Underlying condition treatment (autoimmune disease will be immunosuppressed)
 If no improvement  administer EPO to stimulate RBC production
 If no improvement  packed RBC transfusion to get RBC count back up

4389: Suspicion of Colorectal cancer: signs/symptoms


 An older man (>60) with abdominal pain, microcytic anemia, (+)fecal occult blood, and hepatomegaly
with a hard liver edge  prompt you to think infiltrative colon cancer
o Any old person with microcytic anemia (esp. men) or blood in stool  colon cancer
o Hepatomegaly with a hard liver edge  liver cancer (most often from metastatic colon cancer)
o Mildly elevated AST/ALT/Alk.phos in this setting  infiltrative/cholestatic disease (which
metastatic liver cancer would be a part of!)
 Dx: because this pt has signs of colon cancer and liver abnormalities without significant elevations in
LFTs, an abdominal CT should be sought as metastatic malignancy is suspected.

2210: Dr. Jones’s words echoed in Mike’s head, “An old person with iron deficiency anemia, especially a man,
has a GI bleed until proven otherwise.”
 The first step is a fecal occult blood test  if positive, then it’ll guide further investigation, but if
negative it means NOTHING!
 If you have this clinical scenario and you get a negative FOBT, you then get endoscopy/colonoscopy on
the patient for concern of GI malignancy
Note that radioisotope erythrocyte scintigraphy is only useful in active, significant bleeding. If it’s a slow or
non-active bleed, that test won’t tell you shit.

3926: Although weird, some older patients may have an “idiopathic anemia of the elderly”. Classically these
patients have multiple comorbidities (CHF, etc.) that make them poorly tolerant to anemia

4348: Pica/Pagophagia
 Pica – appetite for substances other than food (clay, dirt, or paper are common)
o May have exotic appetite (hair, light bulbs, etc.) when associated with psychiatric disease
 Pagophagia – appetite for ice
 Both classically associated with iron deficiency anemia and may appear before iron deficiency is
present, thus any patient with these signs/symptoms should have bloodwork done

4356/4350/3067/3066: Pernicious Anemia


 B12 deficiency caused by anti-intrinsic factor antibodies causing destruction of parietal cells/intrinsic
factor resulting in failure of B12 uptake in the terminal ileum
o Note that anemia is due to impaired DNA synthesis of thymidylate and purine molecules for
RBC production (ineffective hematopoiesis)
 Presentation: macrocytic anemia (pale conjunctiva, fatigue, pallor, MCV >100), glossitis (beefy red
tongue), and peripheral neuropathy/loss of lower extremity vibratory sensation (dorsal column
destruction), or cognitive function defects. Sometimes jaundice from destruction of large RBCs
o Smear: mactocytic RBCs, hypersegmented neutrophils
o Classically associated with atrophic gastritis (parietal cells being attacked as they make intrinsic
factor) increasing the risk for intestininal-type gastric cancer and gastric carcinoid tumors.
 Dx: B12 level, testing for autoantibodies, Homocysteine level/Methylmalonic acid level to differentiate
from folate deficiency
 Tx: B12 injections + periodic stool blood testing for gastric cancer monitoring
4357: Differentiating B12 from Folate deficiency anemia
 Both show elevated homocysteine (both involved with conversion of homocysteine  methionine)
 Only B12 def will show elevated methylmalonic acid (conversion of methylmaonyl-CoAsuccinyl-CoA)
 Note that folate deficiency is more common and should be suspected in any malnutrition picture first
as you can become deficiency in 5-10 weeks

4147: Several drugs can cause megaloblastic anemia due to folate deficiency [HIGH YIELD]
 Anti-epileptics (phenytoin, phenobarbital, primidone)  impair folate absorption in the gut
 Bactrim (TMP-SMX)  inhibits dihydrofolate reductase, thus decreasing folate activation
 Methotrexate  inhibits dihydrofolate reductase (leucovorin [folinic acid] is indicated as a concurrent
supplement when giving methotrexate because of this)

3086: Macrocytic Anemia due to Nutrient Deficiency


 Folate: found in leafy green veggies/liver; destroyed by cooking; depleted within 4-5 months; more
common to get through diet
 B12 (cobalamin): found in all animal products (strict vegans); depleted within 3-4 years; typically, this
is due to pernicious anemia (anti-parietal antibodies) NOT diet

2872: Note that basophilic stippling is not as specific as you might think, and that it can occur in alcoholism
(may be part of folate deficiency macrocytic anemia), lead poisoning, and thalassemia.

4330: Note that megaloblastic anemia due to B12 deficiency that’s only treated with folate supplementation
will result in worsening or emergence of neurologic symptoms associated with B12 deficiency. This id
because the folate will aid in RBC production, which will use up the dwindling B12. Thus megaloblastic anemia
should always be treated with folate AND B12.

4468: Oddly enough, patients with sickle cell disease are at higher risk for folate deficiency and megaloblastic
anemia. Because there is chronic hemolysis due to poorly deformable sickle cells, there is high RBC
turnover/production meaning there’s a high demand for folate/B12. These patients can easily become folate
deficient thus  a sickle cell patient (or any chromic hemolytic patient) presenting with macrocytic anemia
should first be suspected for a folate deficiency.
 Daily folate supplementation is recommended for any person with Sickle Cell Disease!

4037: Sideroblastic Anemia


 Anemia resulting from defective heme synthesis from problem with pyridoxine (Vit B6) dependent
synthesis of protoporphyrin.
 Presentation: normocytic to anemia (fatigue, pallor, etc.)
o Labs: increased iron with decreased TIBC (poor RBC function, but iron is OK)
o Smear: dimorphic RBC population
 Hypochromic population –some bone marrow failing to make proper proteins
 Normochromic population – some bone marrow that’s still doing OK
o Classically this can happen with tuberculosis treatment with isoniazid, alcoholism, drugs, etc.
 Dx: clinical presentation, can be confirmed with bone marrow biopsy showing ringed sideroblasts
 Tx: pyridoxine (B6) administration
4349/3978: In chronic kidney disease, anemia can develop as the kidney is responsible for erythropoietin
(EPO) production. The mainstay of anemia with chronic kidney disease is supplemental EPO + supplemental
iron. Iron is supplemented as EPO will surge RBC production, causing a high need for iron for new heme. Plus,
someone with chronic kidney disease may already be anemic from chronic inflammation!
 Iron supplementation alone should always be tried first before EPO
 Side Effects: BP increase (30%, >10mm diastolic; not well understood), Headaches, Flu-like syndrome,
Red cell aplasia (rare) [HIGH YIELD]
 Note that the goal of EPO administration during dialysis is to get Hb to 10-12 g/dL; higher than this has
been found to have worse outcomes (increased cardiovascular death)

4385: Indications for RBC transfusion


 Hb <7: always indicated
 Hb <7-8: heart surgery, heart failure, or cancer treatment patients
 Hb 8-10: any surgery, acute coronary syndrome, on-going bleeding, symptomatic anemia
 Hb >10: rarely indicated

Case 10 – Acute Diarrhea


Definitions
 Acute Diarrhea – persists for <2wk duration
 Subacute Diarrhea – persists between 2-4wk duration
 Chronic Diarrhea – persists for >4wk duration
Presentation: diarrhea and/or vomiting, sometimes with fever, and abdominal pain
 Determine volume status - membranes moist or dry, skin turgor, or capillary refill time
 Determine Oral intake ability – are they vomiting too much to keep anything down?
 Get history – changes in stool, sick contacts, other symptoms, recent travel, recent meals, etc.
Dx: clinical presentation
 Lab: stool sample for leukocytes, culture, ova/parasites, or C.diff toxin
 Note that most acute diarrhea is viral and does not need further workup, only supportive care
Tx: typically consists of re-hydration, anti-secretory therapy, and antibiotics or probiotics if necessary
 Mild: often can tolerate oral intake; if not able to drink, then pt should be hospitalized for IV fluids
 Severe: profuse/blood diarrhea, severe pain, fever, children/elderly/immunocompromised should all
be hospitalized for supportive care and treatment
Quick Tips
 Anti-secretory drugs aid symptoms but do NOT speed length of recovery time
 Best prevention of viral diarrhea is hand washing and staying home from school for recovery
 Serve food immediately from refrigerator or after cooking; room temp = bacterial growth
 Note that clindamycin is classically associated with C.diff development and pseudomembranous colitis;
however any antibiotic may be associated with this development
Traveler’s Diarrhea
 E.coli is the most common etiology (classic Hx of trip to Mexico)
 Best to avoid local water/ice cubes/raw fruits and vegetables
 Prophylaxis is typically ciprofloxacin 500mg BID for 1-3 days
 If a pregnant woman or child: azithromycin 1000mg one-time (adult) or 10mg/kg for 3 days (child) as
floroquinolones may damage development of fetal/child bones and connective tissue
 Rifamixin 200mg TID for three days may also be used, although less effective
 Classically bismuth subsalicylate has been used as prophylaxis, BUT the large quantities needed
predispose to salicylism, thus should be avoided.

Case 10.5 – Diarrhea Tips from UWorld


3917/3590: Diarrhea in HIV/AIDS typically occurs with different etiologies based on CD4+ count
 Cryptosporidium (<180) – severe watery diarrhea, wt loss, fever
o Classically from animal contact, water, or person-person
o Dx: modified acid-fast stain shows cryptosporidial oocytes (4-6mcg)
o Tx: anti-retroviral therapy (resolves with CD4 receovery)
 Microsporidium or isosporidium (<100) – watery diarrhea, wt loss, cramps, no fever
 Mycobacterium Avium Complex (<50) – watery diarrhea, wt loss, high fever (>102F)
 CMV (<50) – bloody/small volume diarrhea, abdominal pain, wt loss, fever
o Dx: colonoscopy with biopsy – intracytoplasmic eosinophilic/basophilic inclusions
o Tx: ganciclovir
o Must examine eyes to rule out CMV retinitis

2269: Profuse watery diarrhea in HIV/AIDS patients is typically from a opportunistic infection (bloody would
be more high infectious things).
 First step in diagnosing what is causing the diarrhea is stool examination for ova/parasites as often
parasitic infections will be the cause here!

3613: Typically, if a post-transplant pt shows lung problems (cough, dyspnea, CXR infiltrates) and GI problems
(abdominal pain, dysphagia, diarrhea) you should immediately consider CMV infection.
 Bone marrow suppression (fatigue, cytopenias), and myalgias are other possible manifestation
 Dx: bronchoalveolar lavage with PCR for CMV
 Tx: IV acyclovir

3249: Traveler’s diarrhea is not uncommon, however it often lasts a <1 week and remits. If it lasts >2 weeks
parasitic infections should be considered:
 Cryptosporidium, Cystoisospora, Giardia or microsporidia spp are major parasites to be considered.
 Dx: stool examination should reveal parasites

3248: In normal, healthy folks, there are TONS of reasons why you’d get GI problems and the specific organism
is based off the symptoms and cultures (often USMLE wants to diagnose based off scenario!) So here’s some
classics to remember:
 B.cereus  diarrhea within 6 hours of eating re-heated rice
 S.aureus  rapid onset of vomiting (within 6hr) and sometimes diarrhea; classically from mayonnaise
 C.difficile  post-antibiotic Tx with diffuse, watery stools
 C.perfringens  brief cramps, fever, diarrhea associated with unrefridgerated food
 V.vulnificus  raw/undercooked shellfish causing vomiting/cramp/diarrhea
o May be life-threatening with invasive infection in immunocompromised!
 E.coli  watery or bloody (shiga-toxin) diarrhea typically from undercooked hamburger beef
 Shigella  bloody diarrhea/bacteremia classically with travel outside the US or undercooked chicken
 Salmonella  someone who ate undercooked chicken or mayonnaise
 Campylobacter  bloody diarrhea, can cause Guillian-Barre
 Rotavirus  classically diarrhea in the winter and exposure to kids (often daycare workers)
3250: Foodborne Illnesses are wide-ranging with a huge amount of possible presentations. We can narrow
down possible bugs by the symptoms of the patient:
 Vomiting predominates: S.aureus, B.cereus, or Norovirus
 Watery diarrhea predominates: C.perfringes, ETEC, Cryptospoidium, Cyclospora, interinal worms
 Inflammatory (bloody) diarrhea predominates: Salmonella, Campylobacter, EHEC, Shigella,
Enterobacter, Vibrio spp, Yersinia spp.
 Non-GI symptoms predominate: you’ll typically know these based on symptoms/history

3887: Giardiasis (Giardia Lamblia or G.intestinalis parasite)


 Presentation: profuse, watery, foul-smelling diarrhea with bloating/abdominal cramps;
malabsorbtion/weight loss can occur in more long-standing infections
o Transmission is classically after a trip abroad/camping trip through contaminated water/food
or fecal-oral route
 Dx: Stool-antigen assay (immunoflourscence or ELISA) for oocysts or trophozoites
 Tx: oral Metronidazole

Case 11 – Adult Female Health Maintenance


[Ch.2 The OBGYN Role in Screening/Preventative Care – Topics 3 & 7]
The HPV vaccine is nearly 100% effective in prevention of HPV strains it’s against
 Vaccination age range is 13-26 for females AND males
 Best if given before sexual activity, but sexually active people can still benefit
 Do not give vaccine during pregnancy
 Do give vaccine during breastfeeding
The Pap smear (cervical cancer) and mammography (breast cancer) are the only acceptable screening tests for
gynecologic cancers
 Note that there’s no acceptable screening test for ovarian, endometrial, vaginal, or vulvar cancers and
a biopsy of the region is NOT considered a screening test
 Leading causes of cancer death in women: 1Lung, 2Breast, 3Coleorectal
Breast Cancer
 Clinical breast exams and screening mammography are effective screening tools
 “Breast self-awareness” should be emphasized; shown to increase detection rates of breast problems
 Regular self breast exams are NOT recommended anymore
 ACOG recommendations:
o Age 20-39: clinical breast exam every 1-3yrs
o Age 20-39 (high risk): consider annual clinical breast exams
o Age 40+: annual clinical breast exam + annual screening mammography
o Age 40+ (high risk): regular recs + annual MRI of breast tissue
Cervical Cancer
 Cervical Intraepithelial Neoplasia (CIN) is the precursor to cervical cancer. CIN 1 often spontaneously
resolves, but CIN 2 or CIN 3 will progress to cervical cancer and must be treated.
 Pap Smear (exfoliative cytology + papanicolaou test) recommendations:
o Age <21: no screening (regardless of sexual activity)
o Age 21-29: screening with cytology every 3 years
o Age 30-65: screening/cytology + HPV co-testing every 5yr (preferred) –OR– screening/cytology
every 3 yr
o Age 65+: no screening if >3 consecutive negative tests + no hx of CIN 2
o Special Situations:
 Hysterectomy with removal of cervix: no screening if removal was for benign reasons
and no Hx of CIN 2 or greater
 Hx of CIN 2 or greater: screening continues 20yrs after regression, then resume normal
screening guidelines
 HPV vaccination: follow normal guidelines
Colorectal Cancer
 Screening recommendations:
o Age 50: colonoscopy every 10 years
o Alternatives to colonoscopy:
 Annual Fecal Occult Blood test or Fecal Immunochemical testing
 Flexibal Sigmoidoscopy, Barium Enema, or CT colonography every 5 years
Sexually Transmitted Diseases (STDs)
 HIV
o All women should be screened at least once for HIV
o Women with the following risk factors should be offered annual testing/repeat testing:
 IV drug users, have partners who are IDVU or are HIV infected, exchange sex for
drugs/money, have had an STD in the past year, have had a new sexual partner
o ELISA blood screening is the most common screen
o Abnormal ELISA testing should be followed with a confirmatory Western Blot
 Chlamydia
o The most commonly reported STD in the USA
o Typically asymptomatic
o Screening guidelines:
 Age <25 and sexually active: annual screen (NAAT of endocervical/vaginal swab/urine)
 Age >26 and at risk: annual screen (NAAT of endocervical/vaginal swab/urine)
 Gonorrhea – Same screening as Chlamydia applies here
 Syphilis
o All ‘at risk’ women should be screened annually
o All pregnant women should be screened:
 During gestation as early as possible (so intervention can occur)
 Again during 3rd trimester if from high-risk population
 Again during delivery
o Screen is the VDRL or RPR with confirmatory FTA-ABS or other testing
Metabolic/Cardiovascular Diseases
 Osteoporosis
o Screening guidelines
 DEXA scan with BMD/FRAX at age 65
 Testing at younger age if at increased risk for fracture (alcoholism, smoking, hx of
fragility fracture, body weight less than 127lb, medical cause of bone loss, rheumatoid
arthritis)
o Bone Mineral Density (BMD) – an indirect measure of bone fragility assessed with a DEXA scan.
Basically a person’s BMD is charted against population data to find the T-score/Z-score
 T-score BMD – based on young, healthy adult population
 Z-score BMD – based on reference population that patient fits within
 Normal BMD is > -1
 Osteopenia BMD is between -2.5 and -1
 Osteoporosis BMD is < -2.5
o Fracture risk assessment tool (FRAX) – assessment tool used to determine the risk of a women
>40 with low BMD’s risk of a bone fracture in the next decade
o Preventative measures include, Ca2+/Vit.D consumption, regular weight-bearing exercise,
smoking cessation, decreasing alcohol intake, and prevention of falls

 Thyroid Disease
o TSH at age 50, with repeat every 5 years (may present as dementia in older adults!)
o Earlier if risk factors or clinical signs of hypothyroidism
 Diabetes Mellitus – Screening fasting blood glucose at age 45, with repeat every 3 years
 Hypertension – screen with blood pressure reading annually in all women >13 years
 Lipid Disorders – screen with lipid profile starting age 65, with repeat every 5 years
 Obesity – measurement of height/weight with BMI calculation should be with any physical exam

Case 12 – Minor Sprains and Strains


Basic Principles of Musculoskeletal Injury Evaluation
 Inspection (swelling, bruising, deformity, use of support devices, mobility of joint)
 Palpation (pain localization, muscle tightness, swelling, sensation, temperature, pulses, capillary refill)
 Passive Range of Motion (mobility of the joint/joint damage)
 Active Range of Motion (muscle/tendon function, neurovascular function)
 Specific Testing (depends on joint, listed at end of section)
 Imaging (X-ray is first choice, & MRI used for soft tissue damage)
Ankle
 Presentation: pain, swelling, redness, warmth, or loss of range of motion, typically following injury
o Classically the lateral ankle is weaker and more susceptible to injury (Anterior talofibular
ligament [ATFL] > calcaneofibular ligament > posterior talofibular ligament)
o The medial ankle is much more durable due to the deltoid ligament
 Dx: clinical, but requires staging and possibly imaging
o Staging: 1 (stretching of ATFL) 2 (partial tear ATFL, sprain CFL) and 3 (complete tear ATFL/CFL)
o Ottawa Ankle Rules: nearly 100% sensitive in ruling out malleolar/midfoot fractures. If any of
these conditions are present, then a plain X-ray should be undertaken.
 Bony tenderness of posterior edge or tip of medial or lateral malleolus
 Inability to bear weight on affected ankle for 4 steps
 Tenderness to palpation of the navicular bone or base of 5th metatarsal
 Tx: PRICE (Protect, Rest, Ice, Compress, Elevate) + NSAIDs or Acetaminophen + Physical Therapy
o PT should be initiated 48-72hr following the injury at tolerated. The biggest thing causing lasting
mobility problems is not initiating PT early enough!
Knee
 Ottawa Knee Rules: strong sensitivity/specificity; imaging should occur if any of these apply:
o Pt age 55yr or older
o Isolated patellar tenderness
o Tenderness at the head of the fibula
o Inability to flex the ankle to 90o
o Inability to bear weight on affected knee for 4 consecutive steps (even if there’s a limp)
Specific Rotator Cuff Testing
 Empty Can Test (supraspinatus) – outstretched “thumbs down”, don’t let doc press hands down
 Lift-Off test (subscapularis) – dorsum of hand on lower back, lift off from back
 Internal Rotation (subscapularis) – elbows at 90o, rotate hands toward stomach
 External Rotation (infraspinatus/teres minor) – same at internal but rotate outward
 Hawkins Impingement (subacromial impingement of supraspinatus tendon) – shoulder and elbow
flexed. Then internal rotation of the shoulder in that position
 Drop-arm test (large rotator cuff tear) – lift arm into air and slowly adduct the arm
Specific Ankle Testing
 Anterior drawer (ATFL) – stabilize shin and pull ankle anteriorly
 Inversion stress test (CFL) – stabilize shin and invert the ankle
 Squeeze test (syndesmosis; high ankle sprain) – squeeze tibia/fibular at midcalf
Knee
 Lachman test (ACL) – knee at 20o, pull forward to attempt to mobilize the knee
 Anterior Drawer (ACL) – knee at 90o, pull forward to attempt to mobilize the knee
 Posterior Drawer (PCL) – knee at 90o, push backwards to attempt to mobilize the knee
 Valgus Strain (Medial Collateral [MCL]) – stabilize shin 30o flexion and apply force to left side of knee
 Varus Strain (Lateral Collateral [LCL]) – stabilize shin 30o flexion and apply force to right side of knee

Case 13 – Evaluation of a Nevus + Tips from UWorld


2762/2603: Basal Cell Carcinoma
 Most common, often an excellent prognosis. Correlates with cumulative sun exposure
 Presentation: pearly/rolled edge nodule with keratinized or ulcerated center. Sometimes featuring
spider angiomata on the lesion, oozing or crusting
 Dx: clinical picture and biopsy
 Tx:
o Superficial – electrodessication or cryotherapy
o Deep – MOHS surgery

2604/4033/4319: Squamous Cell Carcinoma


 Intermediate incidence with intermediate prognosis, typically not metastatic when found, but will
invade if left untreated. Correlates with # of severe sun exposure events (burns, tanning beds, etc.)
 Presentation: rough scaly plaque/nodule with possible hyperkeratosis/ulceration/bleeding. Classically
can cause neurologic damage due to perineural invasion (numbness/parastheisa)
o Sunburns, radiation, immunosuppression, burns, scars, and warts are all risk factors
o Note that if you’ve a lesion on the lower lip (more sunlight exposure) is SCC
o Note that a Marjolin ulcer is an SCC arising from the site of a scar/burn/previous injury and has
an increased risk of metastases
 Dx: skin biopsy that includes the deep reticular dermis (assess depth of invasion) showing
dysplastic/anaplastic keratinocytes/keratin pearls
 Tx:
o Low risk: excision or local destruction (cryotherapy/electrodessication)
o High risk or cosmetic areas: Mohs micrographic surgery

2767/2765/4312: Malignant Melanoma


 Least common skin cancer, but most deadly
o Superficial Spreading – most common type; shallow penetration with spread radially in
superficial skin layers. Often raised with brown/pink/white-grey/pink. Common in both sexes,
but men get it on their back) and women get in on their legs
o Lentigo Maligna – least common subtype. A tan-brown lesion with very irregular borders.
Classically on the elderly (most commonly diagnosed in the 7th decade of life)
o Acral Lentiginous – flat irregular brown-black lesions on the palms of the hand or under the
nailbed. Most common in Black and Asian people.
o Nodular Melanoma – most aggressive and often invasive at time of diagnosis. Brown-black with
nodular appearance; often grow from existing nevi.
 Presentation: new or existing melanotic skin lesion demonstrating the ABCDEs (Asymmetry, Raggy
Border, verigated Color, Diameter >6mm, Elevation/Evolution)
o Considered the most common malignancy of women age 25-29
o Women typically get it on the legs while men get it on the back
 Dx: excisional biopsy to show depth of invasion (most important diagnostic factor)
o Rarely malignant with <1mm depth of invasion
 Tx: surgical excision with wide (1cm) margins
o We actually wait for this as the wide excision may disrupt lymphatic flow

2764: Seborrheic Keratoses


 Benign lesion that tends to favor the face/trunk/old people and is often consider unsightly
o May be indicative of underlying GI malignancy as the Sign of Leser-Trélat where many of these
lesions appear rapidly.
 Presentation: waxy/greasy, “stuck-on”, well circumscribed lesion that may be flat or raised with normal
surrounding skin. Can vary from pink/white to pale to brown/black. Typically, slowly enlarging.
 Dx: clinical
 Tx: none needed except for cosmetic reasons

4410: Actinic Keratosis


 Presentation: hyperkeratosis causing “sand-paper like” papules in areas of sun exposure. Often begin
small/flat but may accumulate becoming “cutaneous horns”. Pre-malignant for SCC
 Dx: shave biopsy with light microscopy showing characteristic cell proliferation
 Tx: electrodessication or cryotherapy

Case 14 – Asymptomatic Hematuria + Tips From UWorld


Hematuria found on routine urinalysis can be a sign of something further and needs to be investigated. A short
list of things that could cause hematuria includes:
 Recent UG/genital trauma, urinary infection, renal disease/tumors/calculi, BPH, exercise, menses, etc
 Presentation: laboratory finding of 3 or more RBCs should prompt proper work-up
o Gross hematuria (specimen is visibly discolored from blood) or microscopic hematuria (gross
sample looks normal, but RBCs detected on analysis)
o Eosinophiluria is classically due to Acute Interstitial Nephritis (drug-induced)
 Risk factors for intrinsic kidney disease are: smoking, occupational hazards (benzenes/aromatics), age
>40, Hx of gross hematuria/urologic disease/UTI/analgesic abuse/pelvic irradiation
 Dx: First step: Repeat urinanalysis; if persistent, then proceed with further workup
o Basic labs (CBC and BMP including serum creatinine)
o Microscopic evaluation of sample/urinary sediment
o Cytology and culture of urine sample
o Imaging (often CT abdomen & pelvis) of upper and lower urogenital tracts; IVP, retrograde
pyelogram, ultrasound, CT, CT urography, and cystoscopy are all options for imaging
 Note that if giving any IV contrast; renal function needs to be assessed PRIOR to
administration with proper hydration +/- N-acetylcysteine or IV sodium bicarb
 Tx: depends on etiology of hematuria
o If urine culture if positive; give antibiotic treatment; no need for further follow-up if hematuria
resolves with Abx
o Exercise induced microscopic hematuria should resolve within 72hrs
o If workup if unrevealing, then follow-up blood pressure, urinalysis, and urine cytology at 6, 12,
24, and 36 months should be undertaken. If these are normal, then no further follow-up is
needed.
Note that gross hematuria always deserves a full workup! Even in a bodybuilder who has been working out.

3955: Classic Urine Casts [HIGH YIELD]


 Muddy Brown cast – acute tubular necrosis
 RBC cast – glomerulonephritis
 WBC casts – interstitial nephritis or pyelonephritis (inflammation in the kidney)
 Fatty casts – nephrotic syndrome
 Broad casts or waxy casts – chronic renal failure

4491: Contrast-Induced Nephropathy


 Renal vasoconstriction/tubular injury due to administration of IV contrast for imaging procedures
 Presentation: spike in creatinine following contrast administration, with resolution around 5 days later
o Pt with impaired renal function (diabetes, chronic renal insufficiency, etc.) are at high risk
 Dx: clinical presentation
 Tx: supportive care for recover, prevention is key
o Pre-contrast hydration with IV fluids (isotonic bicarb or normal saline)
o N-Acetylcysteine may be used if concern is high (vasodilation/antioxidant)

3061/2242: Acute Interstitial Nephritis


 Typically drug-induced damage to interstitial tissue in the kidney
 Presentation: acute renal failure, fever, macropapular rash, classically following offending drug use (5
days to several weeks following drug), and sometimes arthralgia
o Urinanalysis: Pyuria, hematuria, eosinophiluria, and WBC casts
o Biopsy: inflammatory infiltration and edema
 Dx: clinical presentation with agreeing labs
o Drugs: penicillins, TMP-SMX, cephalosporins, rifampin, diuretics, or NSAIDs
 Tx: discontinue offending drug, glucocorticoids, and supportive care

4807: Chronic Interstitial Cystitis (Painful Bladder Syndrome)


 Poorly understood disorder most commonly in women with anxiety/pain syndromes
 Presentation: suprapublic pain with bladder filling relieved with voiding, increased urinary
frequency/urgency, and dyspareunia/pain with exercise/pain with alcohol consumption
 Dx: bladder pain for >6 weeks without attributable cause with normal UA
 Tx: psychiatric therapy, amitriptyline, and analgesics for exacerbations

2241: Post-Streptococcal Glomerulonephritis


 Autoimmune disease in response to a streptococcal throat/skin infection occurring 10-20 days post-
infection. Classically happens in kids and young adults.
 Presentation: Fever, rash, arthralgia, periorbital edema, hypertension
o Urine: oliguria, proteinuria, hematuria, sterile pyuria, eosinophiluria, WBC casts
o Blood: eosinophilia, low serum C3 compliment levels
o Biopsy: sub-epithelial humps due to complement deposition
 Dx: clinical presentation with elevated ASO titer or anti-DNase B antibodies
 Tx: supportive care, should resolve on its own
o Prognosis good in children but very bad in adults often leading to renal failure

2230/10287: IgA nephropathy


 Autoimmune deposition of IgA in the meconium of kidney causing damage 5 days after URI, classically
in young men. Can recur any time after an URI as it’s the result of over-activation of the normal
immune response. Does not occur with skin infections.
o The most common glomerulonephritis in adults; remember that damage occurs in glomerulus!
 Presentation: recurrent gross hematuria following a URI
o Labs: normal serum compliment levels
 Dx: presentation. IgA demonstrated deposition in renal biopsy
 Tx: supportive care (often benign) with close watch for progression

2229: Myoglobinuria
 Acute renal failure following diffuse muscle damage, classically after a tonic-clonic seizure
 Presentation: acute elevation of creatinine, often with hypertension/tachypnea
o Urinanalysis: large amount of blood with few RBCs on microscopy
 Dx: clinical picture with classic disparity in urinanalysis (RBCs is detection of hemoglobin OR myoglobin
thus, a standard UA will say “large RBCs amount” but microscopy will show few RBCs/hpf)
 Tx: hydration and supportive care until return of baseline renal function

4006: Never forget, rifampin causes red/orange urine discoloration as well as discoloration of tears, sweat,
and saliva. Classically it’s known to stain contact lenses.
 It’s important to do a simple urinalysis to rule out more serious conditions.

Case 15 – Thyroid Disease + Tips from UWorld


Hyperthyroidism
 Signs: progressive nervousness, difficulty concentrating, palpitations, weight loss, moist skin, fine
resting tremor, dyspnea on exertion, diarrhea, and heat intolerance
 Symptoms: Rapid pulse, elevated BP (systolic > diastolic), and rarely atrial fibrillation
Hypothyroidism
 Signs: lethargy, depressed affect, sluggish mentation, forgetfulness, weight gain, dry skin, constipation,
cold intolerance; may be confused with depression (women) or dementia (elderly)
o 1o hypothyroidism will show an high TSH but low T3/T4 (poor thyroid function)
o 2o hypothyroidism will show a low TSH  TRH administration is used
 A normally functioning pituitary will respond and TSH will rise. If it doesn’t then there’s
something wrong with the pituitary (get imaging of the pituitary)
 If the pituitary responds then there may be something wrong with the hypothalamus
(not making TRH, thus no TSH production)
 Symptoms: low blood pressure, bradycardia, non-pitting edema, hair loss, dry/cracked skin, slowly
relaxing reflexes, and rarely myoedema
 Tx: thyroid hormone replacement (typically 1.7 mcg/kg daily)
o Children (use a lot for growth) and pregnant women (increased TBG) may need increased doses
o Elderly (decreased TBG) may need decreased doses
o TSH is regular follow-up for adequate response 4 weeks after any changes in dose
4324/4588: Evaluation of Hyperthyroidism
 Any suspicion of thyroid problems should begin with TSH/T4 lab values
o TSH low/T4 normal with signs of hyperthyroid  measure T3
 Normal T3  subclinical hypothyroid, early pregnancy, non-thyroid problem
 High T3  If signs of Grave’s disease, it’s Grave’s disease. If no signs, do a Radioactive
Iodine Uptake (RAIU) scan
o TSH low/T4 high with signs of hyperthyroid  consider primary hyperthyroidism
 If signs of Grave’s Disease, then you’re good; if not then RAIU scan
o TSH normal - high/T4 high with signs of hyperthyroid  consider secondary hyperthyroidism
 MRI of pituitary for TSH secreting tumor
 Radioactive Iodine Uptake (RAIU) scan performed when hyperthyroid is considered, but Grave’s
disease is not likely due to symptoms
o Low uptake  measure thyroglobulin (Tg) levels
 Low Tg  likely exogenous use of thyroid hormone
 High Tg  thyroiditis, extra-glandular production, or iodide exposure
o High uptake diffuse  Grave’s disease
o High uptake nodular  Toxic adenoma, or multinodular goiter

4324: Central Hyperthyroidism (TSH secreting adenoma)


 Presentation: signs of hyperthyroidism (weight loss, tachycardia, palpitations, etc) with tumor mass
effect signs (headache, visual symptoms, impaired secretion of other pituitary hormones)
 Dx: TSH high with high T4
 Tx: surgical excision or ablation

4415/3488/4726/4132/2179: Grave’s Disease


 Presentation: hyperthyroidism, exophthalmos, diffusely enlarged thyroid gland
o Low TSH with high T3 or T4
 Dx: clinical picture with confirmed TSH receptor-antibodies
 Tx: three major options
o Radioiodine ablation – basically goes in and destroys the thyroid gland cells. Apparently the
point of this treatment is to become euthyroid and complete destruction is NOT intended.
However, pts very often have the side-effect of hypothyroidism.
 Initial worsening of symptoms/exophthalmos (10%) in first week or so due to release
of stored thyroid hormone from cell destruction. Often OK in young, healthy pts.
 Pre-treatment with glucocorticoids often minimizes treatment symptoms
 Hypothyroidism (80%) may develop, and is most likely in Grave’s Disease as the entire
thyroid gland is dysfunctional, instead of just part.
o Surgery – removal of the thyroid is always on the table
 Risk of damage to recurrent laryngeal nerve or hypoparathyroidism due to damage or
unintentional removal of parathyroids
o Propylthiouracil (PTU) or methimazole (MMA) – thioamides that block production of T3/T4
 MMA a teratogen in the 1st trimester, may cause cholestasis
 PTU can cause hepatic failure, ANCA-associated vasculitis, and agranulocytosis
 Does not permanently cure disease. Typically used in pts that cannot tolerate other
treatments (old people that cannot handle worsening symptoms or surgery)
2178/3487: The life-threatening side-effect of Propylthiouracil/Methimazole is agranulocytosis (0.3%). This
classically presents with fever and sore throat within 90 days of treatment initiation. Should this occur:
1. Stop PTU/MMA immediately and check WBC levels.
a. WBC >1,500 – PTU isn’t causing the problem
b. WBC <1000 – PTU should be discontinued for life
2. Get throat culture, give oral penicillin/acetaminophen, administer broad spectrum antibiotcs before
even knowing the result as lack of immune response could be fatal. Pseudomonas is especially
worrysome in these patients

3952: Thyrotoxicosis can largely affect the heart causing changes in rhythm (atrial fibrillation, PACs/PVCs, and
sinus tachy), hemodynamics (increased CO  systolic HTN and increased myocardial O2 demand), heart
failure, and angina.

4307: Hyperthyroid Bone Disease


 Excess thyroid gland activity stimulate osteoclast activity, thus causing bone resorption. The resulting
hypercalcemia inhibits PTH production ultimately leading to hypercalciuria and net calcium wasting.
 Brittle bones, tendency toward bone fracture, and decreased bone density are all possible problems!
 Note that a hot nodule (functional, likely toxic goiter) is nearly NEVER NEOPLASTIC, as it’s an area of
isolated hyperacitivty/hypertrophy. A cold nodule is an area of low activity but growth, which is high
suspicious of thyroid cancer.

4286/10958/11277: Thyroid Storm


 Presentation: high fever, cardio (tachycardia, HTN, CHF, atrial fibrillation), CNS (agitation, delirium,
coma), hyperthyroid (goiter, lid lag, tremor), GI (nausea, vomiting, diarrhea, jaundice)
o Typically caused by: thyroid/non-thyroid surgery, acute illness, childbirth, or acute iodine load
(such as in IV contrast!)
 Dx: clinical presentation with precipitating factor
 Tx: propranolol (decrease adrenergic effects), propylthiouracil (decrease thyroid hormone
production), glucocorticoids (suppress TRH/T3T4 conversion), and ID trigger to treat/remove

3483: Metabolic changes in hypothyroidism include hyperlipidemia (decrease in LDL receptors),


hyponatremia, and asymptomatic elevations of CK (>10x normal).
 Note that hyperlipidemia may persist for months despite adequate replacement therapy
 Statins will increase the chance of hypothyroid myopathy, thus are used with caution

3878/4382: Hypothyroid Myopathy


 Presentation: normal hypothyroidism signs with elevated CK (mild) to rhabdomyolysis/polymyositis
o CK elevation can be present years before clinical signs of hypothyroidism
o Typically, normal reflexes
 Dx: any pt with unexplained elevated CK/myositis should have TSH/T4 checked as this may be the
manifestations of hypothyroidism early on.
o If normal, then muscle biopsy should be considered
 Tx: treat underlying cause

3495: Euthyroid Sick Syndrome (“Low T3 syndrome”)


 Presentation: acute, severe illness accompanied by abnormal thyroid function tests
o Labs: low T3 with normal T4 and TSH
o Result of of decreased deiodination of T4 from caloric deprivation, glucocorticoids, and acute
phase reactants
 Dx: typically thought to be hypothyroidism, but often thyroid levels are unreliable in acute illness
 Tx: treatment of underlying disease should resolve the abnormal tests

3496: Thyroiditis
Hashimoto’s Thyroiditis
 Presentation: predominantly hypothyroid symptoms with non-tender diffuse goiter
 Dx: Anti-thyroid peroxidase (TPO) or anti-thyroglobulin antibodies in high-titer
o Note that nearly 10% of the normal population is (+) for anti-TPO antibodies
o The most common cause of hypothyroidism is the USA
Silent (painless) Thyroiditis
 Presentation: variant of Hashimoto’s with a mild, brief hyperthyroid phase (destruction and release of
T3/T4), with a small, non-tender goiter and slowly going back to euthyroid
 Dx: Anti-TPO antibodies and low radio-iodine uptake
Subacute (DeQuervain’s) thyroiditis
 Presentation: post-viral inflammatory disease causing prominent fever / hyperthyroidism with
painful/tender goiter
 Dx: presentation with elevated ESR/CRP and low radioiodine uptake

2191: Struma Ovarii


 Rare thyroid hormone producing ovarian teratoma. Classically in women over age 40 with pelvic mass,
ascites, and abdominal pain

4318: Toxic Adenoma


 Presentation: thyrotoxicosis with single hot thyroid nodule on exam
 Pathophys: activating mutatinos in the TSH receptor (auto-activation)
 Dx: demonstration of single, hot nodule on radioactive iodine scan
o Note that toxic multinodular goiter would have a patchy distribution on RI scan
 Tx: ablation or removal

3498/3499/3497: Classes of Thyroid Malignancy


 Papillary – epithelial origin, unencapsulated, most common, best prognosis
o Typically metastasizes to the lymph nodes
o Increased risk from family history and childhood radiation of head/neck
o FNA often reveals large cells with pale, grooved nuclei
o Often demonstrates Psamomma bodies and may demonstrate Hurthle cells
 Follicular - epithelial origin, encapsulated, decent prognosis
o Early hematogenous spread (lung/brain/bone) and invasion of thyroid capsule
o FNA often shows normal looking follicular cells. Invasion through capsule/hemotgenous
spread is necessary for diagnosis of carcinoma (vs adenoma), thus surgical diagnosis must be
made to see if invasion outside the capsule occurs
o May demonstrate Hurthle Cells
 Medullary – parafollicular c-cell origin, good prognosis, produces calcitonin, 80% are sporatic
o Associated with MEN2a/MEN2b; screening for these diseases is recommended
 Anaplastic – epithelial origin, rare, classically in old people
o Poor prognosis, death within 6 months typically
o May involve other adjacent structures (trachea/esophagus/etc.) due to rapidly enlarging
thyroid mass
 Primary Thyroid lymphoma – rare, typically in the setting of Hashimoto’s thyroiditis
o Has rapidly enlarging thyroid gland but good prognosis
 Note that because thyroid cancers are often indolent, it can be prudent to wait till after pregnancy to
evaluate and treat the problem in a pregnant woman

4154: Oddly enough, oral estrogen replacement therapy will increase Thyroid Binding Globulin (TBG) levels
due to decreased breakdown of TBG in the liver.
 Pts with a normal thyroid function will make more and compensate without trouble.
 Pts on levothyroxine for hypothyroidism will have to increase the dose (as the TBG increase will mean
less free T4 present). The increased dose will saturate the increased TBG, and restore euthyroidism.
 *transdermal estrogen does not have this effect as it bypasses the liver

Case 16 – Labor and Delivery/ROM


[Chapter 8: Intrapartum Care – Topic 11]
Early Signs of Labour
 Uterine contractions – frequent (occur every 3-5min) and uncomfortable transitioning in to painful
with cervical dilation are characteristic. Often pain radiates to the lower back/abdomen
o Braxton-Hicks contractions (false labour) are more sporadic and are not associated with labour.
These are less intense and can be relieved by hydration/ambulation/analgesia. Sometimes,
these can be distinguished from true contraction by being able to palpate through the uterus.
True uterine contractions make this palpation more difficult.
 “Lightening” – a sensation of the baby feeling lighter due to engagement of the baby’s head with the
pelvis. Mom may feel the baby is “dropping” and she can breathe more easily.
 “Bloody show” – with the loss of the mucus plus from cervical dilation, blood-tinged mucus will come
out of the vagina. This is a common sign that labour is occurring.

Evaluating for Labour


 Moms should be instructed to immediately contact their healthcare provider for four major reasons:
o Contractions occur every 5 minutes for at least 1 hour (corresponds with labour)
o Sudden gush of fluid from the vagina (corresponds with ROM)
o Significant vaginal bleeding (many reasons, often not good)
o Significant decrease in fetal movement (concern for fetal demise)
 Abdominal Examination should be done with the Leopold Maneuvers
o Fundal Determination – palpate the fundus (top) of the uterus on the belly to find either the
head (breech lie), or lower extremities (vertex lie) if the baby is laying longitudinal
o Locating the Small Parts – palpate down the uterus to either feel the spine or the hands/feet
o Identifying the Presenting Part – palpate suprapubically to find the presenting part. Then
manipulate the part. If the entire body moves, then it’s a breech; if only the presenting part
moves, it’s likely the more mobile head.
o Identifying the Cephalic Prominence – If this is palpable, the fetus is likely above the 0 station.
 Vaginal Examination – digital examination for effacement and dilation are periodically done to chart
the process of labour. The presenting part may also be palpated.
 Fetal Station – Ranges from –3 to +3 with 0 being the fetal head being at the level of the ischial spines.
When the baby’s head is at 0 station, it’s indicative that labour is progressing properly.

Stages of Labour
 First Stage – onset of labour  full cervical dilation (10cm); mom may feel the urge to push but should
be discouraged as the cervix may not be fully dilated
o Latent Phase – cervical effacement and early dilation
o Active Phase – more rapid dilation, typically starting around 4cm
 Second Stage – full cervical dilation  delivery of the infant; pushing is now encouraged. Molding or
caput succedaneum can occur, giving a false sense of descent.
o Episotomy MAY be used in certain situations but is not routinely recommended as there is
increased risk of 3rd/4th degree laceration
o Excessive traction should not be placed on the fetus for fear of brachial plexus injury
 Third Stage – delivery of the infant  delivery of the placenta; typically indicated by 1rising of the
uterus, 2change of uterus to a more globular shape, 3lengthening of the visible umbilical cord, and 4a
gush of blood. Traction should be light for fear of uterine inversion. 30 minutes allotted for this stage.
 Fourth Stage – after the delivery of the placenta when mom is undergoing physiologic adjustment; this
is the time where mom is at greatest risk for post-partum complications (esp. bleeding)

Mechanisms of Labour (“Fetal Cardinal Movements”)


 Engagement (descent of the fetal head into 0 station; nulliparous women typically have this days to
weeks before active labour, multiparous women typically have this with active labour)  Flexion (fetal
head flexes to move into mom’s pelvis)  Descent (movement into the birth canal; fastest rate occurs
in the active stage 1 of labour and stage 2)  Internal rotation (allows passage of baby’s head through
mom’s pelvis)  Extension (baby’s neck extends as it reaches the introitus)  External Rotation
(“restitution”; delivery of the head results in further head rotation into a ‘face forward’ position) 
Expulsion (rapid delivery of the body; this is why “catching a baby” is a thing)

Induction of Labour
 Oxytocin – helps stimulatie uterine contractions
 Cervical ripening – aiding in dilation of the cervix, especially if induction is not favorable
o Oxytocin or Prostaglandin E2 – vaginally administered, these medications help with dilation
and contraction of the uterus. May cause hyperstimulation if both are administered. Dual
administration is contraindicated in pts with prior c-section or uterine surgery due to increase
risk of uterine rupture
o Laminaria japonica rods – hygroscopic rods are inserted into the uterus and swell with
absorption of fluids, helping slow dilate the cervix. Not often used.
o 30mL foley catheter – works similarly to the L.japonicum rods
 Membrane Manipulation – “stripping” or “sweeping” the amniotic membranes. Not routinely
recommended as it increases the risk for infection as well as bleeding if an undiagnosed placenta
previa is present

[UWise Chapter 8 Questions]


 Braxton Hicks Contractions – irregular, uncomfortable contractions in the lower pelvis/groin without
cervical dilation or effacement. Can happen at any time in pregnancy (even at term!)
 Group B strep. recto-vaginal swab/culture is typically performed at 35-37wk and if positive, women
should be treated prophylactically during labour (GBS is fast growing; could grow back within 1 week!)
o If a woman has GBS bacteuria in pregnancy  no need to swab, just give Abx intra-partum
o If a woman has Hx of GBS infection in prior pregnancy  no swab, just give Abx intra-partum
o GBS prophylaxis does not need to be given if a C-section is done before membranes rupture
 Upon presentation to the hospital in labor, first steps are: review of the prenatal records with special
focus on the antenatal complications and dating criteria, a focused history and a targeted physical
examination to include maternal vital signs and fetal heart rate, and abdominal and pelvic examination.
o Other tests may be needed (nitrazine test, ultrasound, etc.) but these would be indicated by the
clinical scenario and are not routine
 Remember! Fetal Heart tones MUST be assessed during placement of an epidural (fear of fetal
bradycardia). If a pt cannot sit still for Doppler readings to be assessed during placement, a fetal scalp
electrode monitor is the way to go. Mom can move all she wants and you can still get proper readings
on baby to ensure epidural placement is OK.
 In this question, pt was presenting in labor with the fetus at +2. While initially, heart rates were fine,
the latest reading was that baby’s heart rate was in the 60s. In this situation:
o Measure mom’s heart rate to make sure Dopplers aren’t measuring her
o Encourage mom to push to try and finish the 2nd stage of delivery
o IF mom cannot complete delivery of the baby’s head within 1-2 pushes, opt for operative
forceps or vacuum-assisted delivery to expedite the process of labor
 In this question, you were trying to place an intra-uterine pressure catheter (IUPC) and 300cc of frank
blood/amiotic fluid came out! The next best thing to do is to confirm baby is OK, thus withdraw the
IUPC and check fetal heart tracings.
o If reassuring  attempt to place the IUPC again
o If non-reassuring  consider placental abruption/uterine damage and go to C-section
 Some classic fetal heart tracing changes:
o Variable decelerations – umbilical cord compression
o Late decelerations – placental insufficiency
o Fetal head compression – early decelerations
o Sustained fetal bradycardia – umbilical cord prolapse or tachysystole
o Sustained fetal tachycardia – Maternal fever/chorioamnionitis
o Loss of fetal variability – opiate drugs taken/administered to the mom
o Sinusoidal wave patterns – fetal acidosis, anemia, or hypoxemia
 When umbilical cord prolapse is diagnosed (typically through palpation of cord in the vaginal vault) the
two things to do are 1manipulate the fetus to minimize cord compression, and 2call for assistance for
immediate C-section
o Replacement of the cord, continuation of labor, and surgical-assisted delivery are not indicated
as fetal status can quickly deteriorate or rupture of the cord can occur.
 Performance of a median episiotomy is the single greatest risk factor for third- or fourth-degree
lacerations and more highly associated with anal incontinence

[Chapter 9: Abnormal Labour and Intrapartum Fetal Surveillance – Topics 22 & 26]
Abnormal Labour (dystocia) is any form of abnormal progression of labour
 Three Major Factors that can lead to dystocia (“The Three Ps”)
o Uterine Contractions (“Power”) – if you’re not contracting properly, then you’re not having
proper labour. Typically monitored with external tocodynamometry OR IUPCs. “Adequate”
contractions measure >200 Montevideo units over a 10-minute period. “Too frequent”
contractions are not optimal as relaxation between contractions allows for bloodflow to baby.
o Fetal Factors (“Passenger”) – if baby is big/presenting poorly then it’ll be harder to squeeze
through the pelvis! Estimated weight of >4000-4500g (9-10lbs) increases risk of getting stuck.
Poor presentation (brow, face, mentum anterior face, occipitoposterior, or compound
presentations) can make the fetal diameter larger than if in other presentations. Fetal
anomalies (hydrocephaly, soft tissue tumors, etc.) may play a role as well!
o Maternal Factors (“Parent”) – if mom’s pelvis is too small or if she has some soft tissue growth
that could impede the birth canal, you could have trouble passing baby. However, clinical
pelvemitry, CT, or radiographic analysis of the pelvis is a poor predictor of maternal vaginal
delivery success, barring very small diameters.
 Risks of abnormal Labour – several risks are increased with dystocia, mainly infection (esp. with
ruptured membranes) causing sickness to mom and baby.
 Diagnosis/Management – basically there’s two major types, Protraction (slow labour) or arrest (halted
labour). These are usually called based on mom’s measurements compared to the Friedmann Curve.
o First Stage Disorders include Prolonged Latent Phase (lasts >20hrs in nulliparous, or >14hrs in
multiparous) or Prolonged Active Phase (dilation <1cm/hr in nulliparous, or <1.2cm/hr in
multiparous). Management options are:
 Observation – just watch mom and see if she begins progressing more normally
 Augmentation – consider this if mom’s contractions are inadequate (<3
contractions/10min, or contractions <25mmHg above baseline. May be achieved by
amniotomy (artificial ROM) +/- oxytocin (stimulates uterine contractions). If mom
doesn’t enter the active phase after amiotomy, consider adding oxytocin.
 Continuous Labour Support – basically support from family/friends/midwife.
o Second Stage Disorders – considered with 2nd stage 1lasting >3hr with anesthesia, 2>2hr without
anesthesia, or 3if pushing has lasted >1hr
 Prolonged 2nd stage of labour is NOT an indication for operative delivery as long as baby
and mom are doing well under surveillance. Labour should be continued with efforts to
increase pressure to expel baby and open mom’s pelvic canal
 Operative or cesarean delivery are considered if surveillance shows distress
Operative Delivery
 Types – basically different conditions warrant different naming/set of risks
o Outlet operative vaginal delivery – scalp is visible without separating the labia, fetal skull has
reached the pelvic floor, sagittal suture is anteriorposterior or right/left occiput, fetal head is at
the perineum, and rotation does not exceed 45o
o Low operative vaginal delivery – fetal station at +2 or lower
o Medpelvis operative vaginal delivery – fetal station is above +2
 Indications – no absolute indications but prolonged/arrested 2nd stage of labour, fetal compromise, or
benefit to mom by shortening labour are all indications
 Contraindications – 1if <34wk, no vacuum deliveries (inc. risk of hemorrhage); 2if the fetus as a bone
defect, bleeding diathesis, or is unemerged (inc. risk of hemorrhage)
 Forceps – Foreceps are applied to baby’s head/neck; indicated to aid mom’s efforts if they’re
inadequate. There are risks to mom (trauma, hematoma, pelvic flood injury) and baby (brain/spine
injury, cornea abrasion, and shoulder dystocia if >4000g).
 Vacuum Extraction – application of a soft, mechanical vacuum on baby’s scalp. Typically less trauma to
mom but baby has major risks (intracranial hemorrhage, subgaleal hematoma, scalp lacerations,
hyperbilirubinemia, retinal hemorrhage, or separation of the scalp causing cephalohematoma; 5% risk).
Breech Presentation – there are three types of breech (Frank, complete, or footling), all of which increase
fetal/maternal risks as opposed to cephalic presentation.
 External Cephalic Version (ECV) – manual rotation of baby to achieve a vertex lie. 50% success rate.
Best done around 36wk gestation as spontaneous cephalic version likely would occur before this time.
Note that the fetus must have reassuring heart tones, adequate amniotic fluid, and the presenting part
must NOT be in the pelvis. Tocolytic agents MAY be used to relax the uterus to aid version.
 If a baby cannot be verted, C-section is preferred as most physicians lack experience in breech
deliveries. Make sure to inform mom if risks if attempting a breech delivery and get C-section consent.
Shoulder Dystocia – basically baby’s shoulders get stuck in mom’s pelvis. The turtle sign (emergence and
retraction of the head in a contraction) is a classic sign of shoulder dystocia
 McRoberts Maneuver – hyperflexion of mom’s hips (open up pelvis/sacrum) and applied suprapubic
pressure (angle shoulders to make diameter smaller) is a common first step
 Zavanelli maneuver – flexion of the head with re-insertion of baby into mom’s uterus to re-establish
bloodflow for a C-section. This is a last resort kind of maneuver.
 Episiotomy is typically NOT indicated as it’s the bony tissue that baby is stuck on
 Incidence of brachial plexus injury is 4-40% BUT <10% of these injuries are persistent. Typically mom
can be re-assured that baby will be OK with shoulder massage.
Intrapartum Fetal Surveillance
 Pathophysiology – the uteroplacental unit is the source of oxygen/nutrients/waste removal for the
fetus. If it is interrupted, it leads to hypoxia/shunting of blood to baby’s brain/heart/adrenals. If this
hypoxia goes uncorrected, anaerobic metabolism will occur and lactic acid buildup will begin, causing
progressive damage to the fetal organs.
 Intrapartum Fetal Heart Rate Monitors – This is basically all the things you know about fetal heart
monitoring. If you want a refresher, go to pgs.114-117
 Ancillary Testing – because false-positive rates of fetal distress can be high, other tests can be used to
confirm worries with a non-reassuring FHR tracing.
o Fetal stimulation – scalp sampling, allis clamp scalp stimulation, digital scalp stimulation, or
vibroacoustic stimulation are all methods. Basically you’re looking for accelerations with any of
these applied on the fetal heart monitor.
o Fetal blood pH or lactate – blood sample is taken from the scalp. If pH is decreased, its bad
o Pulse oximetry – not useful, don’t use it
 Diagnosis of Non-Reassuring Fetal Heart Tones – don’t forget your Stage I, Stage II, and Stage III criteria
o Turning mom on her left side, applying oxygen, correction of hypotension, and discontinuation
of oxytocin are all reasonable first steps to correct fetal hypoxia.
o If acidosis or persistent fetal hypoxia are present, then C-section is warranted
Meconium – thick, tarry poo that’s present in the fetal gut in-utero/at birth. If the baby is stressed in-utero
then it may poop out some of this meconium into the amniotic fluid, staining it dark green/black.
 Meconium aspiration syndrome – fetal inhalation of meconium-stained amniotic fluid causing
pneumonitis, pneumothorax, or pulmonary HTN. If this occurs, the infant should be intubated if not
vigorous/thick meconium is present. If the baby is vigorous, then it’s likely OK.

[Chapter 9 Questions]
 A patient failing to dilate further during active labour (like they’ve been stuck at 5cm for 8 hours)
should warrant suspicion arrest of dilation. Interventions are as follows:
o Oxytocin infusion (increase contraction frequency/strength; promote dilation)
o If not adequate: place an IUPC and see if contractions are adequate (>200 MVU/10min)
o If not adequate: Consider C-section
o Note that ambulation DOES NOT aid in progressing delivery
 In the question, a patient is at term (40wk) and in severe back pain, desiring induction of labour. She’s
closed and 20% effaced. The suggested course of action:
o Give Misoprostol (cytotec) for “ripening” of the cervix, then induction with oxytocin (Pitocin)
can be initiated once a more favorable cervix is achieved
o Note that waiting till 42 weeks in this setting severe back pain is not appropriate. The patient is
at term and extremely uncomfortable, thus ripening and induction are reasonable. Plus, going
to the absolute maximum of 42wk is associated with problems (macrosomia, oligohydramnios,
uteroplacenta insufficiency, etc.)
 Prematurity, multiple gestation, genetic disorders, polyhydramnios, hydrocephaly, anencephaly,
placenta previa, uterine anomalies and uterine fibroids are all associated with breech presentation.
 Note that prolonged latent phase is defined as >20 hours for nulliparas and >14 hours for multiparas,
and may be treated with rest or augmentation of labor. Artificial rupture of membranes is not
recommended in the latent phase as it places the patient at increased risk of infection
 Never forget! Gestational diabetes is the biggest risk factor for macrosomia. Maternal obesity,
diabetes mellitus, postterm pregnancy, a prior delivery complicated by a shoulder dystocia, and a
prolonged second stage of labor are all associated.
 Secondary arrest of dilation: lack of cervical change in the active phase for over four hours.
Management includes:
o Amniotomy (artificial ROM) as first step
o If still inadequate: Careful observation and oxytocin admin to increase contractions
o If still inadequate or fetus is non-reassuring: consider C-section
 Don’t forget! Prolonged 2nd stage (active) labour alone is NOT an indication for operative delivery or
C-section. Fetal macrosomia, feto-pelvic disproportion, or materal/fetal distress would all give
indication for them, but not prolongation alone!
 Any Category III fetal heart tracing warrants immediate C-section.
 Frank breech is the most common type of breech presentation, occurring in 48-73% of cases and the
buttocks are the presenting part.
 First Stage Labour (Active) arrest of dilation should be assessed with the following:
o Place IUPC to check if contractions are adequate (>200MVU/10min)
o If not adequate  consider oxytocin administration
o If still not adequate  consider operative-assisted delivery or C-section
 Note that misoprostol (Prostaglandin E1) and cervadil (Prostaglandin E2) are contraindicated in VBAC.
However, operative assisted delivery is OK TO USE in VBAC.
 Don’t forget! Amnioinfusion may be used for repetitive variable decelerations; not for recurrent lates.
 Initial measures to evaluate and treat fetal hypoperfusion (occasional late decelerations):
o Change in maternal position to left lateral position which increases perfusion to the uterus
o Maternal supplemental oxygenation
o Treatment of maternal hypotension
o Discontinue oxytocin
o Consider intrauterine resuscitation with tocolytics and intravenous fluids, fetal acid-base
assessment with fetal scalp capillary blood gas or pH measurement

Case 17 – Hypercalcemia + Tips from UWorld


Calcium physiology: there are really 3 ways that calcium is controlled in the body:
 Bones – release via osteoclasts or sequestration via osteoblasts/mineralization
 Intestines – increased/decreased absorption from the gut
 Kidneys – increased/decreased excretion in to the urine
There are two major hormones that control Ca2+: PTH and calcitonin
 PTH – raises blood calcium levels via activating osteoclasts (bone breakdown) & kidney resorption of
Calcium/excretion of phosphate, stimulates kidney 1a-hydroxylase mediated conversion of 25-OH VitD
 1, 25-OH VitD (stimulates many things, but importantly kidney Ca/phosphate reabsorbtion)
 Calcitonin – decreases blood calcium via deactivating osteoclasts, increasing kidney Calcium excretion,
and limiting activation of Vitamin D
Etiology of Hypercalcemia: extremely variable, but some common threads can be found based on phys.
 Thyrotoxicosis (most common) – hypermetabolic state causing increased bone breakdown
 Primary hyperparathyroidism – results in too much PTH secretion (familial, sporatic, or in MEN 1 or 2)
 Malignancy (2nd most common)
o PTH-rP secretion – effectively too much PTH secretion (lung, head&neck, renal cancers)
o Direct osteolysis – cancer spreads to the bone causing breakdown (breast, multiple myloma,
prostate cancer)
 Hypervitaminosis A – works on RAR or RXR receptors to stimulate osteoclasts/inhibit osteoblasts,
resulting in increased bone turnover (vitamin A over-supplementation or Analogue acne treatments)
 Immobilization – poorly understood, but thought to be a lack of osteoblast activation due to poor
bone stimulation without loss of osteoclast activation. (post-surgery, elderly, paralyzed for any reason)
 Hypervitaminosis D – increased absorption of Ca2+ (over-supplementation)
 Granulomaous Disease (Sarcoid, TB, Hodgkins disease) – epitheliod histiocytes/granulomas can
convert 25-OH VitD  1,25-OH VitD
 Milk-alkali syndrome – calcium over-supplementation (Ca-antacids or osteoporosis supplements)
causing hypercalcemia, metabolic alkalosis, & renal insufficiency
 Mediations
o Thiazides (HCTZ or chlorthalidone) – decreased calcium excretion based on mechanism
o Lithium – poorly understood, but may affect both the kidney and parathyroids
 Rhabdomyolysis – massive myocyte hemolysis causing Ca2+ release into blood
 Adrenal Insufficiency – poorly understood; possibly from hemoconcentration secondary to loss of
aldosterone production and diuresis

Hyperparathyroidism is a common cause, especially on tests; so let’s examine how it happens:


 1o – typically parathyroid adenoma (one dominant thyroid gland with other atrophy)
o Constitutional PTH overproduction; kidney responds to it
o Hyper-calcemia, hypo-phosphatemia, high PTH
 2 – hypocalcemia or low Vit.D; classically from early renal failure; parathyroid glands are normal sized
o

o PTH overproduction via physiologic response to low Ca2+; kidney responds poorly
o Normal/hypo-calcemia & hyper-phosphatemia, high PTH
 3 – long-standing renal failure resulting in constitutional parathyroid hyperplasia even with renal
o

transplant or regular dialysis


o Long-standing PTH production able raise Ca2+, but kidney is still failing
o Hyper-calcemia, hyper-phosphatemia, high PTH

2169/2633: Hypercalcemia
 Presentation: stones (renal calculi), bones (arthritis, osteoporosis), abdominal moans (abdominal pain,
constipation, N/V, pancreatitis, anorexia), psychiatric overtones (confusion, stupor, coma) and others
dehydration (polyuria/poor oral intake) & cardiac problems (shortening QT/arrhythmias)
 Etiology: underlying malignancy, malignancy hyperPTH secretion, thiazide diuretics, others
 Dx: high Ca2+ on BMP
 Tx:
o Mild (<12), Moderate (12-14) or asymptomatic – no immediate treatment. Avoid
diuretics/lithium and give hydration to ensure hemoconcentration does not occur
o Severe (>14) or symptomatic – immediately give IV saline + calcitonin and stop loop diuretics
unless pt needs them for CHF. Long-term give bisphosphonates to decrease the amount of Ca2+
being introduced to the body through bone resorption.

4308/4134/4309/3102: Evaluation of Hypercalcemia


 Step 1: confirm hypercalcemia (repeat Ca2+ level/measure serum albumin to correct for it)
 Step 2: Determine the source (measure PTH)
o High/normal PTH: PTH dependent hypercalcemia
 Primary hyperparathyroidism, familial hypocalciuric hypercalcemia, or lithium
o Low PTH: PTH independent hypercalcemia
 Malignancy (Ca >15) from bone mets or PTHrP production (suppression of PTH)
 Vit D toxicity (too much Ca++ retaining), granulomatous diseases (makes Vit. D),
thiazide toxicity (Ca+ resorbtion), milk-akali syndrome (over ingestion of Ca++),
thyrotoxicosis (increased bone resorbtion), Vit. A toxicity (direct osteoclast simulation
in high doses), prolonged immobilization (increased osteoclastic activity without
matching osteoblastic activity) – often more mild elevations

2599: Treatment of Mild Hypercalcemia (<12mg/dL)


 Classically due to bone lesions from metastatic cancer. It does NOT require emergent fluid therapy but
initiation of bisphosphonates helps slow osteoclastic activity and decrease risk of pathologic
fracture/malignant hypercalcemia

8876: Hypocalcemia
 Presentation: fatigue and weakness
o Labs: low Ca2+
 Dx:
o Low magnesium? Drugs that cause hypoCa? Recent Blood transfusion (citrate sequestration)?
 correct it if yes.
o If no, measure PTH level
 Low/normal PTH – surgical damage to parathyroids, autoimmune destruction of
parathyroids, or some infiltrative disease (Wilson’s, hemochromatosis, etc.)
 High PTH – Vit D deficiency (check 25-Vit D levels), CKD, sepsis, tumor lysis syndrome

4722: Primary hyperparathyroidism is a classic cause of secondary hypertension, typically due to a


parathyroid adenoma (80% of cases). The PTH causes excessive Vit D activation, leading to renal calcium
resorption, GI absorption, and bone breakdown
 “Stones, bones, abdominal groans, psychiatric overtones”
 Hypercalciuria (renal stones), bone pain (resorption of calcium from bone), GI upset/constipation
(intense absorption) and confusion/depression/psychosis (not sure!)
 It’s actually NOT entirely clear how PHPT causes hypertension, but significant HTN may be related to
pheochromocytoma in the context of MEN 2A.

2176/4304: Primary Hyperparathyroidism


 Presentation: typically, asymptomatic, but may present with fatigue, constipation, abdominal pain,
renal stones, bone pain, or neuropsychiatric symptoms
o Labs: hypercalcemia, elevated PTH, elevated urinary calcium excretion
o Note that MEN1a is associated with parathyroid adenoma!
 Etiology: parathyroid adenoma (or other tumor), parathyroid hyperplasia
 Dx: diagnostic labs with clinical picture
 Tx: can be treated medically or surgically via parathyroidectomy if symptomatic (osteoporosis, renal
manifestations), age <50, or extremely high serum (>1g) or urine (>400g) calcium

2165: Familial Hypocalciuric Hypercalcemia


 Benign, autosomal dominant mutation in the calcium-sensing receptor (CaS-R) of the parathyroid
glands. This causes increased elevation of serum Ca2+ to activate them to stop PTH secretion. This
results in increased Ca2+ resorption in the renal tubules
o Differentiated from primary hyperparathyroidism by calcium urine excretion (low in 1ohyper-
parathyoidism, as the kidney is trying to resorb the Ca2+)
 Presentation: asymptomatic with normal renal function
o Labs: hypercalcemia with low urine calcium excretion (<100mg/24hr)
 Dx/Tx: lab abnormalities, no treatment needed in the absence of symptoms

2228/3435/3895/4058: Calcium Renal Stones


 Calcium oxalate – the most common type of renal stone. Radiopaque. Small, envelop-shaped crystals
are seen on microscopy. Classic risk factors are small bowel disease, bowel resection, or chronic
diarrhea, as they decrease the absorption of fat in the gut. This is a problem as the fatty acids will
chelate Ca2+, leaving oxalic acid free of Ca2+, so it can be easily absorbed into the body!
 Calcium Phosphate – another radiopaque stone common in primary hyperparathyroidism or renal
tubular acidosis.
 Some suggestions to minimize formation of these stones are:
o Use of thiazide diuretics (passive increased reabosrbtion of Ca2+)
o Increased fluid intake (dilute it out to make calcium not form stones)
o Decreased Na+ intake (increases Na/Ca reabsorbtion, thus decreasing amount in kidney)
o Normal Ca++ in diet (poor Ca++ intake increases oxalate absorbtion, thus promoting the
formation of calcium oxalate stones!)
o High K+ diet (decreases Ca++ excretion)
o Low animal fat diet (decreased acid production to be filtered out by urine)

2227/11109: Management of Acute Renal Stone


 Imaging: abdominal x-ray will often work, but abdominal CT will pick up even radiolucent stones
 Pain control: NSAIDs preferred in people with normal renal function, but narcotics can be used
 Stone Size: stones <5mm in size should pass spontaneously. Increasing fluid intake to >2L/day will aid
in passing the stone as well as preventing further stone formation
o BMP along with urinalysis should be undertaken in any patient with their first stone
o Tamsulosin (a1-blocker) may be used in relax the bladder/ureteral walls to aid in passing the
stone by preventing reflex ureteral spasm
 Urology: anuria, urosepsis, or acute renal failure all warrant immediate urology consult

4361: Imaging for Renal Stones


 Renal ultrasound “stone study” is best as it is non-invasive, not toxic, cheap, and can visualize all types
and sizes of stones
o Test of choice in pregnant women
 Non-contrast spiral CT can be used as first line, or after an ultrasound. Due to it’s cost and radiation, it
is less preferred, but necessary sometimes as it’s much higher resolution
o never use contrast in a stone study, contrast is opaque and the stone is opaque…thus you’re
making it impossible to see the stone
 Abdominal X-ray is a decent modality in a pinch and will visualize 85%, BUT you can only catch
radiopaque stones, thus it’s not preferred.

3083: Milk-Alkalai syndrome


 Occurs due to over-supplementation with calcium and absorbable alkalai. Essentially you take too
much result in renal vasoconstriction, decrease in GFR, Na/K/2Cl inhibition (diuresis)
 Presentation: N/V, constipation, polyuria/polydipsia, AKI, suppressed PTH in the setting of calcium
supplementation (this patient was taking supplements for osteoporosis)
o Labs: hypercalcemia, hypomagnesiemia, hypophosphatemia, metabolic alkalosis
 Dx: clinical presentation
 Tx: discontinue supplementation with IV fluids/furosemide to flush out the kidneys

4171: Side effects of Thiazide diuretics (HCTZ or chlorthalidone)


 Glucose intolerance – worse insulin release from pancreas AND tissue resistance to glucose uptake.
Worse in pt with diabetes or metabolic syndrome
 Poor lipid metabolism with increased LDL/triglycerides – similar to glucose intolerance. Worse in
diabetes or metabolic syndrome
 Hyponatremia/kalemia/magnesemia and hypercalcemia – based on mechanism of action. Actually
beneficial in staving off osteoporosis
 Hyperureciemia and worsening gout – reduces renal uric acid excretion

3083: DON’T FORGET!!! THIAZIDE DIURETIC HYPECALCEMIA IS MILD AND WON’T CAUSE SYMPTOMS.

3100: Because much of serum calcium is bound to albumin, hypoalbuminemia can result in alteration of
measured serum levels. Thus, something like proteinuria can easily cause hypoalbuminemia, leading to
falsely decreased serum calcium levels.
 Correction to assess ionized (unbound, active) serum calcium is necessary in these situations:
o Corrected Ca = Serum calcium + 0.8(4.0 – serum albumin)

3587/3520/2630/3494: Multiple Endocrine Neoplasia (MEN)


 MEN1:
o Primary Hyperparathyroidism (hypercalcemia, high urine calcium, high PTH)
o Enteropancreatic tumors (most commonly gastrinoma causing Zollinger-Ellison syndrome;
VIPoma, glucagonoma, and insulinoma all possible)
o Pituitary adenomas (most commonly prolactin secreting)
 MEN2a: RET-protooncogene mutation
o Medullary Thyroid Cancer (parafollicular C-cells, produce calcitonin)
o Pheochromocytoma (elevated plasma metanephrines, episodic hypertensive crises),
o Parathyroid Hyperplasia
 MEN2b: RET-protooncogene mutation
o Medullary Thyroid Cancer
o Pheochromocytoma
o Mucosal/cutaneous neuromas, Marfanoid habitus

2615/2632: Major Primary Cancers of the Lung


 Presentation: cough with hemoptysis, chest CT shows characteristic nodule. May have paraneoplastic
syndrome associated with it
Types
 Adenocarcinoma: peripherally located solitary nodule, female, non-smokers,
clubbing/osteoarthropathy. Most common overall regardless of smoking status.
 Squamous Cell Carcinoma: central with necrosis/cavitation, hypercalcemia (PTHrP production:
anorexia, constipation, increased thirst, easy fatigability OR metastases to bone)
o Will cause suppression of PTH with elevated calcium  always considered malignancy
 Small Cell Carcinoma: central hilar mass, smokers, paraneoplastic syndromes associated (Cushing’s
from ACTH; Lambert-Eaton from anti-Ca2+channel; SIADH from ADH), often presenting with metastatic
disease. Systemic chemotherapy mainstay of treatment (“too small to cut out”)
 Large Cell Carcinoma: peripheral, associated with gynecomastia and galactorrhea

4200/3046: Sarcoidosis
 Should you ever see a patient with dry cough, malaise, and bilateral lymphadenopathy it’s sarcoid!
 Presentation: classically it’s a young, often black, woman with cough/malaise/bilateral
lymphadenopathy on CXR. However, lots of lovely signs could be present:
o Anterior/posterior uveitis (vision loss/pain)
o Peripheral lymphadenopathy/hepatosplenomegaly
o Polyarthritis, sometimes with periosteal bone resorption
o Central diabetes insipidus or hypercalcemia (hypervitaminosis D)
o Arrhythmias (Complete AV heart block most common) – granuloma disruption of myocardium
 Cardiac sarcoidosis should be suspected in any patient <55y/o with unexplained 2nd/3rd
degree heart block or with known sarcoidosis
o Lofgren Syndrome: erythema nodosum, hilar lymphadenopathy, migratory polyarthritis, fever
 Dx: characteristic bilateral lymphadenopathy on CXR with biopsy showing non-caseating granulomas
either from the lung or from peripheral tissue should systemic involvement be present
 Tx: corticosteroids

Case 18 – Health Maintenance in the Elderly + Tips from UWorld


Functional Assessment – old people aren’t great at doing stuff. It can cause a huge drop in quality of life if
they cannot perform proper activities of daily living, thus, IADLs and ADLs should be monitored
 Activity of Daily Living (ADL) – bathing, dressing, eating, toileting, grooming, movement/transfers
 Instrumental ADL – driving, shopping, cooking, money management, cleaning, laundry, etc.
 Pts who cannot perform IADLs independently have a greatly increased risk of dementia
Vision Screening – most sensitive screening is a vision test with a Snellen or Jaeger card, although referral for
ophthalmologic examination is a prudent idea if feasible
 Age-related Macular Degeneration – the leading cause of severe vision loss in the elderly.
o Atrophy of cells in the central macula of the retina (loss of central vision)
 Glaucoma – elevated intraocular pressure that causes optic neuropathy, thus vision loss
 Cataract Disease – the most common cause of blindness worldwide
o Opacification of the lens due to changes in lens proteins
 Diabetic Retinopathy – leading cause of blindness in adults in the USA
Hearing Screening – correlated with social isolation/depression, hearing loss is common and best assessed
with a whispered voice test or Hearing Handicap Inventory (HHIE-S), although audiometry is most accurate
 Presbycusis – hearing loss of the elderly, classically with difficulty hearing with background noise, but
none in a quiet or silent room. Selective high-frequency loss and difficulty with voice discrimination.
Not associated with complete hearing loss
 Noise-induced hearing loss – wear and tear hearing loss from noise exposure. Can occur at any age;
difficulty with voice discrimination, hearing over background noise, and tinnitus
 Cerumen impaction – reversible transient conductive hearing loss; lovely to clear out
 Otosclerosis – autosomal dominant disorder of the inner ear bones causing hearing loss onset from
age 20-40yr. Oddly, speech discrimination is well preserved.
 Central auditory processing disorder – CNS dysfunction causing hearing loss
Fall Assessment – the leading cause of non-fatal injury in the elder is a fall; and often falls can cause hip
fracture leading to significant functional impairment a loss of quality of life.
 Always ask about falls, dizziness, and assess gait in old people
Cognitive Screening – dementia rate doubles every 5 years after age 60; thus we should screen for it
 Typically, a clock-draw and a three item recall are easy, reliable tests to screen for dementia
 A positive screening should be followed by Mini-mental status exam for more complete assessment
Incontinence Screening – they happen; ask about the poops
Depression Screening – often treatable and can vastly improve quality of life if treated; may also present as
pseudodementia, blurring the line between cognitive and emotional dysfunction
 Asking Have you felt down/depressed/hopeless in the last 2 wk? and Have you felt little
interest/pleasure in doing things? Is a solid screening (high sensitivity)
 Geriatric Depression Scale should be done on any elderly patient screening positive
Nutrition Screening – serial weight measurement/questioning about about appetite are often good screens
 Daily multivitamin supplementation is recommended for all elderly patients
 Protein malnutrition is common in nursing homes and associated with increased infection risk,
anemia, orthostatic hypotension, and decubitus ulcers
Hypertension Screening – heart disease and CVA are the leading causes of death in the elderly. Thus, BP
screening should always be part of assessment. Thiazides are commonly used in elderly patients
Stroke Prevention – risk doubles with every 10yr increase in age
 HTN – keep within normal range with diet/exercise/medication
 Atrial fibrillation – worry of mural thrombus/embolization; anticoagulation should be undergone
Cancer Screening – often regular screening can be stopped when life expectancy is below 5-10yrs
Osteoporosis Screening – DEXA scan at 60; make sure to supplement Ca2+/VitD
Immunization – follow normal guidelines; specific elderly considerations include: flu shot each year, Tdap at
age 60, Shingles vaccine at age 60, and Pneumovax at age 65.
End of Life Issues
 Advance Directives – prior oral/written statements to guide care if pt is incompetent. These
statements trump any and all other input as they are the pt’s exact wishes
 Durable Power of Attorney – appointment of a surrogate decision-maker to make the decisions that
the patient is assumed to have wanted based on their experiences with the patient
 DNR – only 15% of CPR interventions are successful with significant risk for injury (rib fracture,
laceration, neurologic damage). These decision should be about how a patient will die, rather than if
they’ll live or not as the outcomes are often poor.

2836: Presbycusis
 Sensorineural hearing loss of aging, typically seen in >60yr old patients
 Presentation: high-pitched, symmetric sensorineural hearing loss. Patients typically complain of trouble
hearing in crowds/noisy environments but do well when things are quiet.
 Dx: clinical picture
 Tx: no real treatment. Hearing aid may be useful if loss is severe/impairing
Case 19 – URI and Sinus Infections
4566/4864: Acute Bronchitis
 Presentation: cough from [5 days – 3 weeks] productive for yellow/green/blood-tinged sputum
however, the cough alone may last for months
o Wheezing/rhonchi with sore chest wall easily resolved with coughing (sputum is getting out!)
o No systemic symptoms (lack of fever, chills, etc.)
o Classically after a viral URI (influenza, parainfluenza, adnovirus, rhinovirus, etc.)
o Note that production of yellow-green sputum does NOT indicate bacterial infection here
 Dx: clinical picture; CXR used to rule out pneumonia (fever, consolidation, Hx of lung dx, or elderly)
 Tx: NSAIDs/bronchodilators; antibiotics are not recommended

4201: Rhinitis
 Non-allergic Rhinitis
o Nasal congestion, rhinorrhea, sneezing, post-nasal drip
o Red, boggy nasal mucosa
o No obvious allergic trigger with symptoms year-round
o Typical onset >20y/o
o Tx: intranasal anti-histamine and/or intranasal glucocorticoid
 Allergic Rhinitis
o Watery rhinnorhea, sneezing, with eye symptoms (red, watery, itchy eyes)
o Pale/bluish nasal mucosa
o Obvious allergic trigger or associated allergic disorder with seasonal patterns
o Typical onset <20y/o
o Tx: intranasal glucocorticoid

3285: Rhinosinusitis
 Presentation: purulent nasal discharge, maxillary tooth pain, facial pain, maxillary sinus
pain/tenderness, and non-specific URI symptoms lasting >7 days
o Micro: S.pneumo and H.flu are most common organisms
 Dx: clinical presentation, sometimes with naso-pharyngeal swab evidence of infection
 Tx: antibiotics, short course (<3 days) decongestants, and NSAIDs/Acetaminophen
o 1st line – amoxicillin or TMP-SMX
o 2nd line – amoxicillin-clavulonic acid, 3rd gen cephalosporins, floroquinolones, or macrolides

Pharyngitis
 Presentation: non-specific
o Micro: typically, viral (mycoplasma, chlamydia pneumoniae, Arcanobacterium haemolyticus)
o Examination to rule out more serious causes (Epiglotitis, Peritonsilar abscesss, Group A strep)
 Dx: clinical presentation with ruling out of more serious problems
 Tx: supportive treatment; should resolve with time

2195/2847: Peritonsilar Abscess (‘Quinsy’)


 Presentation: fever, chills, sore throat, muffled “hot potato” voice, uvular deviation, unilateral
lymphadenopathy, pooling of saliva, and trismus (unable to open jaw) from muscle spasm
o Airway obstruction and spread to parapharyngeal space (carotid sheath involvement)
 Dx: clinical scenario with signs on oropharyngeal exam
 Tx: immediate abscess drainage via needle aspiration with culture/empiric antibiotics
o GAS and respiratory organsisms should be covered

2845: Epiglottitis can actually be a severe, life-threatening infection as it can compromise the airway
 Presentation: high fever, severe sore throat, drooling, stridor, “tripod position” to breathe
 Bugs: H.flu or Strep.pyogenes (rare in USA due to vaccinations, but immigrants may have this)
 Dx: clinical scenario with swab and culture
 Tx: immediate airway establishment (intubation or cricothyroidotomy) with emergent treatment to
stop asphyxiation  treatment of the underlying infection

Group A Strep (S.pyogenes)


 Possible GAS manifestations (Rheumatic fever, Post-strep glomerulonephritis, Toxic shock syndrome.
Peritonsillar abscess, Meningitis, Bacteremia)
 Dx: rapid antigen test (faster) or throat culture (gold standard)
 Tx: oral or IM penicillin; if allergy cephalosporins or macrolides can work well

Rheumatic Fever
 Occurs a few weeks after a GAS strep throat
 Antibodies formed from infection cross-react with host antigens (notably in the valves of the heart)
due to molecular mimicry
 Dx: JONES criteria
o J – joint pain from migratory polyarthritis
o ♥ – pancarditis initially with mitral/aortic valve damage over years
o N – nodules in the skin (often firm, but painless)
o E – erythema marginatum (rash with a distinct red border)
o S – Syndenham’s chorea (neurologic problem)
o An ASO titer being high can also be helpful to confirm diagnosis
 Tx: full treatment for GAS to eliminate ANY bacteria possibly there, then prevention via antibiotic
prophylaxis for years

2241: Post-Streptococcal Glomerulonephritis


 Autoimmune disease in response to a GAS throat/skin infection occurring 10-20 days post-infection.
Classically happens in kids and young adults.
 Present: Fever, rash, arthralgia, periorbital edema, hypertension, ‘tea-colored urine’, Hx sore throat
o Urine: oliguria, proteinuria, hematuria, sterile pyuria, eosinophiluria, WBC casts
o Blood: eosinophilia, low serum C3 compliment levels
o Biopsy: sub-epithelial humps due to complement deposition
 Dx: clinical presentation with elevated ASO titer or anti-DNase B antibodies
 Tx: supportive care, should resolve on its own
o Prognosis good in children but very bad in adults often leading to renal failure
3742: Toxic Shock Syndrome
 Bacterial infection (S.aureus most common) with Toxic-Shock Toxin-1 (TSST-1), a super-antigen that
results in massive T-cell activation/cytokine release, resulting in loss of intravascular fluids to the
extravascular spaces and shock!
 Presentation: rapid development of high fever, Hypotension (<90 systolic), diffuse red/edematous rash
(looks like sunburn), skin desquamation of palms/soles, multisystem failure
o Classically (50%) associated with a tampon left in post-menstruation, typically arising 2-3 days
after the last menstrual period
o Non-menstrual infections can also cause it (post-surgery, sinutitis, etc)
 Dx: clinical syndrome with history
 Tx: supportive therapy for shock, removal of foreign materials causing infection, anti-staph (penicillin +
vancomycin) are all part of treatment

3263: Erysipelas is infection of the superficial dermis


 Presentation: fast-onset prominent inflammatory rash with sharply demarcated borders, often
accompanied by fever/chills/etc.
 Bugs: GAS (most commonly S.pyogenes)
 Dx: presentation
 Tx: penicillin (if S.pyogenes) but culture is needed for specific Tx

4354/3131/2869: Infectious Mononucleosis (Epstein-Barr virus)


 Presentation: fever, malaise, posterior cervical lymphadenopathy, tonsillar exudate, and possibly
splenomegaly in an otherwise healthy young adult
o Initial HIV infection can look similar but will be fever, malaise, lymphadenopathy, diarrhea, and
rash  it’s actually reasonable to test for HIV if monospot is negative
o Autoimmune hemolytic anemia/thombocytopenia is a complication of EBV-mono from EBV-
antibodies (IgM cold-agglutinin anti-I antibodies) cross-reacting with RBCs/platelets. Occurs
approximately 2-3 weeks post infection
 Dx: heterophile antibody (monospot) test; Anti-EBV antibody test
o Smear showing variant lymphocytes with convolute nuclei and vacuolated cytoplasm
 Tx: rest and avoidance of contact sports for >3 weeks if splenomegaly present

3007: Infectious mononucleosis (Epstein-Barr virus infection): fever, lymphadenopathy (tonsillitis, swollen
cervical lymph nodes, swollen eyelids), exhaustion, +/- hepato-splenomegaly
 Dx: Positive Monospot (25% false negative on first week), atypical lymphocytosis, transient hepatitis
 Tx: rest and avoidance of strenuous activities for >3weeks after onset (risk of splenic rupture)
o Splenic rupture: abdominal pain and anemia

Otitis Externa
 Presentation: ear pain/itching worse with manipulation, sometimes with inflamed/swollen external
ear canal with drainage and discharge.
o Micro: Staph, Strep, and rarely Pseudomonas (swimmer’s ear; most common in diabetics)
 Dx: clinical presentation
 Tx: irrigation, debridement, and antibiotics (IV for 4-6wk if bones involved)

2828: Malignant (necrotizing) Otitis Externa


 Presentation: intense, unrelenting ear pain that’s worse at night. Purulent drainage/feeling of fullness
that does not respond to topical medication. Ear canal shows granulation tissue/edema. May
progress to osteomyelitis of the skull base with cranial nerve involvement (facial droop) or TMJ
involvement (pain worsened by chewing)
o Associated with poorly controlled diabetes/immunosuppression in elderly patients
o Typically caused by Pseudomonas Aeruginosa
 Dx: clinical presentation with CT/MRI evidence of infection
 Tx: IV ciprofloxacin (first line)
o Fluoroquinolone resistant: other IV anti-pseudomonals (Pip-tazo/cefepime/ceftazidime)
o Failure to respond to Abx: surgical debridement/biopsy to exclude cancer

3260: Malignant Otitis externa


 Presentation: severe ear pain often radiating to the jaw (TMJ involvement, exacerbated by chewing),
ear discharge, granulation tissue in external auditory meatus
o Pain/infection will not improve with topical antibiotics
 Bugs: 95% of these are caused by P.aeroginosa (classic if you’re also diabetic!)
 Dx: clinical presentation with swab/culture
 Tx: anti-pseudomonals will likely work (pip-tazo is my go-to)

2839: Serous Otitis Media


 Presentation: Middle ear effusion resulting in conductive hearing loss without infectious symptoms.
Ear canal shows dull tympanic membrane with hypomobility on pneumatic otoscopy and inflammation
o The most common middle ear pathology in HIV/AIDS due to lymphadenopathy obstructing the
Eustachian tube
o Micro: typically, viral (URI pathogens) but may become bacterial (superimposed)
o Complications (rare) – mastoiditis, meningitis, brain abscess, subdural empyema
 Dx: clinical presentation, sometimes with ear swab and culture
 Tx: often resolves spontaneously, but amoxicillin is first line

Case 20 – Chest Pain


Initial Management of Chest Pain  should be done within 10 minutes of presentation
 There’s a HUGE differential for chest pain, but we need to rule out things that will kill the pt
 Best to get an H&P to see if you can determine etiology
o First priority: 12-lead EKG & CXR  if cardiac etiology then treat immediately!
o Next priority: Place two IVs and draw labs (CBC, CMP, BUN, PT/PTT/INR, CK/CK-MB/Troponins)
 EKG considerations
o Angina: ST-elevation/depression or T-wave inversion
o MI: signs of angina with elevated CK/CK-MB/Troponins
o Don’t forget that not all myocardial infarctions have pathologic signs and clinical picture
should drive interventions. If there’s high suspicion of MI, then interventions may still be sought
Myocardial Infarction
 Presentation: “crushing” sub-sternal, central chest pain sometimes radiating to the jaw, shoulder, arm
or hand (often on left side), nausea/vomiting, persisting >20-30min with decreased exertion
o Angina alone should subside with decreased exertion
o Risk factors for CAD: male, age >40yr, HTN, tobacco use, DM
o Risk factors for MI: all CAD risks, cocaine use, dyslipidemia, LVH, Hx of CAD
 Dx: clinical suspicion with EKG + cardiac enzyme findings
 Tx: MONA = morphine, O2 (2-4 L/min), nitroglycerine (sublingual), Aspirin (325mg chewed)
o Primary Tx: MONA therapy + B-blocker + GP IIb/IIIa blockers
 Cath lab for PCI if within 90min of MI
 ACE inhibitor if within 24hr of MI
 Magnesium infusion if Mg2+ becomes low (decrease risk of Torsades)
 Ca2+ channel blockers have shown NO benefit in clinical trials
o Secondary Tx: modification of risk factors/exercise program + Aspirin/nitrates/B-blocker/Statin

2732: Post-MI remodeling is part of the normal healing process in the heart and will result in a white scar.
However, ventricular dilatation can be significant and actually change the ventricle so much that it can result
in eventual CHF.
 Ventricle will be dilated, globular, and thinned with scarring
 Tx: ACE inhibitor treatment within 24hrs of MI has been shown to limit this dilatation/remodeling and
reduces the incidence of remodeling-induced CHF

[Differential Dx for Chest Pain]


4127: Stable angina pectoris is predictable chest discomfort occurring with exertion and relieves with rest in
<30 minutes due to insufficiency of coronary bloodflow during increased myocardial oxygen demand.
 EKG, CXR, and cardiac enzymes should be used to rule out MI if this presents
 1st line: beta blockers  avoided in low BP/bradycardia, or if side effects aren’t tolerated well; work
well as they decreased blood pressure and contractility of the heart
 2nd line: Ca++ channel blockers (-“dipines”) or long acting nitrates  used after B-blockers fail

2722/2723: Variant (Prinzmetal) Angina: a vasospatic angina that typically occurs in young women without
cardiovascular risk factors, onset in the middle of the night, and often exacerbated by smoking.
 May be precipitated by exercise, hyperventilation, emotion, cold exposure, cocaine abuse; but will not
be consistently exacerbated with exercise like Typical Angina
 EKG can show transient ST elevation with complete return to baseline after ischemic vasospasm
 May be associated with other vasospastic disorders (such as migraines or Raynaud’s phenomenon)
 Tx: Cessation of smoking + nitrates or Ca++ channel blockers (vasodilation)
---------------
3521: Pericarditis: sharp, stabbing chest pain that’s worse with inspiration (more preload = more stretching of
pericardium) and better with leaning forward; classically occurs with a friction rub
 Characteristic EKG: diffuse ST elevation with depression in aVR
 Post-MI Acute pericarditis: occurs a few days after MI
 Dressler’s syndrome: an immunologic pericarditis against heart tissue that occurs a few weeks after a
heart attack (takes time for the titers to rise to do damage)
o Fever/malaise/elevated ESR are all signs to look out for
o Tx: NSAIDs or Corticosteroids for refractory cases

3635: Constrictive Pericarditis: occurs when the pericardium is damaged, loses elasticity, and becomes fibrous
resulting in altered heart function
 Etiology: idiopathic, viral, cardiac surgery, radiation/chemotherapy, or tuberculosis
 Presentation: fatigue/dyspnea on exertion with peripheral edema/ascites, and increased JVP with
sharp X/Y waves (forward flow compromise)
o Pulsus paradoxus/abnormal septal movement: systolic BP drops upon inspiration; heart can’t
distend well and when you breathe in, more blood flows into the right ventricle…so the septum
compensates moving into the left ventricle, diminishing outflow
o Calcifications on X-ray of the pericardium
o Kussmaul’s sign: failure of central venous BP decrease on inspiration
 Tx: Diuretics (temporary relief) with pericardiectomy (definitive Tx)

10764: Constrictive pericarditis: pathologic thickening of the pericardium which results in heart constriction
and ultimately failure of diastolic function
 Dx: signs of right heart failure (fatigue/dyspnea, elevated JVP, cardiac cirrhosis/nutmeg liver);
Kussmaul’s sign (lack of JVP decrease on inspiration), pericardial calcifications, or midsystolic
pericardial knock (like the beat is knocking against something hard)
 Etiology: idiopathic, viral, cardiac surgery, prior chest radiation, TB (endemic)
 Tx: if new and not severe  anti-inflammatory drugs followed by pericardectomy; if severe then
immediate pericardectomy
 Note that normal JVP is <8cm H20)

3979: Uremic pericarditis is pericardial inflammation due to high blood nitrogen (BUN) levels.
 Typical signs of pericarditis (chest pain, friction rub, better with leaning forward)
 Elevated BUN (>60 is classic) with elevated creatinine will clue you into renal failure
 Does not present with typical EKG findings due to a lack of inflammatory penetration with this one
 Tx: hemodialysis (gets rid of the nitrogen waste and will eventually resolve the pericarditis)

2224: Uremic pericarditis: in the setting of renal failure, a BUN >60 mg/dL can result in pericardial sac
inflammation and ultimately a pericarditis (friction rub, chest pain relieved by leaning forward).
 May have abnormal, but not classic EKG due to only low level of inflammation
 Tx: immediate dialysis (typically yields quick recovery from symptoms)
---------------
4720: Acute aortic dissection is a life-threatening, terrifying tearing of the aortic lumen media layer
 Signs/symptoms: tearing chest pain radiating to the back, hypotension, but other signs can result from
involvement of arteries coming off the aorta getting involved
o Pericardium: pericardial effusion  cardiac tamponade/pulsus paradoxus
o Aorta: aortic regurgitation
o Carotid/subclavian As: pronounced hypotension in heat/limb their feeding (blood pressure in
the left arm < right arm)
o Renal artery: stenosis/compression  renal hypoperfusion
 Dx: clinical signs + radiologic evidence of tear (possibly chest CT)

3956: If you see a person with emergent aortic dissection and you need to confirm the diagnosis you:
 Order a CXR  if suggestive of something else, treat that
 If not, get a serum creatinine and check for contrast allergy
o If normal Cr/no allergy  get chest CT with contrast
o If Cr^/allergy  get Transesophageal echocardiography
 MRI may be used if it’s a non-emergent situation

3056: Recognizing aortic dissection can be as easy as seeing a false lumen on a chest CT scan.
 Hypertension is a necessary feature to cause tearing and dissection
o Stanford A: goes back toward the heart  B-blocker with surgical treatment
o Stanford B: goes down the descending aorta  B-blocker and follow-up
 Tx: Labetalol (or possibly another b-blocker)  these will decrease BP and heart rate reducing stress
on the aortic wall quickly
4380: Remember! Aortic dissection can go proximal toward the heart and rip the aorta causing blood to flood
into the pericardium  cardiac tamponade!
 Increased pericardial pressure  diastolic failure  decreased preload and heart failure
 Dx: hypotension, tachycardia, distended JVs, pulsus paradoxus
 Tx: emergency pericariocentesis to drain fluid with surgical repair of dissection
11104: Sudden onset chest discomfort with mediastinal widening and pericardial effusion should prompt you
to think “proximal aortic dissection”. This is a surgical emergency and needs to be rapidly assessed with trans-
esophageal ultrasonography (better picture for better Dx)

4484: Marfan’s syndrome (fibrillin-1 mutation) can have major cardiac implications due to weakened elastic
tissue in the aorta!
 Aortic dissection: painful tearing sensation, radiating toward the back. Uneven BP in the left vs right
arm. Aortic root dilatation causing aortic regurgitation (decrescendo diastolic murmur)
 Mitral valve prolapse may also occur causing backflow into the pulmonary circuit.
---------------
2297: Pneumothorax
 Puncture of the lung resulting in airway pressure normalization with atmospheric pressure within the
thoracic cavity  collapse of the punctured lung
 Presentation: sudden SOB, hyper-resonance to percussion, absent breath sounds, decreased tactile
fremitus, compression of the mediastinum causing ventricular compression (hypotension from
impaired ventricular filling) and IVC compression (central venous hypertension from blood backup)
o Note that the mediastinum shifts toward the normal lung
o This is a classic complication of mechanical ventilation in someone with underlying lung
disease. Increased PEEP can cause hyperinflation and rupture of lung parenchyma, due to
already damaged/fragile lung tissue.
 Dx: presentation & CXR showing lung collapse/tracheal and mediastinal deviation are classic
 Tx: chest tube placement to relieve pressure with surgical repair

4520: Spontaneous Pneumothorax from COPD


 Presentation: sudden onset chest/back pain, SOB, reduced breath sounds on affected side, and hyper-
resonance
o Occurs due to chronic alveolar sac destruction forming blebs (seen as enlarged air sacs on
chest CT) that can rupture into the pleural space, compressing the lungs.
 Dx: clinical presentation with Chest CT evidence
 Tx: no complete concensus. Based on clinical scenario and risk stratification.
---------------
11126: Remember how we made fun of people for misdiagnosing heart attacks for GERD? Well that shit is real
because GERD can sound a whole lot like coronary artery disease! A patient with GERD might come
complaining of chest pain with radiation!! Note the characteristic GERD signs:
 Prolonged pain >1hr, post-prandial symptoms, heartburn/dysphagia, relief with anti-reflux drugs
 Often cardiovascular workup is necessary in these patients, but will remain unremarkable

4346: Note that chronic GERD patients may have chronic cough (stomach HCl into lungs) or hoarseness (HCl
into the larynx) as part of their clinical picture.

4335: Asthma exacerbation due to GERD


 Presentation: recent worsening of asthma symptoms with normal lung exam. Often sore throat,
morning hoarseness, worsening cough when laying down (at night is common), need for inhaler
following meals, dysphagia, chest pain/heartburn, regurgitation sensation
o Result of microaspirations causing irritation and vagal hypersensitivity resulting in proclivity to
bronchospasm
o Often coincides with weight gain (possibly due to a stomach ulcer!)
 Tx: PPI trial

2200: Endoscopy’s role in Evaluation of GERD


 Basically a person with GERD isn’t too much of a worry as long as they’re doing OK. However there are
certain situations where further investigation is required
 Let’s say you have a patient with GERD, if:
o Male >50, symptoms >5yrs, cancer risk (smoking, alcoholism, etc.), alarm symptoms present
(dysphagia, odynophagia, weight loss, anemia, GI bleeding, recurrent vomiting)  endoscopy
o They don’t have any of the above characteristics/symptoms  PPI trial 2 months
 If symptoms are well controlled  stay on that PPI
 If symptoms are refractory  consider switching PPI or increasing dose
 If symptoms STILL refractory  endoscopy
---------------
3178: Perforated Peptic Ulcer Disease
 Presentation: chronic epigastric pain that suddenly worsens, with rebound tenderness/guarding
(peritonitis due to gastric contents in peritoneal cavity), air under the diaphragm on CXR
 Dx: immediate CXR, as diagnosis and prompt treatment (<12hr) is critical to limit mortality
 Tx: immediate surgical repair
---------------
2982/4434/2965/2929: Acute Pancreatitis
 Presentation: severe abdominal pain, nausea/vomiting, elevated serum lipase, sometimes xanthomas
o Alcohol (40%), gallstones (40%), hypertriglyceridemia (classically >1000) are common
o Drugs (azathioprine, valproic acid, thiazide diuretics), infections, or iatrogenic causes (classically
cholesterol emboli from cardiac catheterization) can also be causes
 Dx: two of the following
o Acute epigastric abdominal pain radiating to the back
o Increased amylase and lipase x3 the normal limit  typically the first step in workup
o Diffuse pancreatic enlargement with contrast enhancement (CT) or hyperechoicity (ultrasound)
and sometimes with fat-stranding; Ultrasound is preferred in patients suspected with
gallstones as underlying cause (fat/40/fertile/female = gallstone)
 Tx: supportive care (pain control/fluids) as most cases are self-limited (4-7 days)
o If caused by something that can be treated (drug, infection, etc) then treat that

4603: Severe Acute Pancreatitis


 Pancreatitis + evidence of organ failure due to release of pancreatic enzymes into the vasculature.
This causes increased vascular permeability in/around the pancreas, causing shock!
 Presentation: pancreatitis symptoms with additional ones
o Organ dysfunction due to fluid overload (dyspnea, tachypnea, abdominal ditention, etc.)
o Cullen sign: periumbilical blue-ish coloration indicating hemoperitoneum
o Grey-Turner sign: red-brown coloration around the flanks indicating retroperitoneal bleed
 Risk factors: >75yrs, obesity, alcoholism, pulmonary infiltration on X-ray
o Labs at 48 hrs  CRP >150mg/dL, Rising BUN and Creatinine
 Tx: aggressive IV fluid resuscitation to replace lost volume

3833: Drug induced Acute Pancreatitis


 Anti-seizure (valproate), Diuretics (furosemide, HCTZ), Drugs for IBD (sulfasalazine, 5-ASA),
Immunosuppression agents (azathioprine), HIV-medications (didanosine, pentamidine), Antibiotics
(metronidazole, tetracyclines), and possibly many others!

4919: Chronic Pancreatitis


 Presentation: post-prandial epigastric pain that radiates to the back, steatorrhea (loose, greasy, smelly
stools, hard to flush), fatigue/other constitutional symptoms
o Alcohol abuse (most common), cystic fibrosis, or autoimmune/hereditary disorder are all
possible causes
o May result in fat-soluble vitamin (AEDK) deficiencies
o Steatorrhea occurs with 90% loss of pancreatic function
 Dx: altered pancreatic function tests and fecal fat assays (Sudan Stain)
 Tx: pancreatic enzyme replacement
---------------
2213: Diffuse Esophageal Spasm
 Presentation: spontaneous pain/ painful swallowing with hot and cold food. Does not get provoked by
typical angina exacerbants. Relieved by nitrates/Ca++ channel blockers
 Dx: esophageal manometry shows repetitive, non-peristaltic, high-amplitude contractions
 Tx: nitrates/Ca++ channel blockers for symptomatic treatment
---------------
4650: Costochondritis is an important cause of chest pain that’s sharp, non-radiating, lasts for hours, worsens
with inspiration/movements, and importantly REPRODUCABLE WITH PALPATION. It’s not a heart problem,
it’s a joint tenderness problem.
---------------
4334: First thing: if a patient has a history of panic attack and is having chest pain YOU CANNOT GO
THINKING IT’S AN ANXIETY EXACERBATION UNLESS YOU GET A NORMAL ECG TO RULE OUT ACS.
 If a patient presents with chest pain  get a focused H&P, vitals, and venous access
o If unstable  check ABCs, stabilize, and assess for underlying causes
o If stable  ECG + CXR + aspirin if the chance of aortic dissection is low
 If ECG is weird then assess for STEMI/NSTEMI
 If ECG is not weird look at the CXR
 If CXR is diagnostic  treat whatever it is
 If CXR is NOT diagnostic  look for underlying causes
 If you went through all that crap and they don’t have any cardiopulmonary problems or GERD…then
maybe (MAYBE) you can give them some benzos for their panic attack.
---------------
4042: Any person (especially younger) presenting with agitation, dilated pupils, atrophic nasal mucosa, HTN,
and acute myocardial ischemia should be suspected for cocaine abuse!
 a/B adrenergic stimulation = HTN/pupil dilatation/agitation/ischemia
 cocaine also potentiates thrombus formation
 Tx: immediate benzodiazepines/O2 + aspirin, nitrates, calcium channel blockers
 NEVER GIVE A B-BLOCKER: it will cause unopposed a-adrenergic activity and HTN crisis!

3188: If a patient comes in with signs of heart attack, dilated pupils, and blood in the nose consider cocaine
abuse induced MI.
 Tx: (same as normal MI) cardiac catheterization/thrombolysis with aspirin/clopidogrel/nitrates/Ca++
channel blockers or a-blockers/morphine. DO NOT USE A B-blocker as it will cause hypotensive crisis
due to cocaine + B-blockade = unopposed a-activity
---------------
4693: Herpes Zoster Flair (Shingles)
 Flair of varicella zoster virus often associated with aging, stress, and emotions. Typically VZV will lay
dormant in nerve roots (esp the trigeminal nerve!) and reactivate at that nerve’s distribution
 Presentation: may begin with pain/no rash, progressing to pain with vesicular rash across 1-2
dermatomes and NOT crossing the midline, then post-herpetic neuralgia may persist for weeks later
o Rash crossing the midline should prompt investigation for HIV/immunosuppression
o Immunosuppressive therapy (TNF-a inhibitors) may result in a flair
 Dx: clinical
 Tx: acyclovir, famcyclovir, or valacyclovir can all work to resolve flair and diminish post-herpetic
neuralgia

4431: Note that the pain from VZV (shingles) re-activation may precede the rash. Thus any patent with
unilateral pain without other evidence of injury/organ dysfunction should be considered for shingles.
 Classically it arises to bodily stress (infection, chemotherapy, immunosuppression) but can arise
spontaneously in some folks
 Will be followed by dermatomal vesicular rash and sometimes finishes with post-herpetic neuralgia

3422: Don’t forget! Herpes Zoster (shingles) outbreak is treaed with oral vanacyclovir or acyclovir. Both will
work, but valacyclovir has a better side effect profile!

Case 21 – Chronic Kidney Disease


Common Risk Factors: Diabetes (leading cause!), hypertension, and glomerulonephritis
Stages of CKD are determined by estimated GFR
 Normal Kidney function  GFR between 90-120 mL/min
 1  GFR>90 mL/min + signs of kidney disease
 2  GFR 60-89 mL/min
 3  GFR 30-59 mL/min
 4  GFR 15-29 mL/min
 5  GFR <15 mL/min
Presentation – decreased urination, change in appearance of urine, low volume status (poor skin turgor, dry
mucous membranes, orthostatic hypotension, dizziness, tachycardia) or fluid retention (pulmonary & LE
edema, SOB, JVD, rales), occasional emesis
 Labs – elevated creatinine, hyperkalemia, elevated uric acid, metabolic acidosis, anemia, proteinuria or
microalbuminuria, signs of uncontrolled diabetes
 Imaging – asymmetric or bilaterally shrunken kidneys
Dx – based on presentation. Work-up should include CBC, CMP, LFTs, urinalysis, C3/C4 test, hepatitis panel,
HIV test, HbA1c/fasting serum glucose.
 Imaging or renal biopsy may be necessary depending on suspected etiology
Tx – treatment aimed at underlying problem and reversing possible etiologies
 Trial of fluids  help hypovolemia, hypotension, and sometimes infection
 Achieve blood pressure control at 130/80 with ACE/ARB/diuretic  help alleviate HTN disease
 Phosphate restriction/oral phosphate binders  used in stage 3-5 to limit hyperphosphatemia
 Low potassium diet/potassium binders  used if K+ levels become elevated
 EPO administration  aids in reversing/staving off anemia due to kidney disease
 Urinary tract obstruction  seen on imaging, often surgical resolution
 Reno-vascular occlusion  seen on imaging, often surgical resolution
 Referral to a nephrologist should be sought in women with Cr >1.2mg/dL and men with Cr >1.5mg/dL

4349/3978: In chronic kidney disease, anemia can develop as the kidney is responsible for erythropoietin
(EPO) production. The mainstay of anemia with chronic kidney disease is supplemental EPO + supplemental
iron. Iron is supplemented as EPO will surge RBC production, causing a high need for iron for new heme. Plus,
someone with chronic kidney disease may already be anemic from chronic inflammation!
 Iron supplementation alone should always be tried first before EPO
 Side Effects: BP increase (30%, >10mm diastolic; not well understood), Headaches, Flu-like syndrome,
Red cell aplasia (rare) [HIGH YIELD]

Case 22 – Vaginal Disease


[Chapter 28: Vulvovaginitis – Topic 35]
Normal Vaginal Secretions
 Presentation: asymptomatic, with small amount of clear/white/flocculant discharge; pH 3.8-4.2; Whiff
test (-); Microscopic slide: squamous epithelium with numerous lactobacilli
 Dx: clinical
 Tx: none needed

Bacterial Vaginosis
 Polybacterial overgrowth (Gardnerella Vaginalis) in vagina replacing normal lactobacilli
 Presentation: Increased thin/homogenous/white-grey/sticky discharge that worsens after sex; pH >4.5
o Whiff test (+; fishy)
o Microscopic slide: increased WBCs, decreased lactobacilli, clue cells present
 Dx: 3 out of 4 present: Abnormal grey discharge, pH >4.5, (+) whiff test, or clue cells or Gram stain (+)
 Tx: metronidazole (oral or intravaginal) or clindamycin (oral or intravaginal)
o Treatment of sexual partners does NOT help prevent recurrence

Vaginal Candidiasis (“yeast infection”)


 Overgrowth of Candida spp. typically from air-born sources; requires estrogenized tissue
 Presentation: increased thick/white/”cottage-cheese like” discharge with itching and burning that
often “cannot be reached”; normal vaginal pH;
o Whiff test (-)
o Microscopic slide: hyphae/buds present
 Dx: Visualization (+) on slide or culture (+); clinical evaluation is not reliable
o Latex agglutination testing is useful for non-C.albicans Candida as they do not show hyphae
 Tx: intravaginal nystatin or oral fluconazole (150mg)
o Intravaginal botic acid capsules or gentian violet will treat C.Glabrata as it resists azoles
o Sexual partners are NOT treated, as it is not considered an STD

Trichomoniasis
 Growth of a protozoan that only lives within the human urogenital tract
 Presentation: increased yellow-green/grey/frothy/sticky discharge with dysuria, dyspareunia,
itching/burning; pH >4.5; strawberry cervix with punctate hemorrhages
o Whiff test (+/-; fishy)
o Microscopic slide: normal epithelial cells, increased WBCs, motile Trichomonads
 Dx: visualization on microscopic slide
 Tx: metronidazole (oral; don’t forget disulfiram-like rxns) or tinidazole (oral)
o Screening for other STDs should be undertaken with women infected
o Sexual partners also need to be treated

Atrophic Vaginitis may also present with similar complaints (itching, burning, dyspareunia) typically occuring
in older women; usually does NOT involve vaginal discharge; pH often >4.2; improved with vaginal lubricant
 Urinary tract may have similar changes causing increased urinary frequency/increased UTIs
 Tx: vaginal lubricant and topic/oral estrogen replacement therapy
Desquamative inflammatory Vulvo-vaginitis typically occurs in post-menopausal women with purulent
discharge, exfoliation of epithelium, vulvar burning/erythema, few lactobacilli.
 Often caused by overgrowth of Gram (+) bugs affecting both vaginal and vulva
 Tx: clincamycin cream (2%) for 14 days
[Chapter 29: Sexually Transmitted Diseases – Topic 36]
Screening
 When one STD is present, other should be tested for. Because the patient is engaging in risky enough
sexual activity to contract one, they can likely contract more than one.
 Regular Screening based on age should be part of a normal visit
o Sexually active <25yr: regular screening for chlamydia and gonorrhea
o Developmentally Disabled women: one panel of common STDs at least once
o All sexually active women: HIV test at least once
o Women with cervicitis: screen for PID
o Women with PID (salpingitis): screen for chlamydia, gonorrhea, BV, and trichomonas

Prevention
 Education, limiting of sexual partners, and condom use are all cornerstones of prevention
 Immunization of HPV and HepB are important to prevent transmission
 Use of expedited therapy and health department reporting with chlamydia, gonorrhea, and syphilis
o Expedited therapy is the treatment of a patient’s sexual partner when the patient is diagnosed
with certain STDs (trichomonas, chlam, gono, syph, etc.) without testing
o The laws of expedited therapy and reporting differ by state so read up son!

Chlamydia trachmomatis
 Presentation: abnormal vaginal bleeding, cervicitis/salpingitis, mucopurulent discharge, etc
 Dx: NAAT for Chlamydia of endocervical swab
 Tx: azithromycin (1g) or doxycycline (100mg BID Q7) with acute repeat testing 3-4wk after initial
treatment is completed and at 3mo post-treatment

Neisseria gonorrhea
 Presentation: mucopurulent green/yellow discharge from penis/vagina; may be mild and overlooked
o Increased rates of PID compared to other STDs, as well as increased risk for HIV w/ infection
o Micro: gram(-) diplococcus
o Risk Factors: highest rate of infection in teens/young adults
 Dx: swab with culture/gram stain/NAAT
 Tx: ceftriaxone IM (250mg) + oral azithromycin (1g) or doxycycline (100mg BID Q7)
o Testing for chlamydia, HIV, and syphilis are indicated

Pelvic Inflammatory Disease (Salpingitis)


 Presentation: cervical motion/adnexal/uterine tenderness, abdominal pain fever, mucopurulent
discharge, vaginal bleeding, sometime in the setting of previous STD diagnosis
o Micro: Chlamydia or Gonorrhea infection most common
o Labs: leukocytosis, elevated CRP
o Risk Factors: pt with prior PID, adolescence, multiple sexual partners
o Fitz-Hugh-Curtis syndrome (perihepatitis): ascending infection up the right paracolic gutter
resulting in local fibrosis/scarring between the liver and anterior abdominal wall.
o Tubo-ovarian abscess (TOA) – localized form of infection resulting from STD infection often
with high fever, tachycardia, severe pelvic pain, and N/V; rupture may result in septic shock
 Dx: clinical based on symptoms
 Tx: treat for gonorrhea/chlamydia with possible hospitalization if complicated (acute emergency,
pregnancy, severe illness, TOA development

Genital Herpes (Herpes Simplex Virus-1 or 2)


 Presentation: flu-like illness/neurologic symptoms on initial infection followed by eruption of painful
“dew drop on rose petal” vesicular lesion. Often lesions will lyse leaving a yellowish crust. Dysuria and
rarely aseptic meningitis may occur!
o Secondary infection are less severe and are triggered by activation of virus from the Dorsal Root
Ganglion at site of infection; often preceded by tinging/burning with subsequent eruption
o HSV-1 (cold sores) and HSV-2 (genital lesions) are typically ‘above’ and ‘below’ the belt
respectively, but may go ether way
 Dx: viral culture with PCR for viral DNA
 Tx: although it cannot be eradicated from the DRG, oral acyclovir, famciclovir, or valacyclovir for 7-10
days are all viable options
o Typically used as episodic therapy to help shorten flair course
o Can be used as suppressive therapy in pts with frequent recurrences /decrease transmission to
sexual partners and to fetus if pregnant
o If pt has an active flair during labor c-section is indicated to limit vertical transmission
Human Papilloma Virus
 Presentation: condyloma accuminata (low risk; wartly like, mounded up lesions; think that they
‘accumulate’ just like we can accumulate many types of HPV!) or cervical dysplasia (high risk)
o Micro: Low risk (6,11) and High Risk (16, 18, 31, 33, 45) are the ones we really care about
 Dx: physical exam or biopsy showing cellular changes
 Tx: condyloma acuminatata are treated with cautery/removal; cervical dysplasia is treated based on
cervical cancer algoithms
o Gardasil (HPV 6, 11, 16, 18) vaccine is important to give to boys/girls ages 9-26 to help decrease
the chance of the most common types of HPV infection
Syphilis (Treponema Pallidum)
 Honestly I think this topic has been bludgeoned to death by our studies, thus a review of symptoms
should be sufficient.
 Primary – painless, firm, ‘punched out’, excoriated chancre appearing 10-60 days after infection. Local
adenopathy should be present. Resolution occurs spontaneously within 3-6wk
 Secondary – 4-8wks post-infection; rough/red-brown rash starting on palms/soles, lymphadenopathy,
fever, headache, weight loss, fatigue, muscle aches, and patchy hair loss. Condyloma lata (flattened,
more ‘raw’ looking warts).
 Latent – occurs after secondary stage; no symptoms but (+)testing; symptoms may recur or pt may
progress to tertiary stages
 Tertiary – CNS damage (tabes dorsalis), heart damage (vasa vasorum infection/aortic aneurysm),
ophthalmic (Marcus-Gunn pupil), and painful gumma formation can all occur
HIV/AIDS – I’m comfy with this disease; there’s not much written in the text anyway
Other STDs
 Lymphogranuloma venereum (LGV; C.trachomatis L1,L2,L3) – inguinal/femoral lymphadenopathy
(vaginal) or bleeding/purulent anal discharge, constipation, anal spasm (anal)
 Granuloma inguinale (Donovanosis; Calymmatobacterium granulomatis) – ulcerative lesions which
easily bleed; typically acquired in New Guinea, Austrailia, India, or West Africa
 Chancroid (Haemophilus ducreyi) – painful ‘raggy’ ulcers with grey/yellowish base and sharp borders
 Molluscum contagiosum – highly contagious through any skin contact; pox virus causing small,
painless, itchy papules with ‘umbilicated’ tops; often scratched off appearing as ulcers with crust
 Pubic Lice/Scabies – they can get on pubes; ‘nuff said.

[Chapter 28: Vulvovaginitis – Topic 35 (QUESTIONS)]


 Tx for BV is either Metronidazole 500 mg orally BID for seven days, or vaginal Metronidazole 0.75% gel
QHS for five days.
 Phimosis is resorption of the clitoris
 Trich. treatment is metronidazole 2 grams orally in a single dose or metronidazole 500mg orally twice
daily for seven days with empiric treatment of sexual partner
 Treatment for uncomplicated vulvo-vaginal Candidiasis consists of short-course topical Azole
formulations (1-3 days), which results in relief of symptoms and negative cultures in 80%-90% of
patients who complete therapy.
 Vestibulodynia (formally vulvar vestibulitis) syndrome consists of a constellation of symptoms and
findings limited to the vulvar vestibule
o Presentation: abrupt onset severe, “sharp/burning/raw” pain on vestibular touch or attempted
vaginal entry, tenderness to pressure, erythema
 Symptoms can occur with tampon insertion, tight clothing, or sex
 Typically, a primary inciting event cannot be identified
o Dx: clinical presentation
o Tx:
 Medical: Tricyclic antidepressants to block sympathetic afferent pain loops, pelvic floor
rehabilitation, biofeedback, and topical anesthetics.
 Surgical: vestibulectomy is reserved for patients who do not respond to standard
therapies and are unable to tolerate intercourse.
 Treatment of Gonorrhea/Chlamydia mucopurulent cervicitis is ceftriaxone + azithromycin
 If you get an elderly (>55 y/o) woman with a vulvar lesion causing her pain, the next step is to perform
a biopsy to evaluate for vulvar cancer.
 Detrusor instability
o Presentation: urinary frequency/incontinence with negative UTI/STD workup and normal post-
void residual volume
 Inability for the detrusor muscle to relax to allow for bladder filling
 Parasympathetic system stimulation via muscarinic ACh receptors allows for normal
bladder emptying
o Dx: clinical presentation
o Tx: anti-cholinergic drug administration (e.g. oxybutynin); note that TCAs are anticholinergics
but are really dirty drugs and not really the best first line Tx
[Ch. 29 STDs/Ch. 30 Pelvic Support Defects, Incontinence, UTIs – Topic 36, 37 (QUESTIONS)]
 Don’t forget! Herpes (HSV) can begin with a prodrome of viral/flu-like illness followed by onset of
vaginal itching/burning before the onset of vesicular rash. While HIV can also have a flu-like illness
associated with initial infection, there’s rarely vulvar-vaginal itching and burning.
 Hep B transmission is through sexual contact in 38% of infections; if exposed, PEP should be
administered as soon as possible but no later than 7 days if blood exposure and 14 days if sexual
o Source HepB (+) – give immediate HBIG + HepB vaccine series
o Source HepB(-) or unknown – only give HepB vaccine series
o If inoculated is HepB immune – no further treatment needed
 In treating STDs/PID adolescents have no better outcomes from inpatient vs outpatient therapy, but
hospitalization ensures compliance and aggressive treatment may prevent UG tract scarring. The
recommended regimen for inpatient IV treatment for gonorrhea/chlamydia is either [cefotetan or
cefoxitin + doxycycline or clindamycin + gentamicin.]
 The rate of tubal infertility has been reported as 12% after one episode of PID, 25% after 2 episodes
and 50% after three episodes. Recurrent STDs or PID can result in significant long-term sequelae, such
as chronic pelvic pain, hydrosalpinx, tubal scarring and ectopic pregnancy.
 When assessing urinary incontinence, a normal post-void residual (PVR) is 50-60 cc. An elevated PVR,
usually >300 cc, is found in overflow incontinence.
 Apparently, women with a family history of POP have up to a 2.5 fold increase in prolapse and it’s a
huge risk factor for development.
o Although hysterectomy is associated with an increased risk of apical prolapse, studies show
mixed results on in the development of prolapse. The risk of future prolapse may be highest
when hysterectomy is performed in women with existing prolapse, while the risk in women
with normal pelvic support is less clear.
 In comparing surgical procedures for stress incontinence due to urethral hypermobility or intrinsic
sphincter deficiency (ISD):
o Retropubic urethropexy such as tension-free vaginal tape and other sling procedures have the
best five-year success rates for patients with stress incontinence due to hypermobility.
o Urethral bulking procedures are best for ISD with little to no mobility of the urethra. These are
minimally invasive and have 80% success rate.
o Needle suspensions and anterior repairs have lower five-year success rates for GSI.
o Colpocleisis is one option to treat uterine prolapse, and is not indicated for urinary
incontinence.
 A “drain-pipe urethra” is simply another name for intrinsic sphincter deficiency (ISD). The resemblance
is that a drain pipe is very wide at the top, then narrows, just as the widened proximal portion of the
urethra looks with ISD.
 Oxybutynin apparently is the classic anti-cholinergic drug to give for urge urinary incontinence
 Often when a pelvic organ prolapse occurs, the prolapsing organ is affixed to abdominal fascia:
o Central and lateral cystoceles are repaired by fixing defects in the pubocervical fascia or
reattaching it to the sidewall, if separated from the linea alba.
o Defects in the rectovaginal fascia are repaired in rectoceles.
o Enteroceles are repaired by either vaginal or abdominal enterocele repairs.
o Vaginal vault prolapse is treated either by supporting the vaginal cuff to the uterosacral or
sacrospinous ligaments, or by sacrocolpopexy
 Don’t forget! If the patient has pelvic organ prolapse but isn’t symptomatic…intervention is NOT
necessary at this point.
 In a patient with high surgical mortality risk (this lady was 90y/o with heart disease and diabetes)
colpocleisis is a good option; it’s a procedure where the vagina is surgically obliterated and can be
performed quickly without the need for general anesthesia.
o Sacrospinous fixation (cuff to sacrospinous-coccygeus complex) or sacrocolpopexy (cuff to
sacral promontory using interposed mesh) require regional or general anesthesia and is not the
best option for this patient with high surgical morbidity.

Case 23 – Lower GI Bleeding


Basically if a patient comes in with complaint of changes in stools, get a guiac-stool test; if positive or
negative, whatever, it’s likely you’ll have to snag a colonoscopy or some sigmoid/anoscopy to assess for
bleeding depending on the clinical presentation.

3857: Workup of “Bright Red Blood per Rectum”


 Age <40  anoscopy (basically a scope of the anus)
o Typically yields hemorrhoids, which will often resolve with symptomatic treatment or minor
surgery to remove them
o If no source found, do a sigmoid or colonoscopy
 Age 40-49 without red flag signs  sigmoidoscopy/colonoscopy
 Age >50 or red flags colonoscopy
 Red flags include: recent bowel habit changes, abdominal pain, weight loss, anemia, or family history
of colon cancer

4303: Elevated BUN:Creatinine ratio may be a sign of upper GI bleeding (abdominal pain, hematemesis,
melena) as the RBC proteins will be broken down increasing intestinal urea production/absorption. If the
kidneys are hypoperfused due to bleeding, they may reabsorb more urea to increase blood volume. The
increased absorption in both the intestines/kidney raises the BUN increasing the BUN:Creatinine ratio!

Hemorrhoids
 Presentation: hematochezia with defecation sometimes with pain/irritation/palpable lump in anus
from engorged peri-anal veins
o Risk factors: chronic constipation, liver/heart disease, pregnancy, & prolonged sitting
o Internal: above the dentate line and NOT painful
o External: below dentate line and very painful; often palpable
 Dx: presentation
 Tx: high-fiber diet, stool softener, cessation of straining; surgical removal may be necessary

2341: Diverticulosis
 Presentation: painless, gross rectal bleeding (much more than just spots on the toilet paper)
o Large volume may be associated with lightheadedness/hemodynamic instability
o While diverticula are classically in the sigmoid colon, diverticular bleeding is more common in
the right colon, causing dark hematochezia
 Dx: Colonoscopy showing source of bleed
 Tx: Often resolves spontaneously, but may require endoscopic surgical intervention
4086: Diverticulosis
 Outpouchings of the colon due to weakened areas encountering pressure, thus bulging out. They’re
typically asymptomatic and increase incidence with age, but can cause complications.
o Associated with constipation and oddly enough, may worsen existing constipation!
 Complications: diverticular hemorrhage, diverticulitis, perforation, abscess formation
o Adequate fruit/vegetable fiber in the diet and physical activity lower risk of complications
o Meat, aspirin/NSAIDs, obesity, and smoking increase risk of complications
 While these often pose no problem, it’s important to take steps to limit the risk of complication in
those with diverticulitis with lifestyle changes

2204: Acute Diverticulitis


 Presentation: LLQ abdominal pain, fever, nausea, vomiting, ileus/urinary urgency and frequency (due
to peritoneal irritation in sigmoid colon affecting the bladder)
o Labs: leukocytosis,
o May result in obstruction, fistula, perforation, or abscess formation
 Dx: Abdominal CT w/oral & IV contrast (pericolic fat inflammation, diverticula, bowel wall thickening,
soft tissue masses, abscesses)
o Sigmoid/colonoscopy are contraindicated due to concern of perforation
 Tx: bowel rest and Abx

3467: Colovesical Fistula


 Fistula connecting the colon to the bladder
o Caused by: Diverticular disease (most common, outpouching fuse to bladder), Crohn’s Disease
(full thickness lesions cause inflammation/connection to bladder), Malignancy (disordered
growth bridges to the bladder)
 Presentation: Recurrent mixed flora UTIs, fecaluria (poop out the pee hole), pneumaturia (air in the
urine, often toward the end of the stream)
 Dx: abdominal CT with rectal and vesicular contrast + colonoscopy (rule out malignancy)
 Tx: surgical treatment with treatment of underlying cause
---------------

Ulcerative colitis  if relapsing/poor response to treatment; look for CMV (owl eye)
 Mucosal/sub-mucosal inflammation
 Always begins in rectum and moves its way up; never goes past the cecum, but involves a variable
amount of the colon; continuous (absence of “skip lesions”)
 Presentation: LLQ pain (rectum) with bloody diarrhea
 Histology
o Neutrophils with crypt abscesses and crypt rupture
o Architectural disarray indicates longstanding inflammation; restricted to the superficial mucosa
o NO GRANULOMAS (Th2-mediated)
 Endoscopy: ‘loss’ of vasculature due to inflammation
 Gross: friable mucosal ‘pseudopolyps’, loss of haustra causing ‘lead pipe appearance’ on imaging
 Complications
o Toxic megacolon – stricture and buildup of feces; may result in rupture
o Carcinoma – chronic inflammation predisposes to cancer; risk increases with increasing
involvement and duration of disease (>10 years) – colonoscopy screening every 2-3 years is
standard
 Extra-intestinal Manifestations
o Primary sclerosing cholangitis
o p-ANCA positivity – think the neutrophil invasion!
 Smoking protects against ulcerative colitis

Crohn’s Disease  24 yo woman, weight loss 12lbs, appears pale, non-bloody diarrhea up to 6 stools a day,
tenderness in RLQ going on for months (NOT IBS, it’s likely Crohn’s because damage to the terminal ileum is
causing obstruction/destruction, resulting in the pain in a specific region). May have palpable mass in RLQ.
 Full thickness inflammation with ‘knife-like’ lesions
 Can occur anywhere from esophagus to rectum; not continuous (‘skip lesions’)
o Most common place: terminal ileum
o Least common place: rectum
 Presentation: RLQ pain (terminal ileum) with non-bloody diarrhea
 Histology: lymphoid aggregates/non-caseating granulomas (Th-1 mediated)
 Gross
o Cobblestone mucosa – healing of the knife-lesions
o Creeping fat – contraction of mesenteric fat by granulation tissue contraction
o Strictures (‘string-sign’ or ‘air in the ileum’ on imaging) – fibrosis/granulation tissue narrowing
the lumen gauge
 Complications:
o Malabsorbtion with nutritional deficiency – destruction of mucosa
o Calcium oxalate nephrolithiasis – greater oxalic acid permeability; accumulation in kidney
o Fistula formation – perforation and insertion into a nearby bowel segment
o Carcinoma (if colonic disease present)
 Extra-intestinal manifestations
o Ankylosing spondylitis – spine/sacroiliac joints; low back pain (“bamboo spine”, stiffness in
back) often in young men; HLA-B27 association
o Sacroitilitis – inflammation of sacroiliac joints
o Migratory polyarthritis – joint pain, moving between joints, usually larger joints/unilateral
o Erythema nodosum – inflammation of fat cells, often seen as red lumps on the legs
o Uveitis – inflammation of uvea (pigmented area on the eye)
 Smoking increases risk for Crohn’s disease

2581: Note that IBD (Crohn’s or UC) have a bimodal distribution (20s-30s and 60s) and both commonly present
with neutrophilic crypt abscesses, making the two diseases hard to distinguish at times. The classic discerning
factor is depth of inflammation; Crohn’s = transmural and UC = mucosal only

4903/2207: Crohn’s Disease can present with apthous ulcers (small, grey erosions in the oral mucosa) down to
anal fissures that may show granulomas if biopsied (30%). These two questions really highlighted the fact that
Crohn’s disease can affect ANY part of the GI tract, from [mouth  anus]
 Another fantastic sign that an oral ulcer is due to Crohn’s disease would be signs of chronic
inflammation (^CRP, anemia of chronic disease, or reactive thrombocytosis >400,000)

3790: Zinc Deficiency


 Zinc typically found in meat, whole grains, beans, and nuts (Absorbed primarily in the jejunum)
 Risk factors: IBD (Crohn’s or UC) or TPN
 Presentation: alopecia, abnormal taste sensation, bullous/pustulous lesions surrounding mouth/eyes/
extremities, and poor wound healing
o Growth retardation in children

2920: Primary Sclerosing Cholangitis


 Unknown etiology; inflammation/fibrosis/stricture of the intra/extrahepatic bile ducts
o Associated with IBD (Crohn’s or UC) in 90% of cases
 Presentation: typically, asymptomatic with unexplained LFTs/lab values. Sometimes fatigue and
pruritis can occur, but not classically
o Labs: Cholestatic pattern LFTs (AST and ALT elevated but <300; severely elevated Alk.phos and
bilirubin), hypergammaglobulinemia, elevated IgM, pANCA(+)
 Dx:
o Ultrasound  unremarkable and non-diagnostic
o Cholangiogram  multifolcal narrowing/”beading” of intra/extrahepatic bile ducts
o Liver biopsy  duct obliteration with “onion-skinning” of lymphatic vessels. Confirmatory for
diagnosis most times
 Tx: liver transplant within 12 years of diagnosis
---------------
4389: Suspicion of Colorectal cancer: signs/symptoms
 An older man (>60) with abdominal pain, microcytic anemia, (+)fecal occult blood, and hepatomegaly
with a hard liver edge  prompt you to think infiltrative colon cancer
o Any old person with microcytic anemia (esp. men) or blood in stool  colon cancer
o Hepatomegaly with a hard liver edge  liver cancer (most often from metastatic colon cancer)
o Mildly elevated AST/ALT/Alk.phos in this setting  infiltrative/cholestatic disease (which
metastatic liver cancer would be a part of!)
 Dx: because this pt has signs of colon cancer and liver abnormalities without significant elevations in
LFTs, an abdominal CT should be sought as metastatic malignancy is suspected.

2582/4504: Colorectal Cancer (CRC) Screening in High Risk Patients:


 Family Hx of polyps or CRC: colonoscopy every 2-3 years starting 10 years before first familial diagnosis
or age 40 (if the 10 years prior puts you lower than age 40)
 Inflammatory Bowel Disease (Crohn’s or UC): every 1-2 years starting 8 years post-diagnosis
o Can start 12-15yr post-diagnosis if disease restricted to left colon
 FAP: colonoscopy every year starting at age 10
 Lynch syndrome (HNPCC): colonoscopy every 1-2 years starting age 20

3918: Assessment of a colon polyp [HIGH YIELD]


 Three major types of colon polyps:
o Hyperplastic: most common non-neoplastic polyp; no need for further workup
o Hamartomatous: juvenile/Peutz-Jegher polyps…typically benign
o Adenomatous: most common overall with potential malignancy  further workup, but less
than 1% of these will actually be malignant!
 Adenomatous polyps MUST be further assessed as this will determine its risk for malignancy:
o Morphology: sessile [more malignant] or pedunculated [more benign]
o Histology: [more benign] tubular < tubulovillous < villous [more malignant]
o Size: [low risk of invasion] (<1.5cm) < (1.5-2.5cm) < (>2.5cm) [high risk of invasion]
2210: Dr. Jones’s words echoed in Mike’s head, “An old person with iron deficiency anemia, especially a man,
has a GI bleed until proven otherwise.”
 The first step is a fecal occult blood test  if positive, then it’ll guide further investigation, but if
negative it means NOTHING!
 If you have this clinical scenario and you get a negative FOBT, you then get endoscopy/colonoscopy on
the patient for concern of GI malignancy
Note that radioisotope erythrocyte scintigraphy is only useful in active, significant bleeding. If it’s a slow or
non-active bleed, that test won’t tell you shit.

Case 24 – Pneumonia
Pneumonia
o Presentation: productive cough, fever, pleuritic chest pain, dyspnea, tachypnea (children), altered
mental status/confusion (elderly), and some specific signs can give specific types (discussed below)
o Ronchi, rales, egophany (E  A), percussive consolidation may all occur
o Broad Sub-types:
o Community Acquired (CAP) – the classics like S.pneumo (rust-color sputum) or H.flu (COPD)
o Atypical Pneumonia – more common in teens and young adults
o Hospital Acquired (HCAP) – occurs in hospitals, nursing homes or other healthcare facilities;
CAP organisms are still king here, but S.aureus and drug resistant bugs are possible
o Ventilator Acquired (VAP) – occurs on ventilator patients; this is a bad time
o Dx:
o Clinical presentation
o CXR: shows lobar consolidation (classic) or diffuse interstitial markings (atypical)
o Labs: CBC, BMP, and sputum culture
o Tx:
o Often directed at severity of presentation and suspected source (shown below)

4024: Community-Acquired Pneumonia (CAP) can be treated in any setting, however the first step to deciding
treatment options is to assess patient risk to select appropriate treatment setting. This can be achieved via
the CURB-65 algorithm.
 C – Confusion: if confused the patient gets 1 point
 U – Uremia: if BUN >20, the patient gets 1 point
 R – Respirations >30, the patient gets 1 point
 B – Blood pressure <90/60 the patient gets 1 point
 65 – patient >65 yrs old patient gets 1 point
Based on this algorithm, we can approach CAP in the appropriate setting:
 0-1 points – outpatient treatment (macrolide + doxycycline OR macrolide + floroquinolone)
 2-3 points – inpatient treatment (floroquinolone OR Beta-lactam + macrolide)
 4-5 points – ICU treatment (Beta-lactam + floroquinolone or IV macrolide)

4097: The gold standard for diagnosis (and first step) for classic pneumonia is a chest X-ray. Will show an
infiltrate in one/more of the lobes and helps rule out other causes of presentation like lung malignancy.
8818: Normally, lower lobe consolidation (typically from pneumonia) WON’T cause enough obscuration to
result in blunting on the costo-vertebral angle. Pleural effusions (this lady happened to have an effusion likely
from recurrence of invasive breast cancer) WILL do this.

4166/3892: S.pneumoniae is THE most common cause of CAP/HCAP/HAP (not VAP but hey whatever)
 S.pneumoniae (pneumococcus) vaccine
o PCV-13 – give to all >65, followed by PPSV-23 6-12months later or in high risk groups
o PPSV-23 alone – give to pts <65 with chronic medical conditions, then give normal vaccines
once they hit 65
o For smokers/COPD pts: a PPSV23 (S.pneumoniae) vaccine should be given once <65yrs. All
other vaccination guild lines are the same.
o For HIV/AIDS pts: S.pneumoniae: PCV13 (upon dx)  PPSV23 (8wks, every 5 yrs)

2386: The two S.pneumoniae vaccines differ in the way that they induce immune response, giving them
characteristics that drive their immunization protocols. Pneumococcus comes in a HUGE amount of strains,
and infection with one only gives immunity to that strain. Thus high-risk individuals are vaccinated!
 Pneumococcal Polysaccharide vaccine–23 (PPSV23) – contains 23 strains of historically responsible
strains for pneumonia. Only contains bacterial polysaccharides which cannot be presented to T-cells
by B-cells, thus the response is less robust in young/old and has lesser effect, but better coverage
o Given to all adults <age 65 (as the old age would make this rather useless from weak
immunogenic response)
o Given to all immunocompromised pts >65 8 weeks after PSV13 and repeated every 5 years (it
won’t give a great response, BUT these people are so at risk that it may provide benefit)
 Pneumococcal Conjugated vaccine–13 (PSV13) – contains 13 historically responsible strains that are
able to be conjugated to Diptheria toxin protein. This allows B-cell presentation to T-cells giving most
robust responses AND memory of infection, but is limited on coverage.
o Given to all infants and young children (get some coverage that works early on)
o Given if immunocompromised pt hasn’t received it before.

2267: Remember! S.pneumoniae is THE most common cause of pneumonia in HIV/AIDS patients. You might be
temped to think PCP when an AIDS patient has pneumonia, but you better slow your roll:
 S.pneumoniae – unilateral, lobar infiltrate, productive cough, pleural effusions often occur, >200 CD4
o Remember! This guy is encapsulated, thus even people with normal immunity have a harder
time clearing this infection!
 P.jiroveci – bilateral, diffuse infiltrate, dry cough, pleural effusions rarely occur, <200 CD4

2273/2267: PCP pneumonia (P.jiroveci formerly carinii)


 One of the most common infections for an AIDS patient
 Presentation: dry cough, dyspnea, hypoxia, CXR with bilateral diffuse infiltrate, rarely pleural effusion
 Dx: clinical suspicion + bronchoalveolar lavage showing P.jiroveci (little crushed ping-pong balls!)
 Tx: TMP-SMX + corticosteroids (if poor O2 saturation status; PaO2 <70 or A-a >35)  may show initial
worsening of lung function due to death of bacteria and inflammatory reactions in lung

3938: PCP pneumonia is one of the most common infections in AIDS patients
 Sym: dry cough, fever, exertional dyspnea (out of proportion to CXR findings)
 Signs: CXR (bilateral interstitial infiltrates), elevated lactate dehydrogenase
 Dx: bronchioalveolar lavage demonstrating P.jiroveci
 Tx: TMP-SMX (Bactrim) for 21 days + corticosteroids (PaO2 <70)
o IV pentamidine is an alternative for those that cannot take TMP-SMX but it has a lot of side
effects (damage to kidney, liver, heart, etc.)

4115: If you see atypical pneumonia (diffuse interstitial infiltrate, tachypnea, tachycardia, fever, non-
productive cough) in an immunocompromised (immunosuppressive drugs, chemotherapy, or AIDS) then think
PCP pneumonia. If it’s typical pneumonia  S. pneumo. Don’t overthink it hauss.

4071: Ventillatory Associated Pneumonia (VAP) is typically arise 48hr after being intubated/on a ventilator.
 Presentation: fever, purulent secretions, poor ventilation, leukocytosis
 Typical infections:
o Gram+ cocci (MRSA, streptococcus)
o Gram- rods (Pseudomonas, E.coli, Klebsiella)
 Approach: CXR  If abnormal get sputum samples for culture & empiric abx
4418: Influenza pneumonia
 NOT a post-viral pneumonia, but a pneumonia caused by the influenza virus
 Presentation: flu symptoms (abrupt fever, chills, malaise, myalgias, cough, and coryza), sometimes
fever, pulmonary symptoms (wheezes, crackles, coarse breath sounds) often in winter months
o Leukopenia and proteinuria may be a feature
o CXR showing interstitial or alveolar pattern
 Dx: nasal swab showing influenza antigens
 Tx: Neuramidase inhibitors (oseltamivir/zanamivir) within 48 hours (stops release of progeny virus).
Rimantidine and Amantidine can be effective vs Influenza A

4867/4517: S.aureus typically does NOT cause pneumonia, however it often affects certain subgroups:
hospitalized patients, nursing home residents, IV drug users, cystic fibrosis pts and pts with recent influenza
infection. Typically these pts will have the flu (fever, myalgia, N/V, etc.), get treated and recover, only to find
they get pneumonia (this is prominent in older adults). Some other classic pneumonias:
 Pneumocystis jiroveci (PCP) – HIV pts
 Klebsiella pneumoniae – diabetics, alcoholics (aspiration), and nosocomial; classically has the “currant
jelly sputum”, cavitation, and empyema.
 Pseudomonas aeruginosa – cystic fibrosis/bronchiectasis (poor sputum release)
 Mycoplasma pneumoniae – “atypical pneumonia” (productive cough, headache, rash; often better in
morning and worse in evening)
 Anaerobic infection pneumonia – aspiration (often pts have poor dentition from vomiting) and
typically features lung abscess!

3054/4233: Legionnaire’s Disease (L.pneumophilia; lives in water inside protozoa)


 Presentation: pneumonia, watery diarrhea, high fever with bradycardia, headache/confusion
o Labs: hyponatremia is often pronounced, hematuria/proteinuria, hepatitis can occurs
o Classically with exposure to contaminated water (hospital, hotel, nursing home, cruise ship)
 Dx: Gram stain often negative with many neutrophils because bug is poorly staining  urine antigen
testing for serologic diagnosis
 Tx: macrolides (like azithromycin) or floroquinolones (“-floxacin”)

2292: Aspiration pneumonia (anaerobic organisms)


 Should be suspected in pts with poor dentition (more bacteria in mouth), upper airway
instrumentation, upper GI/nasogastric tube, GERD, dysphagia, alcoholism, or impaired consciousness
 Presentation: fever, chills, cough, leukocytosis, signs of lung infection (consolidation/crackles/etc.), and
foul-smelling sputum (because anaerobes)
 Dx: clinical/radiographic signs with sputum cultures
 Tx: clindamycin (great anaerobe coverage) is first line
o Metronidazole+amoxacillin, amoxacillin-clavuronic acid, or a carbapenem (“-penem”) are all
viable alternative options

4118/4119: Things predisposing to Aspiration Pneumonia


 Drug intoxication/Alcoholism/Dementia – both cause impaired cough reflex/glottis closure in
swallowing due to altered consciousness
 Neurologic deficits causing dysphagia
 Upper GI tract disorders – GERD is classic for getting stomach contents into the lungs
 Nasogastric/endotracheal tubes/large volume tube feedings in recumbent position – typically if
person has these they’re not swallowing well/have some form of mechanical compromise in their
upper airway. Can allow for secretions to easily get into lungs
 Protracted vomiting – if you throw up a lot, then there’s a higher chance it’ll get in your lung

3029: Recurrent Pneumonia is typically indicative of some underlying process, with the first main delineating
factor being what lobe it’s occurring in:
 Same lung region: Local anatomic obstruction (neoplasm, foreign body, stenosis, etc.) or recurrent
aspiration (alcoholism, seizure disorder, GERD or other GI regurgitation)
o First step = chest CT to examine the lung for abnormalities
o Second step = bronchoscopy for further evaluation of negative CT or for biopsy of positive CT
 Different lung regions: sino-pulmonary disease (Cystic fibrosis, immotile cilia), non-infectious, or
immunodeficiency

4753/4467: Pneumonia featuring Parapneumonic Effusion


 Basically a transudate or exudate moving form the lung airspace into the lung pleural cavity
 Transudate – the result of increased hydrostatic or decreased oncotic pressure. Requires no further
workup outside of treatment of underlying condition causing the exudate
 Exudate – due to inflammation/irritation of the pleural lining. Defined by the Light Criteria (having only
ONE of these three means its an exudate):
o Fluid Protein/Serum protein ratio >0.5  high protein in fluid
o Fluid LDH/Serum LDH >0.6  high LDH level in fluid
o Pleural LDH >2/3 the upper limit of normal serum LDH  high LDH in fluid
o Note that low glucose (<60mg/dL) is a classic sign of WBCs/bacteria using the glucose for
energy, denoting another sign for exudate. <30mg/dL is highly suggestive of empyema!
 Don’t forget! If the pleural fluid shows exudate, that often means there’s an infection and local
cytokine release has increased capillary permeability to allow for WBC transit!

4631/4568/4073/4053: Para-pneumonic Effusions/Empyema


 Definition: complications of pneumonia with leakage of fluid/bacteria into the pleural space, causing a
mass to form. Defined by the fluid analysis from thoracocentesis.
 Presentation: severe pneumonia with evidence of pleural effusion on imaging
 Dx/Tx:
o Uncomplicated para-pneumonic effusion – transudate fluid with minimal bacteria.
 pH >7.2 (typically around 7.6), normal glucose, LDH ratio >0.6, low protein
 Negative gram stain, negative bacterial culture
 Tx: antibiotics for pneumonia
o Complicated para-pneumonic effusion – exudative fluid with bacterial invasion
 pH<7.2 (fermentation), low glucose, LDH ratio >0.6, high protein
 Negative gram stain, positive bacterial culture
 Tx: antibiotics for pneumonia with chest tube drainage
o Empyema – exudative fluid with frank pus and bacterial colonization
 pH<7.2 (fermentation), low glucose, LDH ratio >0.6, high protein
 Positive gram stain, positive bacterial culture
 Tx: long-term antibiotics with chest tube drainage

Case 25 – Depression and Mood Disorders


Persistent Depressive Disorder (Dysthymia)
 DSM-5 Criteria
o Depressed mood for majority of the time on most days for at least 2 years in adults (1 year in
children/teens)
o Two or more of the following:
 Poor concentration/trouble making decisions; Feelings of hopelessness; Poor appetite or
overeating; Insomnia or hypersomnia; Low energy or fatigue; Low self-esteem
o In the 2-year period of depressed mood:
 Person has not been free of symptoms for >2 months for the 2 year period
 Patient never had a manic episode
 Dx: DSM-5 criteria
 Tx: anti-depressant + therapy is most efficacious

Depression
 Risk factors
o High cortisol associated with HPA-axis disorder associated with depression
o Hypothyroidism is a classic medical reason for signs of MDD
o Adverse life experiences (esp. in childhood); especially loss of parent before age 11
o If you have a first degree relative with MDD
o Pancreatic cancer (like from poor prognosis) increases risk
 Changes in sleep patterns
o Multiple nighttime awakenings, fracturing sleep
o initial/terminal insomnia (trouble falling asleep and waking up extremely early)
o 'Leaden paralysis' upon waking: feels like limbs are extremely heavy
o Earlier entry/greater duration of REM sleep with decreased Stage 3/Stage 4 sleep
o Atypically hypersomnia can occur
 DSM-5 Criteria:
o Must have 5 of the following symptoms for >2 weeks
 Depressed mood most of the time
 Anhedonia
 Change in appetite or weight (increase or decrease)
 Change in sleeping pattern (Insomnia or hypersomnia)
 Feelings of worthlessness or guilt
 Diminished concentration
 Change in psychomotor activity (agitation or retardation)
 Fatigue/loss of energy
 Recurrent thoughts of death or suicide
 Dx: DSM-5 criteria
o All patients with depression should be questioned (rule out bipolar disorder)
o Always ask if pt is suicidal or homicidal
o May appear as Pseudo-dementia in the elderly!
 Tx:
o First line: Therapy + anti-depressant is most efficacious
 Takes 4-6wk for SSRI effects to kick in
 Typically if patient doesn’t improve, increase the SSRI dose
o Electroconvulsive therapy (ECT) can be used if:
 Depression is unresponsive to any pharmacotherapy
 Patient cannot tolerate pharmacotherapy (depression in pregnancy is classic)
 Rapid reduction of depression is needed (pts will not eat/drink/catatonic/immediate
suicide risk)
o If bipolar: SSRI therapy will ignite a manic episode
o If pt is suicidal/homicidal: immediate hospitalization with sitter and initiate an SSRI

Bipolar Disorder
 Risk Factors:
o Family member with bipolar disorder (strongest genetic risk among mental health diseases)
o Dx of cyclothymic disorder (1/3 of patients with this disease will develop bipolar disorder)
o Onset typically before age 30 (most often around age 18)
 Manic Episode
o DSM-5 Includes 3 (with elevated mood) or 4 (with only irritable mood) of these criteria:
 Distractibility 
 Inflated self-esteem/grandiosity
 Goal-directed activity (work, social, sexual) or psychomotor agitation
 Decreased need for sleep
 Flight of ideas/racing thoughts
 More talkative or pressured speech (uninterruptable)
 Excessive engagement in pleasurable activities with high risk of negative outcomes
(sexual indescretion, shopping spree, etc.)
o 50% of manic patients also have psychotic symptoms
o Often the pts require hospitalization to protect themselves
 Hypomanic Episode
o Meets criteria for mania but isn't full-blown (Lasts at least 4 days; No impairment in
social/occupational functioning; No hospitalization required; No psychosis)
 Bipolar I: at least one episode of true mania with bouts of depression/euthymia/hypomania
 Bipolar II: at least one episode of hypomania with bouts of depression/euthymia
 Subtypes:
o Anxious distress - tense, restless, diffiuclty concentration, fear of bad events, “loss of control”
o Mixed features - depressive symptoms persist during manic/hypomanic episodes
o Rapid Cycling - 4 or more mood episodes (depression, hypomania, mania) within 1 year
o Melancholic features - classic criteria of depression predominate in depressive episodes
o Atypical features - hypersomnia, hyperphagia, reactive mood, leaden paralysis, hypersensitivty
to rejection in depressive episodes
o Catatonia - catalepsy (immobility), purposeless motor activity, mutism, bizarre postures,
echolalaia (responsive to ECT)
o Psychotic features - delusions or hallucinations are present
o Peripartum onset - occurs during or up to 4 weeks after pregnancy
o Seasonal pattern - occuring during a certain season
 Tx:
o Lithium - mood stabilizer shown to reduce suicide risk, but has low therapeutic window
o Carbamazepine/Valproic Acid - anticonvulsants that act as mood stabilizers
o ECT an option in patients refractory to Tx, immediate danger, pregnant women
o Important to ask about suicide: 25-50% attempt at suicide with 10-15% success

Bereavement
 Normal sadness/grief after the loss of a loved one
 Patients will often be sad, but consolable and often a bit embarrassed with their emotional outbursts
o Pts want to feel better while their grieving
o Often many symptoms decreased around 6 months
o No signs of psychoses, disorganization, or active suicidality
 Note! While bereavement may seem like MDD, the pt won't meet the criteria. That if patent meets
all the criteria for MDD, even following the loss of a loved one, the diagnosis is MDD

Case 26 – Post-Partum Care


[Chapter 11: Postpartum Care – Topics 13, 14, 29]
Puerperium – the 6-8wk process of the mother’s body returning to it’s normal state following birth
 Involution of the uterus – initial decrease in size is from expulsion of the fetus; subsequently autolysis
of intracellular myometrial protein (decreased cell size, NOT cell number), complete by week 6
 Lochia – basically fluid/clots that are sloughing off the uterus. Often heaviest in the first 2-3 days post
partum. Lochia rubra (blood clots/decidua), Lochia serosa (lighter with less blood), Lochia alba
(whitish, may persist for several weeks; may be confused for pathologic discharge)
 Cervix/Vagina – the cervix reforms by 1 week, with its os retaining a ‘fish-mouth’ shape, rather than a
nulliparous round shape. The vaginal tissues return to normal and sow a hypoestrogenic state,
especially with breastfeeding as ovarian function will be suppressed. Pelvic floor muscles will regain
tone and Kegel exercises are encouraged to strengthen pelvic floor muscles.
 Return of Ovarian function – typically ovulation occurs in 45 days for non-lactating and 189 days in
lactating women due to prolactin suppression of estrogen. Function should resume within a few weeks
following cessation of breastfeeding.
 Abdominal Wall – stretching resolves over weeks and aided by exercise. Striae gravidarum become
more-silvery and lighten and diastasis recti (separation of rectus muscle and fascia) resolves.
 Cardiovascular system – blood loss from birth is balanced with shifting of extracellular fluid into
intravascular space and elevated pulse rate. There may be CV decompensation in mom’s with heart
diseases that should be monitored.
 Hematopoietic system – leukocytosis occurs during labour/puerperium which may actually mask
leukocytosis occurring from post-partum infection.
 Renal System – GFR remains elevated but returns to normal in a few weeks. Some urinary retention
due to urethral edema is to be expected but urinary incontinence >90 days should be evaluated.
Management of the Immediate Post-Partum Period
 Hospital Stay – typically 48hrs (vaginal) to 96hrs (cesarean) after the day of birth. Mom may be able to
be discharged earlier with specific signs of health, but baby may not be should there still be need for
routine neonatal care to be administered. Focus should be on prepping mom for care of baby.
 Maternal-Infant Bonding – mom should be with baby as much as possible; skin-to-skin and
breastfeeding are encouraged. Should monitor for signs for depression/poor bonding
 Postpartum Complications – some moms will have post-partum hemorrhage (1%). Some moms will
have increased bleeding on days 8-14 post-partum corresponding to separation/release of the
placental eschar. This is self-limited, but delayed post-partum hemorrhage may occur, which typically
responds to medical therapy.
 Analgesia – it’s common to prescribe analgesia, especially after C-section; clinical call as to what to use
 Ambulation – encouraged after day 1 to decreased DVT
 Breast Care
o Breast engorgement – occurs typically in women who do NOT breastfeed 1-2 day post-partum.
Gradual, bilateral pain without fever/symptoms. Pts should be encouraged to wear a well fitting
bra/use ice packs or analgesia to relieve pain. They should also avoid nipple stimulation and
expression of milk as it will make the process of physiologic return to normal longer.
o Galactocele – plugged duct causing gradual, unilateral, localized pain/swelling, without fever or
symptoms of sickness. Should resolve with total breast drainage of milk.
o Mastitis – infection causing sudden, unilateral, localized intense pain with fever/symptoms of
sickness. Typically S.aureus or other local bacterium in baby’s mouth. Pt should continue
breastfeeding and receive proper antibiotics
o Breast Abscess – same as mastitis but with fluctant mass. Same management as mastitis except
with additional incision and drainage of abcess.
 Immunizations – mom should receive rubella/Tdap/RhoGAM if indicated. Baby should receive HepB
surface antigen vaccine and proper screening tests.
 Bowel/Bladder Function – constipation for 1-2 days is common (esp. since patients havn’t eaten much
during labour), hemorrhoids may be exacerbated from pregnancy, and urinary output should be
monitored for 24hrs although it is common to have some urinary retention. Normal treatment of these
conditions can occur while mom is in the hospital or in outpatient basis.
 Care of Perineum – typically there’s some pain that’s relieved with oral analgesia/warm sitz bath.
Unresponsive pain may be a hematoma, which should be evaluated. Dehiscence or infection may also
have occurred if a woman received stitches after birth.
 Contraception – 15% of non-nursing women are fertile at 6 weeks post-partum.
o Combined OCPs – don’t use post-partum; don’t want to screw with mom’s estrogen levels
o Progestin only – OK to use 3-6wks, even if breastfeeding
o Sterilization – often done surgically at the time of a C-section OR after vaginal birth.
Questioning and consent should be sought during pregnancy as to mom’s preferences.
 Sexual Activity – started when patient is comfortable. Minimal risk of hemorrhage/infection at 2 weeks
 Patient Education – reinforce value of healthcare for her/infant. Schedule a post-partum visit.
 Weight Loss – typically loss of 2lbs per month will not affect breastfeeding. Often moms who
breastfeed will lose weight at a faster rate than not.
 Lactation/Breastfeeding – breastmilk provides the best nutrition for a neonate and exclusive
breastfeeding is recommended for 6 months, then as long as mutually desired.
o Contraindications – HIV+ mom (vertical transmission), active/untreated TB (close contact isn’t
OK, but mom can pump breast milk), mom receiving radioactive substances (check breastmilk
until cleared), mom use of illegal drugs, infant has galactosemia.
 Typically, <1% of any medication can appear in the breastmilk. Some drugs that
contraindicate breastfeeding = lithium carbonate, tetracyclines, bromocriptine,
methotrexate, or other listed above.
o Prolactin Release – initial drop in estrogen after birth allows for release of prolactin for breast
milk production. Suckling of the infant stimulates oxytocin for breastmilk ejection (smooth
muscle contraction in lactiferous duct) via nipple stimulation. It also stimulates uterine
contraction (helps with physiologic return of uterine size).
o Colostrum – produced 5 days post-partum; high nutrient/IgA content makes it awesome. It’s
steadily replaced by milk after day 5
o Lactational Amenorrhea – note that prolactin will inhibit release of FSH/LH, ultimately
depressing estrogen/menstrual cycling. While it helps with contraception, it is not reliable
enough alone and medical contraception should be used.
o Nipple Care – fissuring/drying of nipple is common. Use moisturizer or a nipple shield.
Anxiety/Depression in the Post-Partum period – not entirely understood, but thought to be influenced by
hormone changes. Women with personal/family history of depression/anxiety or depression during pregnancy
are good predictive things to note.
 Post-partum Blues – 70-80% of women, typically 2-4 days; mild insomnia, tearfulness, irritability, or
sadness. Typically, self-resolving with no Tx. Increase risk for Post-partum Depression.
 Post-partum Depression – 10% of women, typically 2wk-12mo duration; irritability, labile mood,
insomnia, anxiety that often worsen in the evening. Tx: antidepressants/therapy.
 Post-partum Psychosis – 0.1% of women, variable duration; confusion attention deficit, distractibility,
clouded sensorium. Tx: Antipsychotic/antidepressant drugs.

[Chapter 11: Postpartum Care – Topics 13, 14, 29 QUESTIONS]


 Don’t forget! Uterine atony is the most common cause of PPH and occurs in one in every twenty
deliveries.
 Post-partum hemorrhage can result in Sheehan Syndrome (anterior pituitary necrosis), which may lead
to loss of gonadotropin, thyroid-stimulating hormone (TSH) and adrenocorticotropic hormone (ACTH)
production, as they are all produced by the anterior pituitary. Frequently goes unnoticed for many
years after the inciting delivery. Treatment includes estrogen and progesterone replacement and
supplementation with thyroid and adrenal hormones.
 Factors related to increased rates of infection with a vaginal birth include prolonged labor, prolonged
rupture of membranes, multiple vaginal examinations, internal fetal monitoring, removal of the
placenta manually and low socioeconomic status
 The most common cause of postpartum fever is endometritis. Rate of endometritis is most closely
related to the mode of delivery (vaginal = 2%, C-section = 10-15%). Uterine fundal tenderness is a
common additional symptom.
 Postpartum endometritis are usually polymicrobial resulting in a mix of aerobes and anaerobes in the
genital tract; most common causes amonst the polymicrobial isolates are S.aureus and Streptococcus
 Don’t forget! Post-partum Blues (lasts <2wk) and Post-partum depression (2wk – 6mo)
 A sense of incapability of loving her family and ambivalence toward her infant are classic signs of true
post-partum depression. Baby blues are usually depressive symptoms, but not toward the baby.
 The most significant risk factor for developing postpartum depression is prior history of depression.
Others include marital conflict, lack of perceived social support from family and friends, having
contemplated terminating the current pregnancy, stressful life events in the previous twelve months,
and a sick leave in the past 12mo related to hyperemesis, uterine irritability or psychiatric disorder.
 Apparently the safest method to suppress lactation is breast binding, ice packs and analgesics. No
hormonal interventions are recommended in puerperium mothers due to increased risk of adverse
cardiovascular events.
 Breastfeeding is associated with a decreased incidence of ovarian cancer and breast cancer.
Breastfeeding has NOT been shown to decrease the risk for developing coronary artery disease,
cervical dysplasia and cervical cancer or colon cancer in the mother.
 Side lying position is a good one for breastfeeding, it is important for mother and baby to be belly-to-
belly in order for the infant to be in a good position to latch on appropriately, taking a large part of the
areola into its mouth. If poor latching occurs, baby may not feed as well as it could and mom could end
up with baby sucking around the areola, causing drying/cracking of skin.
 Prolactin is responsible for the synthesis of milk; present in large quantities during gestation, its action
is inhibited by estrogen and progesterone during pregnancy (inhibits alpha-lactalbumin production by
the rough endoplasmic reticulum). After delivery, large amounts of prolactin continue to be secreted,
and milk is produced after estrogen and progesterone decrease (alpha-lactalbumin stimulates milk
lactose production). May take about two days to occur (colostrum often occurs in these two days)
 Bugs causing Mastitis are usually sensitive to penicillin or a cephalosporin; community-acquired
organisms, and even staphylococcal infections are typically susceptible.
 Some classic breastfeeding promotive policies: baby on the breast within a half hour of delivery and
rooming-in for the baby to ensure frequent breastfeeding on demand (i.e. unlimited access).
 Candidiasis of the nipple may occur in breastfeeding if baby has Candida infection in the mouth
o Presentation: intense nipple pain, burning/worsening with breastfeeding; sometimes with
shiny, pink nipples and peeling at the periphery
o Dx: inspection & swab/wet mount of both mom’s nipple and baby’s mouth
o Tx:
 Mom – topical fungal ointment & topical triple antibiotic ointment
 Baby – oral nystatin followed by oral fluconazole
 Signs that a baby is getting sufficient milk: 3-4 stools in 24 hours, six wet diapers in 24 hours, weight
gain and sounds of swallowing. Little spit-ups after feeding is NOT a sign of adequate eating
 Breast Engorgement is a common occurrence after birth. To decrease discomfort: frequent nursing,
taking a warm shower or warm compresses to enhance milk flow, massaging the breast and hand
expressing some milk to soften the breast, wearing a good support bra and using an analgesic 20
minutes before breastfeeding.
 Oxytocin is responsible for milk ejection. Its stimulated best by suckling of the infant
 Note that cortisol and insulin act in concert with other hormones to stimulate the growth and
development of the milk-secreting apparatus.

[Chapter 12: Post-Partum Hemorrhage – Topic 27]


Recognition/Early Detection – defined as >500mL blood loss per vaginal delivery, or >1000mL blood loss of
cesarean birth. However, these are average blood losses for these events, and estimation of blood loss is
extremely subjective. This emphasizes the need for recognition of signs/symptoms of blood loss in the pt.
Note that loss can be overt hemorrhage or more insidious bleeding.
 10% loss – no worries, this is typical and tolerated by most women
 20% loss – first signs of depletion: tachycardia, tachypnea, delayed capillary refill, orthostatic blood
pressure changes, and narrowed pulse pressure (elevated diastolic pressures from vasoconstriction).
 30% loss – continued tachycardia/tachypnea, overt hypotension
 40% loss – oliguria, shock, coma, and possibly death
 Typically bleeding should be stopped as best as possible, with a search for etiology
General Management
 Best to identify pts at risk early on and prepare for hemorrhage management before labour. Prolonged,
augmented, and rapid labours, Hx of post-partum hemorrhage, episiotomy, pre-eclampsia,
overdistention of the uterus, operative delivery, and chorioamnionitis all increase risk
 Treatment should be directed at a suspected cause (most commonly uterine atony) and the use of
transfusable blood products should be understood.
o Packed RBCs – 1 unit increases Hct 3% and Hb by 1g/dL
o Platelets – 1 unit increases platelet count by 5000-10000/mm 3
o Fresh Frozen Plasma – 1 unit increases fibrinogen by 10mg/dL
o Cryoprecipitate – 1 unit increases fibrinogen by 10mg/dL
Causes/Management
 Uterine Atony – normally, the uterine muscle contracts after birth, clamping down on the spiral
arteries to stop bleeding. When this doesn’t occur, uterine atony is diagnosed.
o Presentation: uterine bleeding with “boggy” uterus on palpation.
o Dx: clinical
o Tx: listed in order of consideration
 3rd stage prophylaxis – decreases PPH rates up by 70%. Oxytocin infusion before
placental passage, gentle cord traction, uterine massage, & immediate breastfeeding
 Uterotonic Agents – Methylergonovine maleate (powerful smooth muscle activator,
given IM as IV will cause hypertension from arterial contraction, avoided in HTN pts);
15-methyl prostaglandin F2a (IM or intrauterine); Dinoprostone (suppository);
Misoprostol (suppository) can all be used. Note: if uterus is firm, these will do nothing
(as they only help firm the uterus) and other etiologies of bleeding should be sought.
 Surgical Compression – typically used in failure of other interventions. B-lynch (surgical
compression of the uterus), arterial ligation of uterine arteries, and selective arterial
embolization are first considered and often successful. Hysterectomy is a last resort.
 Laceration of Lower Genital tract – lacerations are common, lacerations causing significant bleeding
are not. Typically, this will be bleeding in the setting of a contracted uterus. These often are identified
via visual inspection and treated with simple surgical closure.
 Retained Placenta – typically uterine contractions aid in separating the zona basialis and the zona
spongiosa to allow the placenta to separate for expulsion. If retained, bleeding will continue.
o After placental expulsion, the placenta should be examined for missing cotyledons, as they
may break off and be retained within the uterus
o A sheared surface or abruptly ending blood vessels may indicate an accessory (succenturiate)
placental lobe which is prone to breaking off for retention in the uterus.
o Manual sweep of the uterus should be done to attempt to identify and remove retained tissue,
although ultrasound can be helpful if diagnosis is uncertain.
o Suction curettage may be used if manual sweep is not successful
o Placenta accreta/increta/percreta may be the underlying cause of retained placenta, as the
placenta will rip apart rather than detach from the deeper uterine tissue. These cases often
involve profuse bleeding and typically require hysterectomy to save mom’s life.
 Hematoma – development of a bleed within tissue, typically from normal birth trauma
o Presentation: exquisite pain at the site of bleed without shock; often with a demonstrable mass
o Dx: clinical
o Tx:
 If <5cm diameter/not enlarging – expectant management with close monitoring
 >5cm or enlarging – should be opened and drained with ligature of active bleeding site
with close monitoring of hemodynamic status
 Coagulation Disorders – any coagulopathy can make normal bleeding worse. However, it should be
noted that bleeding can, in turn, generate a coagulopathy due to loss of blood!
 Amniotic Fluid Embolism – entry of amniotic fluid into the bloodstream; rare and sudden
o Presentation: occurs as a sequence of events after fluid entry [respiratory distress  cyanosis
 cardiovascular collapse  hemorrhage  coma/death]
o Dx: based on clinical presentation
o Tx: cardiovascular/hematologic supportive measures
 Uterine Inversion – literal inversion of the fundus into the uterine corpus and sometimes out into the
vagina, impeding contraction of the uterine muscle. Typically occurs with too heavy of traction on the
placenta during 3rd stage of labour.
o Dx: profuse bleeding, sometimes with round tissue mass protrusion from introitus
o Tx: physical placement of fist into the uterus to revert + uterine massage/uterotonic agent
administration; should reversion be difficult or impossible, utero-relaxant agents + reversion +
uterotonic agents is used; surgical hysterectomy is a last resort.
 Uterine Rupture – tearing apart of the uterine tissue, classically occuring in pts with previous C-section
attempting a vaginal delivery.
o Dx: uterine bleeding, with loss of fetal station, and possible palpation of small parts of fetus
o Tx: depends on severity/hemodynamic status of pt. Less serious situations may be treated with
surgical revision of the opening, but hysterectomy is a distinct possibility.

Chapter 12: Post-Partum Hemorrhage – Topic 27 QUESTIONS]


 Methylergonovine is an ergot alkaloid (potent smooth muscle constrictor). It is also a vasoconstrictive
agent and should be withheld from women with hypertension and/or preeclampsia.
 Prostaglandin F2-alpha (Hemabate) is a potent smooth muscle constrictor, which also has a bronchio-
constrictive effect. It should be used with caution in any patient with a reported history of asthma. It is
absolutely contraindicated in patients with poorly controlled or severe asthma.
 Uterine inversion is an uncommon etiology of postpartum hemorrhage; major causes are:
o Excessive (iatrogenic) traction on the umbilical cord during the third stage of delivery.
o Over-distended uterus (grand multiparity, multiple gestation, polyhydramnios and macrosomia
are all risk factors)
 Presence of a low-lying anterior placenta in a patient with a history of multiple Cesarean births, the
diagnosis of the placenta accreta must be entertained. This is a concern as delivery of the placenta may
result in laceration and severe hemorrhage; typically, only resolvable with hysterectomy.
 The following are associated with retained placenta: prior Cesarean delivery (attachment to scar),
uterine leiomyomas (attachment to smooth muscle), prior uterine curettage (attachment to scar) and
succenturiate (accessory) lobe of placenta (weak connection to main placenta)
 In a patient with post-partum hemorrhage who is NOT responding to uterotonic agents, the least
invasive treatment is placement of the Bakri balloon (device placed into the uterus with a balloon that
is filled with up to 500 cc of sterile fluid) This places pressure on the inside of the uterus. B-lynch
compression stitch, Uterine artery ligation, and hysterectomy all require a laparotomy and should be
reserved for recalcitrant cases. Uterine artery embolization requires placement of embolization
catheters as well as interventional radiology.
 Should the shit really hit the fan and pt requires exploratory laparotomy for assessment/resolution of
bleeding: [1st line - B-lynch compression stitch, 2nd line –Uterine artery ligation, 3rd – hysterectomy]

Case 27- Congestive Heart Failure


Epidemiology:
 1-2% of cases are in people ages 50-59; 6-10% are in patients 65+
 Men: 5-year mortality is 60% with mean survival age of 3.2yr
 Women: 5-year mortality is 40% with mean survival age of 5.4yr
Presentation
 Right Heart Failure: fluid retention, peripheral edema, JVD, hepatojugular reflex, hepatic ascites,
splenomegaly, porto-systemic congestion, nausea/vomiting, distention/bloating, constipation,
abdominal pain, decreased appetite
 Left Heart Failure: pulmonary congestion (dyspnea on exertion, orthopnea, wheezing tachypnea,
cough), bilateral rales, S3 gallop, Cheyne-Stokes respiration, pleural effusion, pulmonary edema
 Common to Both: tachycardia, cardiomegaly, cyanosis, oliguria, nocturia, peripheral edema
Dx: 2 major criteria –OR– 1 major + 1 minor criteria
 Major criteria: paroxysmal nocturnal dyspnea, JVD, rales, CVP >15cm H20, circulation time >25sec,
hepatojugular reflex, weight loss of 4.5kg within 5 days of treatment
 Minor Criteria: bilateral ankle edema, nocturnal cough, dyspnea on exertion, hepatomegaly, pleural
effusions, decreased vital capacity by 1/3, tachycardia
Workup and Labs: don’t forget, CHF is a sign of some other pathologic process going on!
 CBC  elevated WBC (infection or anemia (lack of O2 capacity) which both increase stain on heart
 Serum electrolytes  dilutional hyponatremia and other electrolyte abnormalities are common
 Renal function tests  fluid retention causing CHF signs could actually be from renal failure
 Liver function tests  vascular congestion can cause damage to the liver
 Cardiac enzymes  acute MI may precipitate onset of CHF
 BNP  sign of fluid overload; >500 pg/mL consistent with CHF
 EKG  may show signs of ventricular hypertrophy or other less specific signs
 CXR  cephalization of pulmonary vasculature; perihilar infiltrates (“butterfly pattern”), pleural
effusions (more commonly on right hemithorax), and cardiomegaly may be identified
 Echocardiography  gold standard for diagnosis due to ability to identify enlargement and failure of
the heart functionally
Acute CHF Management
 Oxygen (decrease heart work): 100% non-rebreather  CPAP/BiPAP  intubation/ventilation
o Continuously monitor O2 status; aiming for >90% saturation on room air for discharge
 Furosemide (pee out retained fluid): use if pulmonary edema found on exam
 IV Nitroglycerine (reduce heart O2 demand & decrease BP): pools blood in the veins, should be used
with caution as it may cause precipitous hypotension
 IV morphine sulfate (reduce heart O2 demand & decrease BP): reduces anxiety, thus heart rate, and
acts as a venodilator aiding drop in BP
 Discharge is considered with resolution of symptoms with treatment, meeting O2 demands on room
air, and exclusion of acute coronary syndrome
Chronic CHF Management
 Emphasize diet, weight loss, sodium/fluid restriction
 ACE/ARB: reduce pre-load/afterload/cardiac output through reducing RAAS influence; downside is
they cannot be used in pregnancy, hypotension, hyperkalemia, bilateral renal artery stenosis, and must
be monitored in pts with renal dysfunction
 B-blockers: low dose with titration up reduce progression into chronic heart failure; although in the
acute setting they may worsen CHF
 Diuretics (loop or thiazide): used to decrease fluid retention; regular weighing to measure “water
weight” allows for adjustment to a proper dose
 Spironolactone: an aldosterone antagonist found to reduce mortality in CHF. Monitor for hyperkalemia
 Amlodipine: used in diastolic heart failure as it slows the heart, allowing for more filling time; note that
this is the ONLY Ca2+ channel blocker you can use. Others are shown to increase mortality.
[Tips from UWorld]

Case 28 – Family Planning & Contraceptives


[Chapter 26: Contraception – Topic 33]
Don’t ever forget!
 Progesterone halts ovulation by inhibiting LH
 Estrogen inhibits follicle maturation by inhibiting FSH
Hormonal Contraception
 OCPs (estrogen + progesterone) – suppresses ovulation (HPA axis), regulates cycle, thickens cervical
mucus, discourages implantation (unfavorable uterus)
o Contraindications: smoker >35yr, thromboembolism/vascular occlusive disease/CVA, impaired
liver function, or congenital hyperlipidemia
o Relative contraindications: migraines, severe HTN, DMII, gallbladder Dx, morbid obesity
o Function may be reduced by antibiotics, barbiturate, and antifungal medications
 “Mini-pill” (progesterone only) – thickens cervical mucus; can be used immediately post-partum and
during breastfeeding
 Depo-provera (medroxyprogesterone) – injection every 3 months; often pts cease to have a period;
worst side effects are weight gain, facial hair growth, and irregular bleeding (only in first few months of
use); fertility takes time to return after stopping (depends on how long you were on it)
 Transdermal Patch (Ortho Evra) – same effect as OCPs, but change each week (3 on 1 off)
 Intravaginal Ring (NuvaRing) – same effect as OCPs, but leave in for 3 weeks (3 on 1 off)
 Etonogestrel implant (Implanon) – progresterone only implant effective for 3 years. Similar effects to
Depo-provera, but often less intense due to lower necessary dose
Barrier Methods: seeks to block the passage of sperm and egg meeting
 Condom – only latex condoms are effective in STD prevention. Wrap your dingus boys.
 Female condom – why?
 Sponge – why? Can cause toxic shock syndrome…again I ask why?
 Diaphragm – needs spermicide and be left in place 6hr after sex…who would invent this?
 Cervical cap – diaphragm that can be inserted for 48hr…so you can be uncomfortable for 48hr
 Spermicides alone – dumb. Only use spermicide with a condom if you want to go this route.
Intrauterine Devices (IUDs)
 ParaGard (Copper IUD) – kills sperm on contact for 10yr; can cause increased bleeding; oddly, it’s the
best form of emergency contraception if used within timeframe
 Mirena (progesterone) – local release of progesterone for 5yr. Gives you the benefits of progesterone
without the side-effects. Actually one of the best and most popular contraceptive devices.
Natural Family Planning
 Good for someone trying to sneak a baby into their relationship.

[Chapter 27: Sterilization – Topic 32, 33, 41]


Sterilization is the most common form of contraception in the USA (esp. in people >30yrs or couples who have
been married >10yrs); it’s important to remind patients that it is permanent and although technically
reversible, typically there’s low success.
 Men: the vasectomy is the contraceptive procedure of choice with the benefits of faster sterilization
and easier reversal (although relatively poor outcomes)
o Post-Op complications (5-10%) – bleeding, hematoma, acute/chronic pain, infection
o Some concern about post-op depression, formation of anti-sperm antibodies, and increasing
rates of prostate cancer, however these are not substantial risks
o Pregnancy occurs 1% post-op often resulting from intercourse too soon after procedure
o Multiple ejaculations are needed to clear the collecting ducts of sperm entirely; and couples
should use another method of contraception until semen analysis confirms azoospermia
(typically occurs at 10 weeks post-op)
 Women: several methods, but all involve alteration of the fallopian tubes. Reversal is technically
possible but has low success rate
o Laproscopic: a good option as it’s minimally invasive and often outpatient surgery. All options
of technique serve to occlude the Fallopian Tubes
 Electrocautery – may be either unipolar (one electrified end) or bipolar (two electrified
ends clamping); unipolar has higher risk of injury but has higher success rate
 Hulka clip – minimal tissue damage; easiest to reverse; highest failure rate (1%)
 Falope ring – medium reversibility and failure; higher rates of post-op pain and bleeds
 Filshie clip – lower failure than Hulka clip with similar reversibility
o Mini-laparotomy – most common approach in the world; small infraumbilical incision is made,
the tube is sutured into a ‘knuckle’ and a 3-4mm portion is removed. These open portions are
then sutured shut and may be cauterized to ensure closure
o Hysteroscopy – approaches sterilization through the cervix; with placement of a small metal
device into the fallopian tube causing irritation and growth of tissue for occlusion. Pts take
effective birth control until occlusion is proven by hysterosalpingography
 For some unknown reason, women ho undergo tubal ligation have lower rates of ovarian cancer

Complications
 Risks of surgery (infection, bleeding, pain, etc.) are always present; but there is also increased risk of
ectopic pregnancy with these form of procedures
 Tubal ligation via electrocautery has the highest rate (even more so than mechanical occlusion)

Reversal of Tubal Ligation


 It can happen but really doesn’t have great results
 If a patient has a tubal reversal, they are presumed to have an ectopic pregnancy until intrauterine
pregnancy is firmly established (typically by ultrasound)
 Apparently 26% of pts regret having tubal ligation, although only 1% ask for reversal.

[Chapter 26: Contraception & Chapter 27: Sterilization – Topic 32, 33 QUESTIONS]
 While many women may initially have irregular bleeding when starting Depo-Provera, this often
resolves within 2-3 months of use; 50% of women will actually stop bleeding altogether.
 Apparently when a patient is requesting emergency contraception, you should also immediately
initiate OCPs as well
o Remember that emergency contraception is not considered an abortifacient and has not been
shown to be teratogenic if pregnancy is present
o Plan B or other emergency contraceptives should be initiated within 72hrs of sex (best
outcomes) but can be administered no later than 120hrs after sex
 Contraindications to estrogen (OCPs) include a history of thromboembolic disease, women who are
lactating, women over age 35 who smoke, or have severe nausea with combined OCPs
o These women are better suited for progestin-only or mechanical methods
 Oral contraceptives will decrease a woman’s risk of developing ovarian and endometrial cancer due to
low-dose estrogen exposure.
o Slightly higher risk of developing cervical intraepithelial neoplasia
o Risk of PID, endometriosis, benign breast changes and ectopic pregnancy are reduced

 In pts wih high BMI or previous gynecologic surgeries desiring permanent sterilization, hysterscopic
sterilization (Essure) is a great option due to increased surgical risk
o Pt must take effective contraception for 3mo following surgery, until a hysterosalpingogram can
confirm that the tubes are occluded
 Regret rate after tubal ligation increases with decreasing age (40% of women under 25yr had regret);
women who are not married at the time of their tubal ligation, when tubal ligation was performed less
than a year after delivery, and with conflict between the woman and her partner have higher rates
 Vasectomy and tubal ligation are both 99.8% effective; in a married patient with multiple medical
problems that present high risks for surgery, the husband getting a vasectomy may be the best option
 Don’t forget! A person with Wilson’s Disease (hepatolenticular degeneration) is not a candidate for a
copper-IUD (as they can too much copper from it as a present-source!)
 The patch (Ortho Evra®), while comparable efficiency to the pill in comparative clinical trials, has
significantly higher failure rate when used in women who weigh more than 198 pounds, likely due to
significantly increased subcutaneous fat, inhibiting hormonal diffusion.

Case 29 – Adolescent Health


Guidelines for Adolescent Preventative Services (GAPS) – these are guidelines specifically tailored to
preventing major problems within the adolescent population. Below are things that should be screened in
teens:
 Medical: Hypertension, Obesity/eating disorders, hyperlipidemia (if at risk), TB (if at risk)
 Social/Emotional: Abuse (physical, emotional, sexual), learning disabilities/school problems, substance
abuse, depression, emotional problems, suicide risk assessment
 Sexual: risky sexual behavior/contraception, STIs (esp. Chlamydia/Gonorrhea/HIV), cervical cancer
 General Advisory Topics: healthy eating/maintaining a healthy weight, regular exercise, use of
seatbelts/helmets, optimal sleep/establishing a sleep pattern, avoiding tobacco/drugs/alcohol, limiting
risky online behavior (posting nude pictures, giving out personal info), dealing with bullying
Immunizations often given to Adolescents/Teens
 Hep A, Hep B, MMR, Varicella – Often these are given in childhood; if they have NOT been given
previously, they should be given during the adolescent years
 Tdap booster (every 5 years)
 HPV – given between ages 9-26 in BOTH boys and girls
 Meningococcal vaccine – esp. if going to college or enlisting in the military
 Annual Flu shot – an oldie but goodie
 Pneumococcal vaccine – only if risk risk for infection (lung or immune system problems)
Sports Physicals
 Teen present to the doctor for a sports physical ALL THE TIME, thus it’s important to know what you’re
supposed to be doing in a sports physical. Below are some general guidelines:
o Ensure compliance with vaccine recommendations/GAPs guidelines
o Check for joint health/orthopedic abnormalities
o Ensure that any chronic disease is well controlled for sporting
o Check the heart for HOCM and heart murmurs
 HOCM and heart murmurs
o One of the most common causes of sudden cardiac death in an athlete
o All pts/parents should be asked: exertional chest pain, syncope, Hx heart murmurs, hx of
HOCM, cardiac abnormalities, or premature cardiac deaths
o Signs of Marfan’s syndrome should be assessed: arachnodactyly, arm span greater than height,
pectus excavatum, tall-thin body habitus, high arched palate, eye lens subluxations
 Some tips from UWorld on HOCM
2678: Hypertrophic obstructive cardiomyopathy (HOCM) is an autosomal dominant mutation typically of
myosin binding protein C or cardiac beta-myosin heavy chain gene resulting in abnormal myocardium
 The murmur is decreased with increased blood in the left atrium
o Sustained handgrip (increased afterload), squatting from standing (increased afterload/preload)
or passive leg raise (increased preload)
o These all increase distention of the ventricle, minimizing outflow obstruction
 The murmur is increased with decreased blood in left artrium
o Valsalva, abrupt standing (decreased preload), and nitroglycerin
o All increased obstruction, thus the intensity of the mumur
 Murmur is best heard on the left, lower sternal border/apex

4673: Hypertrophic Obstructive Cardiomyopathy (HOCM) should be suspected in a young person


experiencing syncope, chest pain, and has a creshendo-decreshendo murmur on the left sternal border. While
aortic stenosis CAN cause this murmur, it is unlikely if the patient does not have a familial syndrome/risk
factors that leads to aortic stenosis. Aortic stenosis would be more common in older people.

2711: A CLASSIC reason for a young athlete to get syncope/abnormal heart sounds is HOCM. Oddly, this can
become more severe as the outflow of blood along the narrowed ventricle can pull the mitral valve leaflets
into the ventricle creating outflow obstruction. This outflow obstruction is largely blamed for clinical
manifestations AND the harsh systolic murmur that worsens with standing up suddenly.

Case 30 – Hypertension
Hypertension is bad. It increases the risk for heart attacks, stroke, renal failure, and many other conditions
with risk positively correlated to the elevation of blood pressure. 30% of folks don’t even know they have it,
earning it the name the “silent killer”. Levels of hypertension are as follows:
 Normal BP – <120/80
 Pre-hypertension – 120-139/80-89
 Hypertension – 140-159/90-99
 Severe Hypertension – >160/100
Diagnosis
 Two resting blood pressures, taken with a proper cuff size at least 4hr apart
Workup – once someone is diagnosed, the underlying cause and damage done by HTN should be assessed
 Hx – full Hx making sure to ask about HTN risk factors in Family Hx
 Physical – Vital signs + BP in both arms, BMI, funduscopic exam (retinopathy), oropharynx (OSA),
thyroid exam (hyperthyroid), full body auscultation (bruits/coarctation), AAA, cardiopulmonary exam
 Labs – serum K, Ca, Creatinine, blood glucose, hematocrit, urinanalysis, and EKG
Treatment
 Non-Pharmacologic – always your first option and should be part of all treatment plans. Increase
physical activity, lose weight, reduce EtOH/smoking, and DASH diet (limit sodium!)
 Pharmacologic – often employed when pt on non-pharmacologic Tx isn’t meeting goals
o 1st line – Thiazide Diuretic (well tolerated/strong effect)
o Additional Therapy – ACE inhibitor, ARB, Other diuretics, Ca2+-channel blocker, B-blocker, and
aldosterone antagonists are all options to be used in different pts
 Use if pt >20/10mmHg above goal or with Severe HTN
Some Classic Underlying Causes of HTN are outlined below & others are covered elsewhere: Aortic
Coarctation, Reno-vascular/renal disease, Cushing’s Disease, Hyperthyroidism (elsewhere),
Hyperparathyroidism (elsewhere), Hyperaldosteronism, Pheochromocytoma, Obstructive Sleep Apnea

8819: Aortic coarctation (late presentation): remember that this is a congenital aortic stricture which can
cause significant cardiac signs/symptoms:
 Associated findings: bicuspid aortic valve, Turner’s disease, ventricular septal defect (VSD)
 Dx: classically presents with asymptomatic HTN and epistaxis in a young person
o May also have claudication, headache, heart failure, aortic dissection
o Brachial-femoral delay, upper extremity hypertension with lower extremity hypotension,
continuous cardiac murmur (from collateral flow)
o CXR can show: LV hypertrophy, notching of 3-8 ribs, “3-sign” of aortic indentation
o Confirm with echocardiography
 Tx: balloon angioplasty +/- stenting to open stricture point
 Diabetes Mellitus (covered elsewhere)

4682: Renal artery stenosis has two classic findings:


 Systolic-diastolic periumbilical bruit: apparently if you hear this, think renal-artery stenosis. Don’t get
it confused with AAA…which will produce a systolic bruit and have a pulsating abdominal mass!
 Refractory HTN: the patient in the vignette has a BP 190/120 on 4 different BP lowering drugs…that
ain’t right son. If you see this, consider secondary causes…of which, renal artery stenosis is the most
common.

3894: Renal Artery Stenosis/Renovascular Hypertension


 Presentation: resistant HTN (3 drug regimen cannot control), malignant HTN (end organ damage), new
severe HTN (>180/120) after age 55, recurrent flash pulmonary edema
o Asymmetric kidney size, abdominal bruits, unexplained rise in creatinine after starting an
ACE/ARB, unexplained atrophic kidney
 Dx: clinical presentation with concurrent findings
 Tx:
o Initial: ACE/ARB administration + additional agents if needed. If RAS is unilateral, the
unaffected kidney is no longer subject to the RAAS activation being set forth by the stenotic
one, and will compensate for proper renal function. If bilateral, kidney function can worsen
o Refractory: revascularization or renal artery stenting. Reserved for those who do not tolerate
anti-hypertensives, have flash pulmonary edema, or heart failure due to HTN.

2592/4721/4419: Cushing’s Syndrome


 Presentation: central obesity, buffalo hump, moon facies, skin atrophy with purple striae, proximal
muscle weakness, hypertension (mineralocorticoids), glucose intolerance, skin hyperpigmentation,
eccymoses, women may have hirsutism/menstrual irregularities
o Labs: hyperglycemia, hypokalemia
 Etiology: exogenous corticosteroid administration (most common), ACTH-producing tumor (Cushing’s
disease), ectopic ACTH-production (small-cell lung cancer), or primary adrenal disease
 Dx:
o Establish hypercortisolism with salivary cortisol assay, 24-hr free urine cortisol, and low-dose
dexamethasone suppression test (2/3 must be positive)
o If hypercorticolism present  ACTH level measurement (see if process is ACTH dependent or
independent)
o If ACTH is elevated  high-dose dexamethasone suppression test to determine if source of
ACTH is pituitary (will be suppressed) or ectopic (won’t be suppressed, likely lung cancer)
 Tx: aimed at underlying cause

3231/2725/3230/3832/8897: Hyperaldosteronism
Primary Hyperaldosteronism (Conn’s syndrome)
 Typically, from an adrenal adenoma or bilateral adrenal hyperplasia
 Presentation: hypertension (increased Na+), hypokalemia, high aldosterone, low renin, adrenal
changes on CT scan, and metabolic alkalosis (hypoK increases bicarb reabsorption/H+ secretion)
 Dx: CT findings + aldosterone:renin ratio >20 or adrenal suppression after oral saline administration
o Adrenal venous sampling can distinguish bilateral from unilateral dysfunction
o Best first test is the aldosterone:renin ratio
 Tx:
o Unilateral: surgical excision
o Bilateral or surgery not an option: aldosterone antagonists (K-sparing diuretics) spironolactone
or eplerenone (note that spirono has anti-androgen effects, but eplerenone will have less of
these. Spirono is still the preferred first-line treatment)
 Note that these patients may be on a non-K-sparing diuretic simply because they seem to have
hypertension. But K-wasting due to diuretic use will not approach the degree that
hyperaldosteronism will achieve. Often the two combined will cause the hypokalemia

Secondary Hyperaldosteronism
 Typically, from reno-vascular HTN, malignant HTN, renin-secreting tumor, or diuretic use (basically the
kidney’s normal response to poor perfusion)
 Presentation: hypertension (increase Na+), hypokalemia, high aldosterone, high renin

Pseudo-hyperaldosteronism
 What looks like aldo, smells like aldo, but isn’t aldo? Basically things that increase other “corticoids”
that aren’t mineralocorticoids…but can act as mineralocorticoids. CAH, deoxycortisone-secreting
tumor, Cushing’s syndrome, or exogenous mineralocorticoid use
 Presentation: hypertension (increase Na+), hypokalemia, low aldosterone, low renin

3976: Pheochromocytoma
 Neuroendocrine tumor of chromaffin cells producing catacholamines (typically 5-HT)
o Association with MEN2a/MEN2b
 Presentation: episodic hypertension, palpitations, sweating, pallor classically with increase
intraabdominal pressure (urination, palpation, positional change) or anesthesia/surgery
o Beta blockers alone will cause hypertensive crisis (5-HT activates a-receptors)
o Must give a-blockers before surgical removal (phenoxybenzamine)
 Dx: clinical presentation demonstration of mass on imaging
 Tx: pre-treatment of phenoxybenzamine and surgical removal

3033/4448: Obstructive Sleep Apnea


 Presentation: daytime sleepiness, morning headaches, poor sleep with multiple awakenings
o Labs: erythrocytosis with increased EPO (reaction to hypoxemia)
o Risk factors: Obesity, tonsillar hypertrophy, excessive oropharyngeal tissue, or short mandible
o Sequlae: cor-pulmonale, right-heart failure, pulmonary/systemic HTN, depression, daytime
accidents due to sleepiness, impotence
 Dx: nocturnal polysomnigraphy  >15 apneic or hypopnea events in a night
 Tx: directed at underlying cause

Case 31 – Infantile Abdominal Pain/Vomiting


Intussusception – “telescoping” of bowel with inflammation and eventual obstruction causing symptoms
 Presentation: infant with intermittent vomiting/abdominal pain that resolves completely and
spontaneously; episodes of pain last approximately 20 minutes.
o Typically with vomit of foodbilious vomiting (distal intestinal obstruction)
o Classically a palpable sausage shaped mass (telescoped bowel) can be palpated on the right
side of the gut (occurs most often at the ileocecal junction)
o May show “currant jelly” or mildly bloody stools from bleeding within the gut
o If not resolved, bowel necrosis will set in causing permanent damage to gut
 Dx: plain film X-ray (rule out perforation) or contrast enema (Dx and therapeutic)
 Tx: enema to relieve intusseceptum +/- surgical resolution

Hypertrophic Pyloric Stenosis – hypertrophy of the pylorus causing obstruction in upper GI tract; most
common cause of infantile GI obstruction
 Presentation: non-bilous projectile vomiting immediately following meals; baby will always seem
hungry despite vomiting (“hungry baby!”)
o Vomiting only food/mother’s milk directly after or during feeding sessions
o Olive shaped mass can be palpated and visible peristalsis can be seen in upper abdomen
 Dx: ultrasound imaging of thicken pylorus or upper GI contrast showing the “double-bubble sign”
 Tx: IV fluid repletion and immediate surgical resolution (near 100% success rate)

Gut Malrotation with Volvulus – gut rotates around itself cutting off blood supply due to improper mesenteric
development causing poorly secured intestines within the abdomen
 Presentation: bilious vomiting with abdominal pain that will start and NOT resolve spontaneously
o Often presentation <1mo of age
o If advanced, hypovolemia, hypotension, GI bleeding, peritonitis, and sepsis/death can occur
 Dx: upper GI series imaging showing volvulus, “misplaced duodenum”, or duodenal obstruction
 Tx: stabilization if necessary with immediate surgical resolution

Foreign Body Bowel Obstruction


 Presentation: abdominal pain/vomiting, sometimes with Hx of ingestion
 Dx: plain radiographs or Hx indicative of forgeign body presence
 Tx: stool checks by parents to ensure passage (90% will pass spontaneously)
o Button-shaped batteries – require immediate surgical intervention as they will be able to
conduct electricity within the body causing perforation
o Sharp objects/multiple magnets – have a good chance to perforate the bowel wall and require
surgery. If passed into dueodenum, these may be watched with daily x-rays until passage. If
object doesn’t move in 3 days, surgery is required.

Poisoning
 Presentation: depends on thing ingested; often OTC drugs are of biggest concern but
insecticides/nicotine/personal care items/fumes/etc are all possible
 Dx: based on history of ingestion or symptoms of ingestion (cholinergic or anti-cholintergic syndrome)
 Tx: immediate poison control consult

Case 32 – Dementia
Evaluation and Dx
 Careful Hx and review of medications for broad search for underlying cause
 CBC, B12, CMP, Thyroid hormone testing, and depression screening
 Clock Draw – quick and easy screen for dementia
 Folstein MMSE – comprehensive mental status assessment often used for further etsting
Alzheimer’s Disease
The most common dementia; present in 4% of pts >65 years old
 Histology: neurofibrally tangles and senile plaques in cortex/hippocampus
 Lack of ACh production by the nucleus basalis of Meynert is thought to drive disease symptoms
 Risk factors:
o Old age, female sex, history of head trauma, Down’s syndrome
o ApoE4 (chrom. 19), Amyloid Precursor Protein (chrom. 21), Presenilin 1 (chrom. 14), Presenilin
2 (chrom. 1), a2-microglobulin mutation (chrom. 12)
 Manifestation:
o Early: language/reasoning/behavior normal with minor “slip ups”
o Late: significant impairment of language/reasoning/cognitive function, loss of independence
o Very Late: impairment of motor function, personality changes, hallucinations, delusions
o “Sundowning” is a phenomenon where symptoms become worse at night.
o Impaired sense of smell is a consistent finding in this and other dementias
 Diagnosis
o Clinical signs/symptoms and progressive worsening
o Elevated tau protein or low AB-42 levels in CSF
o MRI showing shrinkage of amygdala, hippocampus, and thalamus
o PET/SPECT showing bilateral tempo-parietal hypometabolism
o Truly can only be diagnosed with pathologic brain samples showing plaques/NFTs
 Therapy:
o Goal is to delay progression and maximize function; disease course lasts approximately 12 years
o Donepezil: cholinesterase inhibitor | Diarrhea, abdominal cramps, hepatic toxicity
o Rivastigmine: cholinesterase inhibitor | Diarrhea, abdominal cramps, hepatic toxicity
o Memantine: NMDA-receptor antagonist | dizziness, headache, confusion
o Galantamine: cholinesterase inhibitor | Diarrhea, abdominal cramps, weight loss
Disease course lasts 9-12 years before death
Vascular (Multi-infarct) Dementia
Dementia resulting from cerebrovascular disease; classically progressing in a step-wise fashion (person at a
level of decreased function, then sudden steps down to another level); Typically complex
attention and executive function are affected; Pt will often have some major sign of vascular disease (bruits,
etc.)
 Causes
o Macrovascular: disease from infarction of major brain vessels
o Microvascular: disease from subcortical ischemia (lacunar strokes/deep white matter damage)
 Risk factors: HTN, diabetes, advanced age, embolic sources, atherosclerosis
 Diagnosis:
o Dementia + two or more of additional symptoms:
o Focal neurologic signs | abrupt, step-wise, or stroke-related onset | brain imaging showing
multiple strokes, lacunes, or extensive white matter damage
 Treatment: prevent further vascular disease and treat risk factors
Dementia with Lewy Bodies
Core features:
 Fluctuations in cognition with pronounced changes in attention/altertness
 Visual Hallucinations are a hallmark and often quite vivid
 Parkinsonism; often mild and unilateral with rigidity, but not full-blown movement disorder
Suggestive features:
 REM-sleep behavior disorder: often parasomnias or acting out dreams (can be dangerous!)
 Severe neuroleptic sensitivity: akathisia/TD/worse parkinsonism with normal dosing
 Low DA-transporter uptake in basal ganglia (SPECT or PET): often a classic sign
 Low metabolism in occipital lobe (SPECT or PET): another classic sign to be aware of
Dx: 2 core features or 1 core + 1 suggestive; definitive diagnosis on autopsy
 Imaging (CT/MRI) are useful for ruling out other possible causes of disease phenotype
 Neuropsychologic testing helps in differentiating AD from DLB
 EEG may be used, but isn’t the most useful thing
 Autopsy will show a-synuclein intranuclear Lewy Bodies
Tx: no way to slow disease progression but some can restore function
 Anti-cholinesterase drugs (rivastigmine, donepezil, galanthamine)  cognitive/behavior problems
 ECT is safe if depression occurs
 DO NOT GIVE NEUROLEPTIC DRUGS
Often difficult to distinguish from Alzheimer’s Disease; we must be able to differentiate the two

Frontotempotal Lobe Dementia


 Manifestations:
o Impaired executive function (poor social function, decision making, trouble with organization or
sequencing, etc.)
o Hallmark Personality changes, often with disinhibition
o Atrophy of the frontal/temporal lobes
 Associated with abnormal tau protein TDP-43

Parkinson Disease
Defined: progressive disease causing bradykinesia, rigidity, and dementia
 Substantia nigra shows loss of pigmentation and/or eosinophilic Lewy Bodies
 Familial variants show mutation in the parkin gene (autosomal recessive)
Signs/Symptoms: insidious onset; average age of 60 for symptoms
 Cardinal Features: onset is often asymmetric, then generalizes
o Resting (pill-rolling) tremor, Cogwheel rigidity (often seen when checking ‘tone’), Hypokinesia
(best correlate to severity of DA loss), with Postural instability
 Stooped/shuffling gait and difficulty arising from a chair are classic signs
 Loss of smell is nearly universal in PD patients
 Pain, sialorrhea, dysarthria, cognitive deficits (esp. executive function) can occur
Dx: Ruling out other diseases is a huge part of diagnosis:
 Multiple systems atrophy – parkinsonism with less pronounced tremor/symmetric onset (“hot-crossed
bun sign” on MRI
 Dementia with Lewy Bodies – parkinsonism with dementia/hallucinations/fluctuations/myoclonus
 Corticobasal ganglionic degeneration – asymmetric parkinsonism with alien limb/dystonia/apraxia
 Progressive supranuclear palsy – Pseudoparkinsonism with inability to look down, upright posture,
and early/frequent falls (because they cannot check the ground)
 Shy-Drager Syndrome – parkinsonism with extreme dysautonomia (orthostatic hypotension is classic)
 Nigrostriatal degeneration?
Tx:
 Levodopa-carbidopa (carbidopa is a COMT inhibitor to keep L-dopa levels high) – may cause dyskinesia
 DA agonists (pramipexole, ropinirole, bromocriptine) – may decrease incidence of dyskinesia if used 1st
 MAO-B inhibitor (selegiline, rasagiline) – augments L-dopa and primarily helps disease symptoms
 Amantadine (NMDA antagonist; useful for eliminating L-dopa induced dyskinesia)
 Deep Brain Stimulation of the subthalamic nucleus
Note: Anti-psychotics and anti-emetics (prochlorperazine/metoclopramide) can induce Parkinsonism

Huntington Disease
Defined: severe genetic disease causing loss of motor control/psychiatric decline
 Autosomal dominant CAG repeat elongation in the huntingtin gene (4p16.3)
 <25 CAG (no dx); 26-39 (sometimes dx); >40 (always dx); 60+ (earlier onset dx)
 Average age of onset 40yrs
 Anticipation may occur especially if paternally transmitted (family should all be screened)
Signs/Symptoms: slow onset of symptoms is typical
 Chorea: jerky, dance-like movements affecting the entire body; initially trouble with
coordination/movement but ultimately swallowing/choking
 Dementia: personality change, disinhibition, depression, anxiety, suicide
 Westphal variant (childhood): more like Parkinsonism; bradykinesia and rigidity; atrophy of the
caudate nucleus and putamen is characteristic
Dx: clinical suspicion with confirmed genetic testing; rule out drug effects
Tx:
 No treatment to stop/slow disease progression
 Chorea:
o Haloperidol: typical antipsychotic (D2 antagonist); stops aberrant activation of basal ganglia.
May be discontinued if TD occurs or later in disease where bradykinesia is a bigger problem
o Tetrabenazine: unknown mech; helps improve symptoms
 Depression/Anxiety: SSRIs
 Swallowing/Aspiration: PEG tube placement
 Genetic counseling for family

Pseudodementia
Depression that mimics dementia; typically, these pts will have dementia-like features but will often show a
lack of impairment with prodding. Classically, a patient who brings themselves into your clinic complaining of
dementia symptoms is depressed. Dementia pts are unaware of their deficits.
 Pseudodementia
o Acute onset
o 'Sundowning' is uncommon
o Pts often answer "I don't know" to questions (may answer properly when pressed)
o Pt aware of problem
o Antidepressants improve cognitive functioning
 Dementia, by contrast:
o Insideous onset
o 'Sundowning' is common (^confusion at night)
o Confabulation is response to questions they cannot answer
o Pt is unaware of problem
o Antidepressants do NOT improve cognitive function

Vit. B12 (cobalamin) deficiency


Remember that B12 has a complicated pathway of absorption into the body:
 R-binder, stomach acid, intrinsic factor all play a role in transport/absorption in the distal ileum
A classic disorder resulting from several possible etiologies:
 Pernicious Anemia: antibodies against the parietal cells of the stomach (no intrinsic factor produced)
 Vegan diet: B12 is found primarily in animal proteins; diet may restrict intake
 Many others that will depend on the clinical picture in the case
Signs/Symptoms
 Stocking-Glove peripheral neuropathy (often an early sign) and Paresthesias
 Beefy, smooth, sore, red tongue (loss of papillae around edges)
 Megaloblastic (pernicious) anemia and hypersegmented PMNs
 Premature hair whitening or yellow-waxy hair
 Flabby/bulky frame, mild icterus, or blotchy skin pigmentation
 Subacute combined Degeneration: Demyelination of Dorsal Columns (loss of position, fine touch,
vibratory sensations)/corticospinal tract (major motor dysfunction is usually the worse manifestation
of disease
 Progressive dementia and memory loss are not uncommon to see, especially in long-term
Dx: clinical suspicion, taking B12 levels/consequences of B12 levels, MRI studies
 Serum B12 <150 pmol/L
 Total serum homocysteine >13umol/L or methylmalonic acid >0.4umol/L
 Folate deficiency may cause the anemia, but will NOT cause the neurologic manifestations
 MRI shows degeneration in the brain white matter of dorsal columns/lateral pyramidal tracts
Tx: replete Vit B12 (IM injection to bypass gut absorbtion) with subsequent injections for storage

Syphilis (Inc. Tabes Dorsalis and Neurosyphilis)


Signs/Symptoms:
 Possible manifestations/history of syphilis infection (may even be treated previously!)
o Primary = characteristic raised, painless lesion at site of infection
o Secondary = mucutaneous lesions
o Tertiary = endarteritis of small vessels, inflammatory neuron damage, gummas, tabes dorsalis
 Lancinating pain: a stabbing/shooting pain that’s classic of neurosyphilis
 Sensory ataxia: worse in the dark, often with a broad-based gait and normal strength
 Argyl-Robertson pupil: won’t constrict to light, but will accommodate
 Cerebellar dysfunction: Romberg sign can be (+) and can contribute to gait
 Dementia or other psychiatric manifestations
Dx: clinical signs/symptoms and history confirmed with testing
 RPR/VDRL: often positive, have questionable sensitivity for neurosyphilis; negative result prompts LP
 LP: elevated protein, lymphocytes, VDRL(+), elevated IgG
 FTA-ABS/TPHA/MHA-TP: all specific tests for T. Pallidum, thus if these are negative then you know it
cannot be neurosyphilis; often these are done with high clinical suspicion but negative prior testing
Tx:
 High-dose IV aqueous penicillin G (2-4million unites, every 4 hours for 2 weeks)
 Doxycycline + ceftriaxone is an acceptable alternative if penicillin allergy
 Eradication is confirmed with CSF testing returning normal; often neurologic losses are nor recovered
fully, but will not progress
Note that HIV(+) pts have a much higher risk for contracting syphilis and may result in an earlier presentation

Normal Pressure Hydrocephalus


Idiopathic, non-obstructive hydrocephalus that typically affects the elderly.
 Occurs episodically causing ventricular expansion and distortion of corona radiate
 Presentation: urinary incontinence, ataxia, cognitive dysfunction with “magnetic gait” (feet stick to
floor) with the classic “Wet, Wobbly, Wacky” presentation
 Often CT head with show abnormally large cerebral ventricles

HIV-associated Dementia
Progressive cognitive function deterioration ether caused primarily by the disease or by other infectious
diseases due to immune-deficiency
 Note that HIV is the most common infectious agent to cause cognitive impairment
 Worse prognosis with low CD4+, high viral titers, low body weight
Signs/symptoms:
 Classic early sign is stumbling/tripping with poor handwriting
 Memory problems, poor concentration/attention are common complaints
 MRI shows diffuse atrophy of the the brain
Dx: clinical suspicion, positive HIV testing; rule out other causes
 CT with an LP MUST be done, as HIV can predispose to meningitis infections that could cause these
signs/symptoms
Tx: with HAART should prevent this from happening due to preventing HIV progression

Creutzfeldt-Jakob Disease
Rare, invariably fatal, rapidly progressive dementia
 Thought to be caused by prion proteins, resulting in abnormal folding/degeneration
Signs/Symptoms:
 Earlier signs: Rapid (within months) deterioration of memory, personality changes, loss of
coordination, poor judgement, and visual disturbances
 Later signs: changes/loss of vision, myoclonus, exaggerated startle reflex, weakness, and coma
Dx:
 Rapid progression of clinical dementia symptoms
 CT: normal; typically used to investigate stroke/tumor as this disease is so rare
 MRI: “pulvinar sign” seen in v-CJD on T2-FLAIR
 LP: 14-3-3 protein found in the CSF
 EEG: periodic sharp wave complexes
 Biopsy/Autopsy: only true way to diagnose; may show spongiform changes and characteristic proteins
Tx: no treatment is effective; best to ease the pts symptoms and have strict protocols to limit transmission
(covering/protection, sterilizing all instruments to 269-273F)

Delerium
Acute organ failure of the brain, characterized by below; should resolve with underlying illness resolution
 Inattention/decreased level of awareness/disorientation
 Acute development of cognitive deficits
 Symptoms fluctuate throughout the day, worse at night
 Recent memory/language deficits
 Hallucinations/illusions (visual is most common)
 Disruption to circadian rhythm

Case 33 – Obesity
Diagnosis – most easily diagnosed via BMI (>30.0 is obese) with waist circumference and truncal deposition of
fat (“apple shaped figure”) increasing risk for complications based on obesity
 Labs: fasting glucose, fasting lipids, TSH, liver enzymes
 Search the body for acanthosis nigricans (a sign of glucose intolerance)
 Note that BMI is inaccurate in pts with heart failure (H20 retention), pregnancy, body builders/pro
athletes, and elderly patients
Pathogenesis – it really boils down to taking in more calories than expended. Sadly, this is easy to do in our
modern society. Obviously there are going to be patients with glandular problems and on medications which
make gaining weight extremely easy. Don’t chalk up obesity to laziness. Often it’s more complex than that.
Treatment – recommended to start at [BMI 25 w visceral obesity] or increased waist circumference (40cm in
men and 35cm in women)
 Diet/Exercise – creating a 500-1000 cal/d produces a weight loss of 1-2lbs/wk (theoretically). Most
diets focus on creating a calorie deficit with exercise for 30min minimum 5-7day/wk. Also, exercise
without diet change is NOT effective. YOU CAN’T WORK YOUR WAY OUT OF A BAD DIET.
 Pharmacotherapy – used with BMI>30 or BMI>27 with comorbid conditions (DMII, etc.)
o Orlistat is the only drug approved by the FDA for weight loss. It blocks pancreatic lipase and
effectively stops you from absorbing fat. You often get horrible steatorrhea (smelly greasy
diarrhea; thus nobody uses this awful drug) and Vitamin AEDK deficiencies
 Bariatric Surgery – used with BMI >40 with failure of diet/exercise or BMI >35 with co-morbid cond.
o Often pt must prove they will adhere to a special diet for weight loss before the surgeon will
perform the procedure. Many do not pass at this step sadly.
4164: Metabolic Syndrome [HIGH YIELD]
 Syndrome characterized by the following parameters:
o Abdominal obesity (men >40inch waist, women >35 inch waist)
o Elevated fasting glucose (>100-110 mg/dL)
o Mild hypertension (>130/80)
o Elevated triglycerides (>150 mg/dL)
o Low HDL cholesterol (Men <40, women <50)
 Insulin resistance is central to the pathogenesis AND chronically high insulin levels has huge effects on
the body as a whole, which is thought to be the major cause of these symptoms.

Case 34 – Headaches
Tension Headache
Your classic headache. Recurrent; bilateral/holocranial; tension sensation “like a vice”; ranges from 30 min –
several days; lack of nausea/exacerbation by physical activity.
 Dx: clinical, although must be careful not to just lump any headache into this
 Tx: NSAIDs/relaxation techniques; withdrawal from caffeine, barbiturates, and NSAIDs may cause
tension headaches so watch out for that!

Migraine Headache
Overview: a super shitty headache. These are fairly common AND debilitating so they’re important to know.
 May be triggered by various things; classics are red wine, cheese, chocolate, menses, and excess or
deficiency of sleep (many, MANY other things)
Signs/symptoms:
 Prodrome: often non-specific, but consistent symptoms occurring hours-days before a migraine
 Aura: typically precedes the headache from 5min-1hr but not during. Visual auras (Scotomas, zig-zags,
flashing lights, or visual distortion) are most common. Numbness, tinginling, aphasias, and hemiparesis
may also occur
 Headache: unilateral pain behind the eye/around the ear (4-8hr in length) usually with
pulsating/throbbing quality. Photo/phonophobia are classic signs; pt typically wants to lay in a dark,
quiet room and sleep it off
Dx: typically, Hx alone can clue you in. Blood studies, ESR, LP, and imaging are used mainly in ruling out other,
more serious diseases that can result in a headache
Tx:
Abortive therapy (stopping the migraine once it stops)
 Triptans (all end with “-triptan”): 5HT-1D serotonin receptor agonists and most effective therapy
o Common side effects: N/V, numbness, tingling
o Contraindications: concurrent ergotamine use, Hx of coronary artery disease/HTN
 Ergotamines: natural derivatives typically used if someone fails to respond to triptans
 Dihydroergotamine: similar to ergotamine but formulated for IM and intranasal use
 Midrin (acetaminophen, dichloralphenazone, isometheptene mucate) for analgesia, muscle relaxation,
and vasoconstriction (respectively); not the most common drug, but a good alterantive
Prophylactic therapy (stopping any onset) – used with >3 migraines per month
 Topiramate (anticonvulsant) is the mainstay of therapy; concern of sedation/numbness/tinginling
 Divalproex (anticonvulsant) an alternative but may have worse side effect profile
 Beta-blockers (propranolol) – a classic treatment but not the best tolerated in patients with normal
blood pressures (concern of syncope)

Cluster Headaches
 Symptoms:
o Pain is ALWAYS unilateral, frontal, retro-orbital (“side locked”); may recur up to 8 times a day
o Called “cluster” because these pop up over a course of weeks then go away for months-years
o Pain is constant, severe, non-pulsating pain (bad enough to cause suicide) - sometimes
described as “a hot poker in my eye”
o Makes patients restless and pace around the room or even banging head against the wall!
o May have unilateral conjunctivitis, rhinorrhea, Horner’s syndrome
 Treatment:
o Acute: subcutaneous sumatriptan with 100% nasal oxygen at 6L/min
o Prophylaxis: verapamil (Ca2+ channel blocker), lithium, valproic acid, prednisone

Chronic Headache
Basically any kind of headache that is occurring chronically/persistently (pt affected >15 days per month)
 Migraine/tension headaches are commonly chronic
 Occipital neuralgia: headache from inflammation of the greater occipital nerve (runs off cervical plexus
from C2 to innervate much of the posterior neck/head). Often bilateral and tender to palpation. May
be severe and patient perceives pain behind the eyes.
 Analgesia rebound (from long term NSAID use) is a classic reason to get a persistent headache. It’s
important to note pts prescribed and OTC drug history when evaluating persistent headache and to get
pt off all analgesia if this is the case
Non-medical treatment
 Physical therapy with head/neck rehabilitation
 Massage therapy has also been helpful in relieving tension
Medical treatment
 Removal of all OTC analgesics (concern of analgesia rebound)
 Addition of anti-headache therapies (triptans, ergots, topitamate, b-blockers, etc.) all can aid in
resolving these headaches
 Typically, anti-convulsants have the greatest success rate for daily chronic headaches

Epidural Hematoma
Defined: bleed between the dura and the skull, typically from a traumatic injury
 Commonly the middle meningeal artery (temple area) is damaged causing the bleed
Signs/symptoms:
 Classic is trauma (knocked out), with lucid period, then rapid deterioration/confusion
 Headache, N/V, pupillary abnormalities, lower GCS, or increased ICP can all present
Dx: trauma with evidence of “lens-shaped” bleed with smooth margins/does not cross suture lines on CT
 Check CBC/PT, PTT/Fibrin products to better understand bleeding tendencies
 Air in the epidural hematoma suggests rupture of mastoid air cells
Tx: stabilize (ABCs), supportive therapy, and reduction of ICP to stop herniation
 CT is always the first step of management
 Elevation of the head/Trendelenburg position to increased venous drainage of the head
 Consultation of neurosurgery for evacuation/repair

Subarachnoid Hemorrhage
Defined: hemorrhage within the brain resulting in abrupt onset of neurologic symptoms
 Sentinel Bleed: intermittent smaller hemorrhages/headaches preceding full blown hemorrhage often
occuring with physical/emotional strain, defication, sex, or head trauma
 Most common etiology is a ruptured saccular (berry) aneurysm (also has worst prognosis) so risk
factors for developing aneurysm are risk factors for this hemorrhage (PKD, marfans, HTN, diabetes,
mycotic aneurysms, oral contraception, etc.)
 Aterio-venous malformation, cocaine/amphetamine use, or trauma may also cause them
 Typically occur in anterior communicating artery or carotid artery bifurcation
 Up to 60% of pts die in the first month following the hemorrhage
Signs/symptoms: sudden onset of “worst headache of my life” with focal deficits or altered consciousness
Dx:
 CT: bleeding within brain parenchyma indicates acute hemorrhage
 LP: blood and xanthochromia indicates active bleeding into the CSF
 Transcranial Doppler/CTa/conventional angiography: may show the aneurysm in question
Tx:
 If neurosurgery is required: Angiography must be done to localize the aneurysm/bleed
 Endovascular coiling – superior to clipping and can be used at any time
 Clipping – needs to be used within 48hrs or after 2 weeks post-hemorrhage (timeframe in-between has
a high risk of vasospasm, thus we’re trying to avoid that)
 Triple H (hypertensive hypervolemic hemodilution) therapy with nimodipine – give aggressive fluids to
keep perfusion up; the calcium channel blocker is to reduce vasospasm

Intraparynchmal (incracranial) hemorrhage


Bleeding within the brain tissue parenchyma
o Presentation: severe headache, N/V, coma
o Trauma - the most frequent cause; a blow to the head causes blood vessel rupture
o Hypertension - a very common cause (some stats say it’s the most common)
 Charcot-Bouchard aneurysm - small vessel aneurysm in the brain from HTN
 Usually occur in lenticulostriate vessels (not seen on angiogram)
 Typically causes hemorrhage in the basal ganglia/thalamus
 Treating the HTN will reduce hemorrhage risk by 50%
o Arteriovenous malformation - these misshapen vessels have a tendency to rupture/bleed
 Treat with obliteration with coiling or surgical removal

Epidural hematoma
Rupture of middle meningeal artery (maxillary artery branch) often secondary to a skull trauma/fracture
(temple area)  high pressure hematoma
o Presentation:
 Traumatic injury with possible temporary loss of consciousness
 Lucid period (up to 48 hours) with headaches, nausea, hemiparesis
 Rapid expansion may cause transtenorial herniation (LOC, respiratory depression,
death) or CNIII palsy (down and out gaze) from compression
o Labs:
 Head CT showing lens-shaped bi-concave hyperdensity not crossing suture lines
o Treatment
 Surgical: immediate evacuation/pressure relief of bleed
 Medical: treat the raised ICP (mannitol, hyperventilation, steroids, ventricular shunt)
Giant-cell Temporal Arteritis
Granulomatous arteritis affecting medium-large arteries classically in people >age 50 in the extracranial
carotid branches (ophthalmic, temporal, etc.)
 Symptoms: jaw claudation with chewing, amaurosis fugax, fever, weight loss, fatigue, “hard”
prominent temporal artery
 Feared complication is anterior ischemic optic neuropathy causing irreversible blindness
 Dx: elevated ESR, elevated C-Reactive Protein, segmental biopsy of temporal artery showing
granulomatous vasculitis (normal biopsy does NOT exclude this disease)
 Tx: immediate high dose prednisone (corticosteroid) to preserve vision

Suspected Meningitis
Sign/Symptoms: classic triad of headache, fever, and neck stiffness
 Macropapular rash  suspect Neisseria gonorrhea
 Kernig’s sign/Brudzinki sign are classics indicating meningitis
Disease Cell Protein Glucos Other Findings
e
Bacterial Meningitis PMNs (neutrophils) High Low Culture/Gram stain may be positive
Viral meningitis Lymphocytes High Normal Viral PCR may be positive
/Encephalitis

Tx:
1. Initiate empiric antibiotic/antiviral Tx  IV ceftriaxone + IV vancomycin + IV acyclovir and if pt is a child
+ dexamethasone (steroids avoid the possible deafness/other losses seen in children)
2. Schedule STAT CT (check for masses/abnormalities) and lumbar puncture with opening pressure
a. Note that papilledema means there’s increased ICP, meaning there is a higher suspicion of
brain mass, thus a CT MUST be performed due to risk of LP causing herniation
3. If etiology not confirmed, look for things like fungi/TB as causative agent
4. Alter treatment based on confirmed etiology with supportive care

Pseudotumor Cerebri (idiopathic intracranial hypertension)


Classically found in young, obese women of child-bearing age; sometimes with menstrual irregularity
 CT with contrast: Slit-like ventricles, enlarged optic nerve sheath, empty sella
 High volume LP (pt laying on side) shows increased opening pressure with normal CSF
 Papilledema noted on funduscopic exam (vision greying out/obscuration are common signs)
 Other focal neurologic signs may be present
o Headache: diffuse, worse in morning, worse with Valsalva maneuver
o N/V, 6th neve palsy, tinnitus, diplopia, and others are all possible
Tx:
 Emergent: high volume lumbar puncture to acutely lower ICP
 Long-term: acetazolamide (carbonic anhydrase inhibitor; will result in increased respirations, lowering
ICP) or nerve sheath fenestration surgery or shunt placement (aid in CSF drainage)

Medication related headache


Don’t forget to review medications and how the patient is using them when they complain of headaches!
Classically a medication overuse or “rebound” headache can occur with NSAIDs, Acetaminophen, Aspirin,
Caffeine use, and many other medications!

Case 35 – High Cholesterol


High cholesterol is NOT a disease in itself, but is a major modifiable risk factor for CHD
 A high cholesterol should be interpreted within the context of overall coronary heart disease risk
 Those with higher overall risk will have more aggressive cholesterol goals
 Typical cholesterol levels:
o LDL cholesterol (the bad one) optimal level is <100 mg/dL
o HDL cholesterol (the good one) optimal level is >40 mg/dL
o Total cholesterol desirable levels are <200 mg/dL
 Adults older than 20yr should have lipids checked every 5 years by a fasting lipid panel
Determining an LDL cholesterol goal
 Based on listed risk factors below:
o Cigarette smoking
o Hypertension (either >140/90 or currently on any medication for hypertension)
o Low HDL (<40 mg/dL)
o Age >45 for men, >55 for women
o Family Hx of CHD
 Note that a high HDL (>60 mg/dL) is a “negative risk” which neutralizes any one of the risk factors
Risk Categories and LDL goals
 CHD or equivalent present: <100 LDL; medications used if LDL >130
 2+ risk factors: <130 LDL; medications used if >160
 0-1 risk factors: <160 LDL; medications used is >190
 CHD and multiple risk factors: <70 LDL; medications always used here
Management
 Best to initiate therapeutic lifestyle goals (smoking cessation, lower dietary cholesterol/saturated fat,
increased dietary plant fiber, etc.) before needing medications
 Pharmacotherapy is used when a patient cannot meet their goals with TLCs alone
o “Statins” (HMG-CoA reductase inhibitors) – vvvLDL, ^HDL, vTRI
 hepatotoxicity/myopathy (esp. with fibrates/niacin)
o “Choles” (Bile Acid Resins) – vvLDL, HDL, TRI
 diarrhea/Vit AEDK deficiency
o Niacin (Vit. B3) - vvLDL, ^^HDL, TRI
 Flushed face (aspirin blocks it), hyperglycemia/hyperuricemia
o “Fibrates” (Fibric Acids) – vLDL, ^HDL, vvvTRI
 gallstones, myopathy (esp. with fibrates/niacin)
o Ezetimibe (Cholesterol absorption blocker) – vvvLDL, ^^HDL, vTRI

2698/3823/4336: Guidelines for Lipid-Lowering therapy (typically a statin)


 Symptomatic atherosclerotic disease  high or low intensity based on age (> or < than 75yr)
 LDL >190mg/dL  high intensity statin therapy
 Age 40-75 with diabetes  high or low depending on 10-year ASCDV risk
 Estimated 10-year ASCVD risk >7.5%  moderate to high intensity statin therapy
[Note that the difference between statin intensity is the dose]

3822: (refer to 2698) A patient with Type II Diabetes Mellitus over the age of 40 should always be on a statin
and initiate positive lifestyle changes according to lipid lowering guidelines. Refer to 2698 for the complete
guidelines.

4227: Statins inhibit HMG-CoA reductase thus decreasing intrinsic cholesterol synthesis  forcing increased
cholesterol uptake from blood to meet demands on the liver. Side effects include:
 Myalgias – common, may have increased creatine kinase; thought to be due to decreased CoQ10
synthesis which is necessary for muscle energy production
 Liver dysfunction – rare but serious

3158: There are two major side effects of statin therapy and they’re both have lab values:
 Muscle damage: elevated CPK with myalgias which may progress to severe rhabdomyolysis and renal
failure (would acutely raise BUN and Creatinine). Stop the statin if this happens
 Liver damage: not as pronounced usually, but can cause elevated liver transaminases (ALT/AST).
Usually these will trend back to normal with stopping the statins.

Case 36 – Family Violence


Intimate Partner Violence
 Obviously spousal abuse doesn’t just happen to women, but can happen in any relationship
 Regular screening for this type of abuse is recommended at any regular checkup/gynecologic exam
 Some Physical Signs of Abuse include
o Contusions, fractures, black eyes, internal bleeding; esp. if treatment seeking is delayed
o New STI, anal/pharyngeal trauma, or unintended pregnancy
o Emotional distress, new depression/anxiety
o Chronic abdominal pain, new substance abuse or eating disorder
 When asking about violence
o Use direct questioning with the patient in the room without the abuser
o Offer resources/numbers to call in a non-directive way
 Reporting varies by state, but apparently Tennessee mandates suspicion of intimate partner violence
to report such suspicions to proper authorities

Child Abuse
 Often unthinkable to a reasonable person, there are several things that could cause an increased risk
for abused of a child including parental depression, substance abuse, social isolation, stress, low
income, poor access to recreational services
 Some signs of child abuse:
o Aggression, anxiety, bedwetting, depression, regression, advanced sexual play
o Burns: Stocking-glove burns, buttock/genital burns, cigarette burns
o Shaken baby syndrome: Retinal hemorrhage, lethargy, spinal/neck trauma
o Fractures: “bucket-handle fractures”, “Spiral fractures”, posterior rib fractures, scapular
fractures, spinous process fractures, sternal fractures, complex/big skull fractures
o Sexual: genital injury, STI or genital warts, circumferential anal hematoma
 You ALWAYS report child abuse if in good faith, no matter what.

Elder Abuse
 Typically as physical/sexual/psychologic abuse, neglect, or financial exploitation
 Women >75yr and those cognitively/physical impaired are at highest risk
 Best to interview elder alone to try and assess abuse

Case 37 – Limping/Leg Pain in Children


Diagnostic Approach
 Pain vs No Pain
o Pain – start exam focused on that area, then branch out from there
o No pain – examine the back, pelvis, buttock, leg, and foot
 Evaluation of the hip is MOST important as the hip is the most likely to need emergent eval.
o Restricted internal rotation of the hip – most sensitive marker of hip pathology
o Lack of abduction – 2nd most sensitive marker for hip pathology
 After localizing pain/likely spot of pathology x-ray imaging and labs should be sought
o Infection or Rheumatologic cause suspected – CBC, CRP and ESR
o Gonorrhea testing of aspirated fluid in teens or Lyme disease in endemic is OK to do
 Limping without pain
o Leg length discrepancy (measure umbilicus to medial malleolus)
o Muscle atrophy/limb deformity or congenital malformation (inspection/movement)

Infants/Toddlers
 Septic arthritis – crying/irritability/fever, monoarticular with swelling/heat from joint. Often child
refuses to bear weight on affected joint (increased swelling/pressure in joint capsule!)
o Labs – elevated WBCs, reactive ESR/CRP
o Micro – GBS/Staph (<4mo); Staph/Strep (<5yr)
o Dx – joint aspiration with gram stain/culture
o Tx – urgent surgical irrigation, debridement, and antibiotic Tx
 Fractures – acute onset of limp or simply refusal to walk/bear weight on injured bone
o Screening for abuse is clever, but Hx may not reveal much
o Dx – plain film of suspected area of fracture
o Tx – immobilization for healing
 Congenital Dysplasia – painless limp/odd ambulation from time child learns to walk
o All children’s hips should be examined for dysplasia/stability/deformity
o Dx – plain films showing misalignment of the hips
o Tx – if early, splinting for alignment; if late, surgical correction
Young Children
 Transient Synovitis – self-limited inflammation, classically of the hip, in children ages 3-10yr typically
following a viral infection
o Labs – no fever, normal CBC, normal ESR/CRP and normal X-ray
o Dx – clinical presentation
o Tx – watch with followup; should resolve within a few days, although a septic joint may develop
o Kocher Criteria for Septic Arthritis Risk in Children
 Septic – Aspirate is purulent, WBC count >50,000/mcL
 Transient synovitis – Aspirate is yellow/clear with WBC count <10,000/mcL
 Legg-Calvé-Perthes disease – avascular necrosis of the head in boys ages 4-8yr of unknown origin
o Gradual onset of hip/knee/thigh pain and limping over a few months
o X-ray – normal  collapse/flattening/widening of femoral head with inflammation
o MRI – necessary to confirm inflammation and make diagnosis
o Tx – conservative treatment with therapy to maintain range of motion

Adolescents
 Slipped capital femoral epiphysis – slipping of the growth plate causing displacement
medially/posteriorly often without acute injury. Often in overweight adolescent boys
o Limited internal rotation with obligate external rotation upon hip flexion
o X-ray – epiphysis widening  slippage of the femoral head
o Tx – surgical pinning of the femoral head to stop slippage
 1/3 will develop avascular necrosis of hip
 1/3 will develop SCFE in the contralateral hip
 Sprain/strains/overuse injuries – the most common cause of limb pain in this population
 Gonococcal arthritis – get a sexual history and aspirate the joint if necessary

Any Age
 Pain at night – suspicious for malignancy
 “Growing pains” – a dx of exclusion that occurs only at night (NOT DURING DAY) bilaterally in the
absence of any other pathology
Case 38 – Post-Operative Fever
Drug Fever (Any Time)
 Classically associated with rash or Lupus-like symptoms; may involve any organ
 Drugs Associated:
o Antibiotics (1/3 of cases): minocycline, cephalosporins, floroquinolones, sulfa drugs, penicillins
o May happen with ANY drug; typically more drugs = more chance to occur
 Often resolves within 72-96hr after stopping the offending drug

Surgical Trauma (within 24hr)


 Cutting stimulates cytokine release, thus bigger surgeries have a higher risk for this type of fever
 Often mild, but NSAIDs or steroids can help increase patient comfort

Malignant Hyperthermia (within 24hr)


 Rare autosomal dominant disorder causing severe reaction to anesthesia (halothane and
succinylcholine are classics associated with this reaction)
 Fever (104oC), tachycardia, metabolic acidosis, rhabdomyolysis/muscle rigidity
 Treated with – stopping anesthesia, physical cooling, IV dantrolene, O2 supplementation, antipyretics

Bowel Leakage of Flora (within 36hr)


 Can invade the peritoneum resulting in a soft tissue infection
 Major signs/symptoms will be from specific bug causing infection

Toxic Shock Syndrome (within 36hr)


 Bacterial infection (S.aureus most common) with Toxic-Shock Toxin-1 (TSST-1), a super-antigen that
results in massive T-cell activation/cytokine release, resulting in loss of intravascular fluids to the
extravascular spaces and shock!
 Presentation: rapid development of high fever, Hypotension (<90 systolic), diffuse red/edematous rash
(looks like sunburn), skin desquamation of palms/soles, multisystem failure
o Classically (50%) associated with a tampon left in post-menstruation, typically arising 2-3 days
after the last menstrual period
o Non-menstrual infections can also cause it (post-surgery, sinutitis, etc)
 Dx: clinical syndrome with history
 Tx: supportive therapy for shock, removal of foreign materials causing infection, anti-staph (penicillin +
vancomycin) are all part of treatment
Pneumonia (within Day 5)
 Often atelectasis (alveolar collapse) can occur after surgery leading to pulmonary complication,
especially in those with lung conditions (COPD, asthma, smoking, old age, etc.)
 Anesthesia and narcotic use can also decrease a patient’s breathing, thus increasing pulm. Risk
 Patients on ventilators are a HUGE risk for VAP (considered a hospital complications)
 Aspiration may occur in patients with GERD or difficulty swallowing (gram negative bugs)

UTI (within Day 5)


 Increased risk with difficulty voiding; if patient is catheterized, it needs to be discontinued
 Often IV antibiotics are used, esp. in patients with complicated UTI

DVT/PE (Day 5)
 DVT: Often asymptomatic but classically with leg tenderness, pain, warmth, and asymmetry
 PE: often with pleuritic chest pain and rapid-onset SOB
 Prevented with compression stockings/anti-coagulation

Surgical Site Infection (Day 5 – 7)


 Local erythema, discharge, pain, purulent discharge at the site of incision
 Prophylactic Abx (1hr before operation; discontinued within 24hr) can decrease the risk
 Sterile technique and careful operating procedure in the OR is the other way to reduce risk

Abscess (Day 10 – 15)


 Pretty much the same as an SSI, but with fluctuant mass that needs drainage
 Rare, but fever may be the presenting sign

IV or Catheter Infection (3 Days or More present)


 Basically these are great sites for bugs to crawl in and set up infection
 IV/catheter sites should be assessed EVERY DAY for signs of infection and removed ASAP when not
necessary anymore. The chance for infection is proportional to the time the line is left placed.
 If infection is suspected, line is discontinued IMMEDIATELY and the tip is cultured for diagnosis

Specific Risks of Specific Surgeries


 Laproscopic surgery – lowest risk of infection due to minimal invasion
 Cardiothoracic surgery – all patients will develop pleural effusion due to incision (5% get pneumonia)
 Abdominal surgery – deep abdominal abscess & pancreatitis
 OBGYN surgery – post-op endometritis, pelvic abscess, pelvic thrombophebitis
 Orthopedic surgery – highest rate of surgical site infection
 Urologic surgery – prostatic/perinephric abscess
 Neurosurgery – increased DVT due to immobilization/less aggressive anticoagulation (protect brain)

3950: Cholesterol Crystal Embolization typically occurs after cardiac catheterization or recent vascular
procedure in the setting of cardiovascular risk factors.
 Features: livedo reticularis, ulcers/gangrene, blue toe syndrome, renal injury, stroke, Hollenhorst
plaques (in the eye), GI ischemia/pancreatitis
 Dx:
o Labs: elevated creatinine (renal damage), eosinophilia/eosinophiluria, hypocomplementemia
o Skin/renal biopsy: biconcave, needle-shaped cholesterol clefts in occluded vessels and
perivascular inflammation with eosinophils

2310: After and MI successfully treated with cardiac catheterization, there’s a good chance, especially with risk
factors, that cholesterol plaques could rupture and send cholesterol emboli showering throughout the body
causing ischemia. This can be manifested through livedo reticularis, blue toe syndrome, renal failure,
gangrene, ulcers, Hollenhorst plaques (yellow, shiny plaques in the retina), stroke, or GI bleeding.
 Tx: statin therapy with support for any other problems
 May happen immediately or delays (up to 30 days after)

Case 39 – Wheezing in Children (NOT asthma)


Case 40 – Irritable Bowel Syndrome (IBS)
Typically affects women 2-3x more than men often occurring in teens/20s

Has an odd relationship with psychiatric illness, where IBS is often worsened by psych illness
 Treating psych illness can often improve IBS symptoms but will not resolve them completely

4595: Irritable Bowel Syndrome (IBS)


 Common functional GI tract disorder with no identifiable cause. A diagnosis of exclusion
 Presentation: chronic, crampy abdominal pain with alternating episodes of constipation and diarrhea.
Passage of stool relieves the pain.
o Classically occurs in young women
o Alarm signs (rectal bleed, severe pain, weight loss, or abnormal labs) should be investigated
further as they point toward something else, NOT IBD
 Dx: Rome III criteria
o Recurrent abdominal pains for >3 days per month
o Two of the following:
 Symptoms improve with bowel movement
 Changes in frequency of pooping
 Changes in forms of poop (diarrhea/constipation)
 Tx: Diet-adjustment & treat symptoms (diarrhea = anti-diarrheals; constipation = fiber or laxatives)

Case 41 – Substance Abuse – likely the worst written chapter in this book; my notes are from psych rotation
Alcohol Intoxication
 Works as a CNS depressant via: Activating GABA/DA/serotonin & Inhibiting glutamate
 Depends on Blood Alcohol Level (BAL)
o Buzzed
 20-50: decreased fine motor control
 50-100: impaired judgement/coordination
o Drunk
 100-150: ataxia and poor balance
 150-250: lethargy, slumped position, anterograde amnesia, nausea/vomiting
o Alcohol Poisoning
 300 or more: respiratory depression, coma, death
 Treatment
o Monitor: airway, breathing, circulation, glucose, electrolytes, acid-base status
o Give:
 Thiamine (B1)/Folate to prevent Wernicke's encephalopathy (FIRST THING TO DO)
 Naloxone as opioids are commonly co-ingested with alcohol with alcoholics
 D5 dextrose with electrolytes
o Scan:
 CT to rule out brain damage/bleed
o Support:
 Mechanical ventillation if pt is respiratory depressed
 GI evacuation only if significant amount of EtOH was ingested within last 30-60 minutes
Alcohol Withdrawal
 Onset between 6-24hr of last drink lasting 2-7 days (depends on severity of alcoholism)
o Mild: irritability, tremor, insomnia
o Moderate: Sweating, HTN, tahycarida, fever, disorientation
o Severe: Seizures, Delerium Tremens, Alcoholic Hallucinosis
 Time since last drink:
o  6-24 hr: Present with alcohol withdrawal symptom
o  12-48 hr: withdrawal may cause tonic-clonic seizures (hypomagnesemia may precipitate this)
o  48-96 hr: delerium tremens may occur
 Tx:
o Mild agitation: Benzos with slow taper (keep patient calm/lightly sedated)
o Severe agitation: Benzos with possible antipsychotics and physical restraints
o Support:
 "Banana bag" - thiamine/folate/multivitamin
 Correction of any fluid/electrolye abnormalities
o Monitoring:
 Use CIWA scale to monitor withdrawal symptoms
 Check for any trauma/brain damage
 Check for hepatic failure
Tx for Alcohol Abuse
 First line for drug aid:
o Naltrexone (Revia, IM-Vivitrol) - opioid antagonist; decreases craving/"high" of alcohol
consumption
o Acamprosate (Campral) - glutamate modulator; used post-detox to prevent relapse; OK in liver
disease, NOT OK in renal disease
 Second line for drug aid:
o Disulfiram (Antabuse) - blocks aldehyde dehydrogenase causing flushing/nausea/vomiting with
alcohol consumption. 
 Contraindicated in heart disease, pregnancy, psychotic episodes
 LFTs must be monitored
 Only use in highly motivated patients
o Topiramate (Topamax) - potentiates GABA/inhibits glutamate receptors; reduces alcohol
cravings

Marijuana
 Intoxication
o Euphoria, perceptual disturbances, conjunctival injection (red eyes), increased appetite
("munchies"), dry mouth
o Hallucinations may occur
o Anxiety/paranoia may occur in the naive user
 Overdose/Chronic Use
o You can't seem to overdose on cannabis
o Chronic Use Found in 50% of daily users
 If smoking: chronic bronchitis/asthma
 Any route of administration: immune system suppression, decreased reproductive
hormones, cancer
o Tx:
 Support/psychosocial intervention
 Treat any other condition precipitated by use
 Withdrawal
o Irritability anxiety, restlessness, strange dreams, depression, insomnia, decreased appetite

Cocaine
 Intoxication
o Mimics a fight or flight response or a manic state
o General: euphoria, high self-esteem, low/high blood pressure, tachy/bradycardia, dilated
pupils, agitation, chills/sweating
o Dangerous: Respiratory depression, seizure, arrhythmia, hyperthemia, paranoia, hallucinations
(tactile; “cocaine crawlies”)
o Deadly: intense vasoconstriction causing MI/stroke/intracranial hemorrhage
o Tx:
 Mild: reassurance/benzodiazepines
 Severe/psychosis: benzo/antipsychotics
 Symptomatic support: control blood pressure (cautious with alpha-blockers), arrythmias
(cautious with anti-arrythmatics), high fever (aggressive with ice-bath, cooling blanket,
other supports)
 Withdrawal
o Post-intoxication "crash" - malaise, fatigue, hypersomnolence, depression, anhedonia, hunger,
constricted pupils, vivid dreams, suicidality
o NOT life threatening (remember how stimulant drug withdrawal is awful, but won't kill you?)
o Should resolve within 3 - 14 days depending on use
o May need hospitalization for supportive measures
4042: Any person (especially younger) presenting with agitation, dilated pupils, atrophic nasal mucosa, HTN,
and acute myocardial ischemia should be suspected for cocaine abuse!
 a/B adrenergic stimulation = HTN/pupil dilatation/agitation/ischemia
 Cocaine also potentiates thrombus formation
o Cocaine + alcohol = cocaethylene  increased risk of coronary vasospasm
 Tx: immediate benzodiazepines/O2 + aspirin, nitrates, calcium channel blockers
 NEVER GIVE A B-BLOCKER: it will cause unopposed a-adrenergic activity and HTN crisis!

Amphetamines
 Intoxication
o Similar to cocaine (due to shared DA/NE increase); euphoria, dilated pupils, tachycardia,
diaphoresis, chest pain
o Unique feeling of closeness/love with others, ^libido
o Teeth grinding 
o Tx: Rehydration, electrolyte correction, hyperthermia treatment if necessary
 Overdose
o Hyperthermia, dehydration (esp. in a hot club), rhabdomyolysis, renal failure
o Ongoing psychosis (due to long half-life)
 Withdrawal
o Prolonged depression

Opioids
 Intoxication
o Constricted pupils, respiratory depression, constipation, altered mental status, nausea/vomiting
o May progress to coma/death in OD
o Tx
 Ensure ABCs are intact
 Ventillatory support if necessary
 Naloxone (opioid antagonist) for overdose (may precipitate withdrawal!)
 Withdrawal
o Dysphoria, insomnia, lacrimation/rhinorrhea, sweating, goose-bumps, dilated pupils, abdominal
cramping, myalgia/arthralgia, HTN, tachycardia, intense cravings
o Unpleasant, but NOT life threatening
o Tx
 While not life-threatening, it's quite horrible, so we try to help pts get through it
 Moderate symptoms: manage with common drugs
 Severe symptoms: Detox with Methadone (long acting opioid agonist)/buprenorphine
(partial opioid receptor agonist)

PCP
 Intoxication
o Depersonalization, hallucinations, impaired judgement, amnesia, ataxia, muscle rigidity,
dysarthria
o Synesthesia (Sensory mixing: hearing a sound produces a visual color)
o Agitation/aggression/assault/high pain tolerance (often violent seemingly 'immune to pain')
o Nystagmus (rotary is most characteristic, can also be vertical or horizontal)
o Hallucinations (visual and tactile)
 Overdose
o Seizure, delirium, coma, death
o Tx:
 Monitor vital signs/temperature/electrolytes
 Minimize sensory stimulation
 Benzodiazepines to treat agitation/muscle spasms/seizure
 Antipsychotics for severe agitation/psychoses
 Withdrawal
o None; but pt may have "flashbacks" due to storage of drug in adipose with gradual release

Inhalants
 Intoxication
o Only lasts 15-30 minutes typically
o Perceptual disturbance, paranoia, lethargy, dizziness, headache, hypoxia (if breathing in a lot),
stupor, coma, etc. (CNS depressant)
o Typically used by teenagers (easy substances to access)
 Overdose
o Respiratory depression/cardiac arrythmia (fatality)
 Tx: 
o ABCs, may need hyperbaric oxygen if hypoxia is present
o Identify the inhalent as it may need chelation (leaded gasoline is a good example)

Hallucinogens
 Intoxication
o Perceptual changes (illusions, hallucinations, synesthesia)
o Labile affect, dilated pupils, tachycarida, HTN, hyperthermia, sweating, palpitations
o High typically 6-12 hours but can last for days
o A "bad trip" is a high consisting of anxiety, panic, or psychoses
 Withdrawal
o No real withdrawal symptoms
o "LSD flashbacks" may occur later in life spontaneously

Case 42 – Heart Palpitations


Workup
 History/Physical (don’t forget psych screening, caffeine/drug use!)
 12-lead EKG  if abnormal a 24hr Holter monitor/echocardiogram/stress test may be used
 Labs: CBC, BMP, urine drug screen, TSH/T3

Primary Rhythm Disturbances – 41%


 Paroxysmal Supraventricular Tachycardia (PSVT)
o Abrupt onset of tachycardia (60-200 bpm) sometimes with quick movement or in response to
atrial fibrillation (A-fib with RVR!)
o Must rule out arrhythmias with an EKG before calling this benign

 4894: Wolff-Parkinson White syndrome occurs due to an abnormal accessory pathway (bundle of
Kent) that connects the atria/ventricles, by-passing the AV node. This causes pre-mature ventricular
excitation (delta wave of the ECG), and can result in a re-entrant circuit resulting in supraventricular
tachycardia (chest fluttering or palpitations are usually described by the patient) and possible sudden
death.

 3069: In Wolff-Parkinson-White (WPW) syndrome, an abnormal accessory pathway between the atrial
and ventricles that bypasses the AV node (slow down for coordination) is present
o Atrial fibrillation is a common, and possibly deadly, arrhythmia in WPW; the quick atrial
depolarization means you’re getting consistent quick ventricular impulses due to the accessory
pathway. Thus AFib can deteriorate in the ventricular fibrillation  sudden cardiac arrest!
o Tx:
 Hemodynamically unstable  immediate electrical cardioversion
 Hemodynamically stable  IV ibutilide or procainamide for rhythm control are used
 Brugada syndrome
o Autosomal dominant genetic disorder Na+ channelopathy
o Often recognized in Asian males, average age 40
 Usually you get ventricular tachycardia causes while asleep (Lai Tai)
 Causes syncope or sudden cardiac death
o Malfunctioning Na+ channels cause blunted sodium currents
 Shortened/failed action potentials or conduction block
 This gives a greater risk for the potential to “loop back in on itself”, causing a recurrent
stream of firing called re-entrant ventricular tachycardia. This is essentially because the
impulse is small and you get a faster recovery and second firing.
o Generally an abnormal EKG is found at rest called “coved type Brugada pattern”
 Persistent >2mm elevated ST segments in V1, V2, V3
 Right bundle Branch Block
 Inverted T-waves present in some leads
 J-point elevation
o A ICD defibrillator that will automatically shock the patient’s heart is the treatment to reset this
arrhythmia

 Long QT interval
o Remember! The QT interval is the phase when the ventricle contracts and relaxes
o Present more often in childhood; while kids are playing/exercise
o This interval (QTc) should last 0.44 seconds
o Many things can cause this to happen (IHD, Low K, Ca, Mg, channelopathy, others)
o We see a pathologic increase in phase 2 ventricular contraction
o Three different major types:
 Congenital long QT Type 1 (LQT1) –IKS
channel fail to open (no outward K+)
 Congenital long QT Type 2 (LQT2) – IKR
channel fail to open (no outward K+)
 Congenital long QT Type 3 (LQT3) – INa
channel fail to close (too much Na+ in)
o The variable repolarization time screws up
coordination causing ventricular tachycardia
o Characteristically we see a torsades de points
(twisting around points) pattern on EKG due to early
after-depolarizations from multiple sites
o A ICD defibrillator that will automatically shock the patient’s heart is the treatment to reset this
arrhythmia

 Sick Sinus Syndrome


o Arrhythmias due to SA node dysfunction, typically from valvular disease/cardiomyopathy
o Typically pt has sinus bradycardia, which can lead to arrhythmia although other variants are
possible

 Premature Ventricular Contractions (PVCs)


o Benign, but abnormal contractions where the ventricle fires too early, leading to a “fusion beat”
o Can be part of normal variation but if several of these are occuing consistently, a more
thorough cardiac workup should be sought

Mental Health Problem – 31%


 Likely there are more than outlined below, but these are common ones to cause panic palpitations
 Panic Attack
o A sudden surge of anxiety, which may be triggered or spontaneous
 intensity peaks within a few minutes
 resolves within 30 minutes
 anxiousness may persist for hours after attack is resolved
o Symptoms
 Dizziness, detachment from self/reality
 Heart palpitations/chest pain
 Shortness of breath
 Abdominal distress/nausea
 Fear of going crazy/losing control
 Sweating, chills, shaking
o It can look a lot like a heart attack! But the EKG/proteins will be normal
 Panic Disorder
o Disorder of spontaneous, recurrent panic attacks
 Attacks are often sudden ("out of the blue")
 Pt may also experience attacks with a trigger, but this is not necessary for the disease
 Pt has debilitating worry of another attack ("fear of the fear")
 Pt may display 'avoidance behaviors' to avoid another attack -> can lead to odd behavior
like agoraphobia!
o Often appears with Major Depression (65%), other anxiety disorders, bipolar disorder, and
alcoholism
o Criteria
 Recurrent, unexpected panic attacks without an identifiable trigger
 At least one attack followed by >1 month of worry of further attacks/maladaptive
changes to behavior
 Not caused by a substance/medication/disease/another mental illness
o Tx: Pharmacotherapy + CBT = best results
 First line: SSRI bridged with a benzodiazepine if necessary
 Second line: TCAs
 Remember to start low and go slow with the SSRI; the initial side effects may worsen
anxiety in panic disorder

 Generalized Anxiety Disorder


o Disorder of general anxiety about their daily lives
 Pts have somatic symptoms (fatigue/muscle tension)
 Often present to their primary care doc initially
 Anxiety is not fixated on a specific trigger, but part of everyday life
o Criteria
 Excessive anxiety/worry about daily events for >6 months
 Difficult time controlling the worry
 >3 of the following symptoms:
 Restlessness, fatigue, impaired concentration, irritability, muscle tension,
insomnia
 Symptoms cause significant impairment/not cause by anything else
o Tx: CBT + SSRI or SNRI
 Short-term benzos, buspirone, TCAs, and MAOIs are all other viable options

Drugs (prescription or recreational) – 5%


Alcohol, caffeine, cocaine, tobacco, OTC decongestants, OTC weight loss drugs, diuretics (cause electrolytic
disturbance), digoxin, albuterol, theophylline, phenthiazine

Intrinsic Structural Defects in the Heart – 3%


 Likely many ways for this to occur. Some Quick notes on Cardiomyopathies are below
 Hypertrophic Obstructive Cardiomyopathy (HOCM) – covered earlier
 4061: Dilated cardiomyopathy is CHF due to myocardial damage resulting in dilation of the ventricles
and diffuse hypokinesia (ventricles can’t pump as well, thus heart failure occurs)
o Dx: classically acute heart failure in a young person suggests dilated cardiomyopathy
 Recent viral infection is a common precipitant (3-5%) due to direct viral and immune
response damage to the myocardium (Coxsackie B, Parvovirus B19, HHV-6, adeno, and
enteroviruses)
 Thiamine deficiency (wet beri beri) is another classic cause
 Dilated ventricles with diffuse hypokinesia on echocardiogram confirms diagnosis
o Tx: supportive for CHF and tx for precipitating factor
 4238: Tachycardia-medicated cardiomyopathy: with a long-standing tachyarrythmia of any nature (a-
fib, a-flutter, rapid ventricular rhythm, etc.) the myocardium can undergo responsive changes
eventually leading to heart failure
o Symptoms: tachyarrhythmia (of any nature), CHF signs, heart palpitations
o Dx: abnormal EKG, rule out CAD, echocardiography showing diffused changes/hypokinesia
o Tx: aggressive rate/rhythm control of arrhythmia (ablations, AV node blockaid, anti-
arrythmatics)
 2699: Restrictive cardiomyopathy: uniform/symmetric thickening of the myocardium typically starting
with prominent right heart failure, eventually progressing to right and left heart failure.
o Etiology: sarcoidosis, amyloidosis, hemochromatosis, endomyocardial fibrosis, or idiopathic
o Hemochromatosis (iron overload) is the only REVERSIBLE cause, treated with phlebotomy
o Sarcoidosis/amyloidosis are both slowed with corticosteroids but cannot be stopped!

Non-Cardiac – 4%
 Anemia – low O2 carrying capacity means the heart ramps up efforts to deliver blood
 Hyperthyroidism – high T3/T4 means increased heart rate, which could lead to palpitations
 Hypoglycemia – a classic sign for diabetics that they’re sugars are low. Kind of like anemia
 Hypovolemia – like anemia. Heart compensating for inadequate blood volume
 Fever – tachycardia may be the result of vasodilation/leakage of fluid into extravascular space in
response to inflammatory cytokines causing widespread inflammation
 Pheochromocytoma – release of catecholamines ramps up heart activity
 Pulmonary disease – likely other underlying signs. Heart could be distorted from abnormal blood flow
or poor oxygen saturation may lead to cardiac compensation
 Vasovagal syncope – basically you’re BP dropped neurogenically, thus your heart is compensating

Idiopathic – 16%
 Sometimes we just don’t know. A good negative workup can only make this diagnosis

Case 43 – Bites and Stings


Insect Stings
 Most commonly from Hymenoptera insects (wasp, yellow jacket, hornet, honeybee, bumblebee, fire
ant) resulting in a reaction that may range from local to anaphylactic
 Approach
o Hx – focus on details of sting and ID the probable organism causing sting
o Physical – look for both local signs and systemic signs of reaction
o Remove stinger preferably by brushing (less chance of squeezing venom out!)
 Local reactions
o Redness, swelling, pain, itching at site of injury due to histamine release triggered by venom
o Onset is immediate lasting a few hours
o Tx: ice and antihistamine for itching + tetanus prophylaxis if not vaccinated
 Delayed Reactions
o Larger area of redness/warmth (10cm) at the site of sting due to IgE mediated reaction
o Onset is between 24-48hr lasting for 3-5 days
o Note that there is NOT an increased risk of anaphylactic reaction if a pt has a delayed rxn
o Tx: oral corticosteroids (decrease inflammation) + tetanus prophylaxis if not vaccinated
 Anaphylaxis
o Mild – nausea, generalized urticaria, angioedema
o Severe – hypotenion, shock, laryngeal edema, death
o Onset typically immediately after sting due to immediate hypersensitivity reaction
o Tx: ABC management as needed (intubation if necessary), IV access, fluids
 SubQ or IM epinephrine given IMMEDIATELY and 10-15 min after if symptoms not
improving. Immediately Epi-Pin use is often a good choice for patient
 Antihistmaines, steroids, and broncodilators may also be needed
 Hospitalization for 12-24hr to monitor response
 Prescription of epinephrine pin and instruction on staying away from stings;
desensitization therapy may also be an option

Animal Bites
 Approach
o ABCs, protection of current injury (splints of fractures, etc), and control bleeding
o Cleaning with soap+water, Saline irrigation of wound, and debridement of dead tissue
o Hx – in attempt to know what animal bit them/why they were bitten
o Control of infection
 Rabies – typically from bats, skunks, foxes, and dogs  washing + double prophylaxis
 Tetanus – given to patients who are not currently vaccinated
 If deep/puncture, likely primary closure should not be done to allow for drainage
 5-7 days of Augmentin (Amoxacillin-Clavulonic acid)
o Micro
 Human bites – most common bug = Eikinella corrodens (30%) followed by S.aureus,
E.coli, Streptococcal spp.
 Dog bites – may contain Pasturella spp. and should not be ignored

3002: When a person is bit by a possible rabid animal, there are really 4 avenues for Rabies post-exoposure
prophylaxis depending on the situation:
 High risk wild animal bite – includes bat, fox, raccoon, coyote, skunk
o If animal unavailable start PEP
o If animal available; euthanize it/test for rabies  PEP if positive
 Low risk animal bite – squirrell, chipmunk, mouse/rat, rabbit
o No PEP
 Pet – if a pet (usually a dog) DOES have rabies, they’ll show signs within 10 days
o If available for quarantine  observe 10 days  if shows signs of rabies, euthanize pet and do
PEP immediately
o If not available for quarantine  start PEP
 Livestock or unknown wild animal – call the health department
PEP for rabies consists of:
 Thorough cleansing of the wound (reduce risk by 90%)
 Administer rabies vaccine (if person hasn’t gotten it before) + passive immunization (human rabies Ig)

Case 44 – Stroke/TIA
Evaluation of Stroke Symptoms
 Stabilize/CT without contrast  shows hemorrhage immediately; if negative consider tPA
o Try to define onset of stroke symptoms to evaluate timeframe
o MRI and CTA may also be used to assess extent of brain damage
 Evaluate patient (ABCs, Define Neurologic Deficits, underlying conditions/medications)
Transient Ischemic Attack (TIA)
 Effectively stroke symptoms that occur and resolve within 1hr (classically 24hr)
 Indicates significant transient ischemia that has resolved (classically cocaine or drug use!)
 Should be treated as a stroke and stroke prophylaxis should be initiated
 Note that severe hypoglycemia may result in stroke-like presentation and should be treated first if
found, to see if stroke-like symptoms resolve with sugar resolution
 HTN is the most important risk factor for stroke onset

Brain Ischemia – 80% of strokes are ischemic!


Thrombosis: localized occlusive processes causing blood flow obstruction
 Atherosclerosis is most common; often occurring in the large vessels in the head/neck
 Primary hematologic problems (polycythemia, thrombocytosis, hypercoagulable states) may cause
platelet clotting within the vessels
 Vasoconstriction/fibromuscular dysplasia/arterial dissection can occur
 Hypertension (medial vascular hypertrophy) commonly affects penetrating intracranial arteries

Embolism: clot forms somewhere in the body and breaks off to lodge/occlude bloodflow somewhere else
 Stroke emboli most commonly come from the heart (atrial fibrillation!) or other major vessels
 Arterial dissection can result in clot formation with embolization
 Paradoxical emboli form in veins (usually DVT) and travel through a heart septal defect of A-V
malformation in the lungs to hit the brain
 Air, fat, cholesterol, bacteria (mycotic), foreign bodies, or placental material have a chance of
entering the vessels, acting as an embolus

Systemic Hypoperfusion: not enough blood pressure means not enough perfusion!
 Heart Failure (infarction or arrhythmia) or systemic hypotension (blood loss, hypovolemia,
overwhelming infection) are the two more common causes
 Watershed regions (regions last to receive blood supply) are the periphery of perfusion zones will be
greatly/diffusely affected

Stroke Syndromes
These are simply outlined on pg. 106 (BluePrints Neuro); name off the syndrome with these prompts!
Anterior Circulation Stroke Syndromes: carotid artery occlusion, MCA embolism, or hemorrhage in basal
ganglia
 Left cerebral hemisphere stroke (in book)
 Right cerebral hemisphere stroke (in book)
Posterior Circulation Stroke Syndromes
 Lateral Medullary Stroke (Wallenberg Syndrome; intracranial VA occlusion)
o Ipsilateral: increased or reduced facial pain/temp sense, Horner’s Syndrome
o Contralateral: reduced body pain/temp sense; arm discoordination
o Bilateral: nystagmus, ataxia; possible dysphagia/hoarseness if severe

 Bilateral Pontine Base/Medial Tegmentum Stroke (Basilar Artery)


o Quadriparesis | conjugate gaze paresis, intranuclear aphthalmoplegia, or CN VI nerve palsy
o Coma may occur if severe
 Cerebellar infarction (PICA or SCA)
o Gait ataxia; dysarthria; ipsilateral dysmetria
 Left PCA infarction
o Right homonymous hemianopia & Sometimes amnesia
o Alexia without agraphia if splenium of corpus callosum involved
 Right PCA infarction
o Left homonymous hemianopia
o Sometimes left-sided visual neglect
Lacunar Strokes: may be anterior or posterior circulation, but will affect penetrating arteries
 Pure Motor Stroke: contralateral face/arm/leg weakness with no sensory/behavioral losses
 Pure Sensory Stroke: contralateral face/arm/leg paresthesia without motor/visual/behavior losses
 Dysarthria-Clumsy hand syndrome: slurred speech and a clumsy hand
 Ataxic hemiparesis: weakness/ataxia of contralateral body (leg>arm)
Stroke in a Young Person
 Defined: basically it’s a stroke, but it’s odd that the person is so young, thus we look for other
etiologies
o Patent Foramen Ovale: can result in a paradoxical embolus, typically with risk factors that can
predispose to clotting (heart disease, diabetes, hyperlipidemia, etc.); best assessed with
transthoracic echocardiology
o Carotid Dissection: usually results in clot formation in the new open space with embolization.
Trauma, chiropractic manipulation, or predisposing diseases (Marfans, Ehlers-Danlos) clue you
in. Because the carotids run with the sympathetic ganglia, may cause an acquired Horner’s
syndrome
o Hypercoagulable disorder: anti-phospholipid syndrome, protein C/S deficiency, ATIII deficiency,
Factor V leiden mutation, hyperhomocystinemia, or prothrombin gene mutation
o Moyamoya disease: a classic disease of occlusion in the Circle of Willis resulting in collateral
vessel hypertrophy (moyamoya = puff of smoke); predisposes to rupture/stroke
o Arteriovenous malformation: can result in aneurysm and rupture, thus stroke
o Drugs of abuse: classically cocaine/amphetamines can cause vasospasm  stroke
 Tx: typically look for these more interesting etiologies and treat accordingly

Treating Acute Stroke…to give or not give tPA


Some major points when considering tPA thrombolytic therapy
 Timeframe: Pt is >4.5 hours away from symptom onset -> no tPA
 Hemorrhage/bleeding: hemorrhagic infarction, tendency to bleed/anti-coagulant meds, recent
surgery, thrombocytopenia, or recent bleeding episode -> no tPA
 Sugars: hypo/hyperglycemia may mimic signs/symptoms of stroke
 Blood pressure: lowering blood pressure causes less brain perfusion, worsening an ischemic stroke!
Don’t lower BP unless there’s signs of end-organ damage from HTN
After acute event, you must anti-coagulate!
 Heparin, Warfarin, Aspirin, or newer Factor Xa inhibitors (Dabigatran, etc.) can all be of use
 Anti-arrythmatics can be helpful if atrial fibrillation is present! A-fib can easily cause clots to be formed
and thrown, thus stopping the arrhythmia may aid in preventing future events
 Control risk factors (HTN, lipid abnormalities, obesity, smoking, drinking, etc.) with medication (aspirin,
anticoagulants, lipid/BP medications) and lifestyle changes (avoid fat, exercise, smoking cessation)

Case 45 – HIV & AIDS


10301: HIV screening is an assay detecting HIVp24 antigen and HIV antibodies recommended at:
 Initial screen if: Ages 15-65, being treated for TB, or being treated for any STI
 Annual screening if: IVDU (person AND sex partners), MSM, homeless, incarceration, possible sexual
contact with a HIV+ individual
 Additional screening if: new pregnancy, occupational exposure, prior to any new sexual relationship

3583: Acute, initial HIV infection begins as a mono-like disease and should be suspected in higher risk pts:
 Presentation: fever, lymphadenopathy, sore throat, arthralgias, generalized rash, diarrhea/N/V
 Dx: often must be made with high viral load detection (virus isn’t being attacked by immune system
yet) because the anti-HIV abs havn’t been formed. CD4+ count often normal.
 Tx: immediate HAART with partner notification/prophylaxis

2265: Remember! Spirochete visualization on dark-field microscopy is diagnostic for syphilis! Usually if a
person has syphilis, they’ve been engaging in some risky sexual behavior…thus they should be tested for other
STIs, especially HIV
3888: Basically, if your CD4+ count is >200 then you can receive any vaccine (simple!) but:
 All HIV pts should receive a pneumococcal conjugate (once) and polysaccharide (every 5 yrs) vaccines
 HIV pts who are sexually active with men should get HepA vaccine (once)
 If CD4+ count is <200  cannot receive live-attenuated vaccines
o MMR, varicella zoster, intranasal influenza, yellow fever
o Should a pt be diagnosed with HIV and be vaccinated with any of these before starting HAART,
they should be vaccinated again after HAART initiation

3252: Opportunistic infections and primary prophylaxis in HIV/AIDS


 P.jiroveci (PCP) – TMP-SMX (<200)
 T.gondii (toxoplasmosis) – TMP-SMX (<100 or positive IgG for toxo)
 Mycobacterium Avium complex – azithromycin/clarithromycin (<50)
 Histoplasmosis – itraconzole (<150 or living in endemic area)
 Herpes simplex virus – acyclovir/valacyclovir (recurrent infections, no CD4 count specified)
 Note that some major HIV related infections are NOT recommended to have primary prophylaxis
o Candida – Fluconazole: reactive treatment is extremely effective and prophylaxis is expensive.
Also concerns of resistance with prophylaxis, so we only treat reactively
o Cryptococcus – fluconazole: cost/concern for resistance are solid reasons, but also incidence is
extremely rare, thus prophylaxis is not administered

3251: If you’re exposed to HIV you immediately stop what you’re doing and initiate PEP for HIV
 Immediate prophylactic triple therapy (tenofovir, embricitabine, and raltegrevir preferred) for 28 days
 Serologic testing at immediately, 6 weeks, 3 months, 6 months
 High risk exposure: blood, CSF, genital secretions, needle-stick
 Low risk exposure: urine, feces, other bodily fluids

4115: If you see atypical pneumonia (diffuse interstitial infiltrate, tachypnea, tachycardia, fever, non-
productive cough) in an immunocompromised (immunosuppressive drugs, chemotherapy, or AIDS) then think
PCP pneumonia. If it’s typical pneumonia  S. pneumo. Don’t overthink it hauss.

2267: Remember! S.pneumoniae is THE most common cause of pneumonia in HIV/AIDS patients. You might
be temped to think PCP when an AIDS patient has pneumonia, but you better slow your roll:
 S.pneumoniae – unilateral, lobar infiltrate, productive cough, pleural effusions often occur, >200 CD4
o Remember! This guy is encapsulated, thus even people with normal immunity have a harder
time clearing this infection!
 P.jiroveci – bilateral, diffuse infiltrate, dry cough, pleural effusions rarely occur, <200 CD4

3938: PCP pneumonia is one of the most common infections in AIDS patients
 Sym: dry cough, fever, exertional dyspnea (out of proportion to CXR findings)
 Signs: CXR (bilateral interstitial infiltrates), elevated lactate dehydrogenase
 Dx: bronchioalveolar lavage demonstrating P.jiroveci
 Tx: TMP-SMX (Bactrim) for 21 days + corticosteroids (PaO2 <70)
o IV pentamidine is an alternative for those that cannot take TMP-SMX but it has a lot of side
effects (damage to kidney, liver, heart, etc.)

2273/2267: PCP pneumonia (P.jiroveci formerly carinii)


 One of the most common infections for an AIDS patient
 Presentation: dry cough, dyspnea, hypoxia, CXR with bilateral diffuse infiltrate, rarely pleural effusion
 Dx: clinical suspicion + bronchoalveolar lavage showing P.jiroveci (little crushed ping-pong balls!)
 Tx: TMP-SMX + corticosteroids (if poor O2 saturation status; PaO2 <70 or A-a >35)  may show initial
worsening of lung function due to death of bacteria and inflammatory reactions in lung

2304: Detecting PCP you can do bronchioalveolar lavage (high sensitivity/spec) or sputum culture (high
specificity but low sensitivity)

2269: Profuse watery diarrhea in HIV/AIDS patients is typically from a opportunistic infection (bloody would
be more high infectious things).
 First step in diagnosing what is causing the diarrhea is stool examination for ova/parasites as often
parasitic infections will be the cause here!

3917/3590: Diarrhea in HIV/AIDS typically occurs with different etiologies based on CD4+ count
 Cryptosporidium (<180) – severe watery diarrhea, wt loss, fever
o Classically from animal contact, water, or person-person
o Dx: modified acid-fast stain shows cryptosporidial oocytes (4-6mcg)
o Tx: anti-retroviral therapy (resolves with CD4 receovery)
 Microsporidium or isosporidium (<100) – watery diarrhea, wt loss, cramps, no fever
 Mycobacterium Avium Complex (<50) – watery diarrhea, wt loss, high fever (>102F)
 CMV (<50) – bloody/small volume diarrhea, abdominal pain, wt loss, fever
o Dx: colonoscopy with biopsy – intracytoplasmic eosinophilic/basophilic inclusions
o Tx: ganciclovir
o Must examine eyes to rule out CMV retinitis

2277: Progressive multifocal leukoencephalopathy (PML)


 Progressive demyelinating disease caused by [JC virus infection + immunocompromised]
 Presentation: any immunocompromised patient getting new onset of focal neurologic symptoms
should immediately be suspected to have PML
 Tx: MRI showing multiple, non-enhancing white matter lesions, typically without mass effect
 Dx: no known treatment, progressively fatal

3253/11108: Kaposi Sarcoma (HHV-8 + AIDS)


 Presentation: multiple skin lesions often starting brownish then becoming purple/red due to increased
vascularization of infected tissue; may accompany other signs of AIDS
o Note that lesions may be quite impressive if infection has been present for prolonged time (see
question 11108 for example)
 Dx: clinical but biopsy and HIV/AIDS serologic evidence often used to confirm
 Tx: HAART treatment of AIDS will often cause regression with CD4+ recovery
o If refractory, chemotherapy may be used to kill off the rapidly dividing tissue

2274/8959: HIV esophagitis occurs with CD4+ count <100; occurs with painful swallowing/substernal burning
 Candida albicans – white plaques in mouth/esophagus (oral thrush!)
o Tx: 3-5 days of oral fluconazole with HAART assessment
o If patient fails this, further workup for viral causes
 HSV – herpetic vesicles with round/ovoid ulcers in esophagus, sometimes w/ HSV infection elsewhere
o Severe odynophagia (painful swallow) WITHOUT dysphagia
o Dx: biopsy with histopathology
 CMV – Deep, linear ulcers in the distal esophagus
o Severe odynophagia (painful swallow WITHOUT dysphagia
o Dx: biopsy with histopathology
 Aphthous ulcers – idiopathic presenting with apthous ulcers
Pill esophagitis can cause some similar symptoms but DOESN’T typically occur in HAART regimens

3103: CMV esophagitis in AIDS pts causes substernal burning/long-linear ulcers, and biopsy shows
intranuclear/intracytoplasmic inclusions (Tx: IV ganciclovir)

2268: Disseminated Mycobacterium Avium Complex (M. avium or M. intracellulare): non-specific symptoms
(fever, cough, abdominal pain, diarrhea, night sweats, weight loss), splenomegaly, and elevated alkaline
phosphatase.
 Typically, in HIV pts with CD4+ <50; pts with this low of CD4 should be given azithromycin prophylaxis
 Dx: blood culture or lymph node/bone marrow biopsy
 Tx: clarithromycin or azithromycin

2295: TB re-activation is common in AIDS patients. The infection is kept latent in the lungs in a granuloma
(macrophages make up much of this), so when the immune system craps out…these do too, releasing the
mycobacteria.
 Sign/sym: subacute mild cough, low-grade fever, fatigue prominent in the morning (due to secretion
accumulation)
o CXR: superior lobe consolidation (higher O2 levels) with cavitary lesions
o Substance abuse is a huge risk factor for TB active infection
 Remember that PCP is a common AIDS pneumonia, but cavitary lesions are uncommon

3246: TB infections (new or re-activated) are VERY common in HIV/AIDS patients. In fact, any AIDS patient
having a PPD test show >5mm induration should be treated prophylactically immediately.
 Tx: Isoniazid (anti-TB) + pyridoxine (B6 to fight off deficiency) for 9 months

4388: Infective endocarditis is typically due to S.aureus infection and there’s an even higher chance with
concurrent HIV/AIDS.
 Typically, right sided (tricuspid) endocarditis has differences than the typical mitral valve version:
o Murmur less likely (lower pressure flow across the tricuspid valve)
o Less systemic signs with more prominent lung problems (as septic emboli get in the lungs)
o Chest CT can show abscesses, infarction, pulmonary gangrene, cavities at lung periphery

3254/9885: Cryptococcal Meningoencephalitis (Cryptococcus neoformans fungus)


 Classically occurs in pts with immunosuppression (AIDS is classic)
 Presentation: fever, lethargy, headache, altered mental status  may progress to coma
o CSF: high opening pressure, low WBCs, low glucose, elevated protein
o Papilledema (indicating increased CNS pressure) can be an important sign
 Dx: Latex agglutination (+), cryptococcal antigen (+), India Ink (+), Sabourard Agar growth
 Tx: IV amphotericin B + flucytosine (2 weeks, induction)  oral fluconazole (8 weeks)
o Intrathecal amphotericin B if failure to respond to IV treatment
o Serial lumbar punctures to limit elevated CSF pressure

3107: Bacillary angiomatosis (B.hensliae)


 Classically due to a cat scratch
 Presentation: immunosuppressed (AIDS pts most common) with bright red, firm, friable, exophytic
(extends beyond the epithelium) nodules
 Tx: oral erythromycin

2264: Treatment of toxoplamosis


 Prophylaxis is TMP-SMX (Bactrim) often for AIDS or immunosuppressed pts
 Tx: empiric sulfadiazine + pyrimethamine if the infection is actually present; diagnosis may be made if
ring-enhancing lesions are seen on brain imaging with improvement of symptoms with this treatment
regimen!

2839: Serous Otitis Media


 Middle ear effusion resulting in conductive hearing loss without infectious symptoms. Ear canal shows
dull tympanic membrane with hypomobility on pneumatic otoscopy
 The most common middle ear pathology in HIV/AIDS due to lymphadenopathy obstructing the
Eustachian tube

Case 46 – Jaundice
2627/2935: Hyperbilirubinemia
 Unconjugated more elevated: hemolysis (overproduction), reduced uptake into liver (portosystemic
shunt or drug effect), or conjugation defect (Gilbert’s syndrome or Crigler-Najar)
 Conjugated more elevated:
o AST/ALT elevated: liver disease/damage (hemochromatosis, hepatitis, cirrhosis, etc) because
hepatocytes are damaged and releasing bilirubin into blood
o Normal AST/ALT/Alk.phos: bilirubin metabolism disorder (Dubin-Johnson or Rotor syndromes)
bilirubin is built up and leaks into blood
o Alk.phos elevated: intrahepatic cholestasis or biliary obstruction (gallstones, malignant
obstruction, intrahepatic destruction, etc.) bilirubin is backed up and leaks into blood

2977: When assessing jaundice, positive urine bilirubin means conjugated (direct) bilirubinemia
 This makes sense, as only conjugated bilirubin is made to be water soluble, unconjugated (indirect)
bilirubin is inherently not water soluble, thus it cannot be filtered by the kidney into the urine.
 However, a positive urine urobilinogen indicates unconjugated (indirect) bilirubinemia, as the huge
amount of unconjugated bilirubin goes through it’s normal metabolism in the large intestine, a large
amount of urobilinogen (a normal by-product) will be made. This is NOT the case in a direct bilirubin,
as a direct bilirubinemia is caused by some obstructive process, and it would never make it to the large
intestine to become urobilinogen!

2975: When presented with a jaundice/direct bilirubinemia, the FIRST thing to do is assess the bile duct
width with RUQ ultrasonography. A dilated biliary tree >3cm suggests extrahepatic obstructive process. A
non-dilated biliary tree (<3cm) suggests an intrahepatic obstructive process.
 ERCP is often performed when dilation >3cm is found as you must figure out exactly where the
obstruction is occurring to uncover etiology

2900: Acute Liver Failure


 Presentation: nonspecific symptoms (fatigue, anorexia, nausea, etc.), RUQ pain, jaundice, pruritus, and
sometimes hepatic encephalopathy as failure worsens. Labs include:
o ^PT/INR (failure to liver to make coagulation proteins)
o AST/ALT x10 normal with increased bilirubin
o Progression may show AST/ALT trending down (liver death is bad that these are being released
less and less) and increasing PT/INR/bilirubin (further loss of liver function; PT is the single
most prognostic factor in Acute Liver Failure)
 Dx: clinical presentation with lab values; must search for underlying cause
 Tx: stabilize pt and treat underlying cause once identified

4635/2936: Chronic Liver Disease (Cirrhosis)


 Presentation: Jaundice, itchiness, GI bleeding, encephalopathy, decrease in total T3/T4 (free
concentration only drops in severe cases) and gender specific signs due to poor metabolism of
estrogens in liver (increased estrogen in blood) and systemic signs
o Men: hypogonadism (small/firm testes, decreased libido, erectile dysfunction, loss of pubertal
hair, gynecomastia)
o Women: amenorrhea, irregular menses
o Systemic: telangiectasias, portal HTN, ascites, hepato-splenomegaly, palmar erythema, “terry
nails”, Dupuytren’s contracture, clubbing of fingernails
o May be totally asymptomatic
 Dx: clinical picture; may be confirmed with liver biopsy
o Endoscopy to assess for varices as these are treatable complications of liver disease. Two other
complications (hepatic encephalopathy/acites) will be obvious from clinical exam
 Tx: stop damaging behavior/disease; some cirrhotic damage cannot be undone
o Treatment to prevent complications (B-blocker for varicies, draining of ascities, etc.) is a major
part of management alongside treatment of current complications
o Cirrhosis in patients without signs/symptoms (“compensated cirrhosis”) should be monitored
for hepatocellular carcinoma (ultrasound liver/a-fetoprotein screen every 6 months)

2911/4278: Alcoholic Hepatitis


 Presentation: signs of hepatitis (jaundice, RUQ pain, ascites, etc.) and sometimes cirrhosis with history
of alcohol abuse and characteristic labs
o Labs: elevated AST:ALT (2:1, but <300), elevated GGT/bilirubin/INR/ferritin (acute phase
reactant), neutrophil leukocytosis
o Note that AST:ALT ratio is thought to be due to alcohol depletion of pyridoxyl-5-phosphate, a
cofactor needed for ALT
o May have elevated MCV due to chronic malnutrition (folate or B12 depletion)
 Dx: history + clinical presentation + labs
o Imaging/biopsy will show fatty liver
 Tx: abstinence/supportive care

4648: Ischemic Hepatic Injury (“Shock Liver”)


 Presentation: any form of shock (septic shock is most common) with rapidly developing AST/ALT
elevations (like in the 1000-2000 range!) and normal total bilirubin/alk.phos
 Dx: clinical syndrome
o Acute HepA/HepB will quickly elevate AST/ALT/Bili/Alk.phos
o Cholecystitis/Hepatobiliary obstruction will elevate Bili/Alk.phos more than ALT/AST
o Other hepatic diseases may raise these levels, but not very high!
 Tx: treat the underlying cause of shock and typically liver enzymes will recover as well!

Hepatitis A (HAV)
 RNA picornavirus; fecal-oral (enteric) transmission
 Acquired by: children, travelers to endemic areas, may be in community outbreaks
o Symptoms occur over a period of weeks (children = asymptomatic; adults = symptoms)
o Shedding of the virus in feces occurs before symptoms begin!
 Causes: acute hepatitis (often community, asymptomatic)
 Testing:
o Anti-HAV IgM antibody = used to detect presence of virus/recent infection
o Anti-HAV IgG antibody = used to detect previous infection/protects from new infections
 Vaccine:
o Havrix: inactived vaccine for children (2 y/o)
o Twinrix: HAV + HBV vaccine for teens (18 y/o)
o Human reservoir/single serotype (good chance for eradication); but it’s not serious disease so
eradication is not a huge priority
 Hand-washing is the BEST defense against this virus!

Hepatitis B (HBV)
 DNA hepadnavirus; parenteral transmission; “Dane” particles may contribute to chronicity
 Acquired through: childbirth/unprotected sex (vaginal secretion, semen), IV drug use/needle stick
(blood); living in the same house as someone infected with HepB
 Causes: acute hepatitis, chronic hepatitis (20%), fulminant liver failure, increased HCC risk
o Higher risk of chronicity with younger age (<1yr: 90% | >5yr: 2%)
o Virus course over a year if chronic; virus shedding occurs BEFORE symptoms
 Replication:
o Virus enters the nucleus with partial dsDNA, which is “fixed” into DNA in the nucleus
o Host nuclear machinery transcribes the HepB DNA into an (+)mRNA intermediate
o (+)mRNA intermediate moves into cytoplasm
 Some mRNA is transcribed by host ribosomes into viral proteins
 Some mRNA is enveloped in these proteins to become the new progeny
o Viral reverse transcriptase converts (+)mRNA -> partial dsDNA within the progeny virus
o New HepB is released out of the cell
o “DNA virus using an RNA intermediate to generate DNA”
 Testing:
o HBsAg = HBV surface antigen used to detect HBV infection
 Detected during acute infection and chronic infection
o HBcAg = HBV core antigen
 Not used in detection of disease
o HBeAg = HBV core antigen detected during active viral replication (high transmissibility)
 Detected during acute infection or chronic-active infection
o Anti-HBs = antibody to HBsAg
 Can detect after disease resolution indicating recovered infection or immunization
 Vaccine is ONLY surface antigen
o Anti-HBc = antibody to HBcAg
 IgM  acute infection or “window period”
 IgG  chronic infection or recovered infection
o Anti-HBe = antibody to HBeAg (low transmissibility)
 Detected during the “window period”, chronic infection, or recovered infection
 Epidemiology: sub-Saharan Africa/China (early life infections)
 Immunization  patients born before 1988 probably havn’t gotten vaccinated unless they’re in a
healthcare field or from University. Anyone after that should have gotten it as a baby or teenager

Hepatitis C (HCV)
 RNA flavivirus; does not replicate in the nucleus; parenteral transmission
o Generates one big protein and then sues proteases to cleave it into functional units
o Proteases are a huge target for HCV drug treatment
o E2 envelope protein  hyper-mutating 30 amino-acid stretch that is often targeted by the
immune system. If it mutates then the virus will become a “quasi species” and it’s less
recognizable to our immune defenses
 6 subtypes (USA commonly 1; IVDU commonly 3); each has a different drug regimen
 Acquired through: IV drug abuse, unprotected sex, before 1992 blood transfusion (now rare)
 Causes: mild acute hepatitis, chronic hepatitis (80%), increased HCC risk
 Testing:
o HCV-RNA test (EIA screening confirmed with PCR) indicates infection presence
o Decreasing RNA levels = resolution of disease
o Persistent RNA levels = chronic disease (quasi-species causing chronic infections)
 Alcohol use should be discouraged completely; Tylenol is safe if less than 2g is taken
 HEP C IS CURABLE

2958/2232: Cryoglobulinemia secondary to Hepatitis C infection


 Occurs via IgM immune complex deposition causing inflammation and damage in areas of
deposit/vascularture
 Presentation: muscle/joints (arthralgias), skin (palpable purpura, Raynaud’s), liver (elevated
transaminases, ALT>AST), kidney (moderate-to-severe glomerulonephritis), and PNS (motor-sensory
axonopathy)
o Labs: decreased compliment levels, cryoglobulins, serologic evidence of HepC
o Hx may suggest Hep C exposure (IVDU is classic)
 Dx: clinical presentation with agreeing lbs
 Tx: plasmapheresis (remove cryoglobulins) and cyclophosphamide/corticosteroids (immunosupp.)

4654: Chronic Hepatitis C and Associated Conditions


 Presentation: asymptomatic or fatigue are the most common; nonspecific symptoms (fever, anorexia,
myalgia, weight loss, etc.) can also occur
o Labs: elevated or normal serum transaminases
o May progress to liver cirrhosis (20%) or hepatocellular carcinoma
 Associated diseases can clue you into the HepC infection!
o Essential mixed cryoglobulinemia (90% of cases have HepC) – palpable purpura, arthralgias,
renal complications due to IgM immune complex deposition into medium sized vessels
 Low compliment levels are classic for this disease
o Membranoproliferative glomerulonephritis – will appear as renal decline with characteristic
biopsy histology of the glomeruli
o Porphyria cutanea tarda – fragile skin, photosensitivity, vesicular rash/erosions on dorsum of
the hands
o Lichen planus – itchy, purple, raised, well demarcated plaques

Hepatitis D (HDV)
 RNA delta-virus; incomplete/cannot function on it’s own
 Requires HBV infection for infectivity; uses HBV surface antigen to infect cells
 Encodes delta antigen, which are capsid proteins; uses host-RNA polymerase
 Causes: chronic hepatitis co-infection (less severe), super-infection (severe/progressive), increased HCC
risk
o Chronically infected pt with a sudden spike in ALT  superinfection!

Hepatitis E (HEV)  25 yo Afghani pregnant woman becomes nauseated, vomiting, elevated liver enzymes,
dies 7 days later. Likely HepE (classic association); but not Hep A, as she was likely exposed in early life
because of the poor sanitation that happens in a war-torn country
 RNA calicivirus (non-enveloped ssRNA); fecal-oral (enteric) transmission (Zoonotic illness in the USA)
 Causes: acute hepatitis, fulminant hepatitis (<1%) in pregnant women, 3rd trimester (high fatality)
 Testing
o Anti-HEV IgM antibody = used to detect presence of virus/recent infection
o Anti-HEV IgG antibody = used to detect previous infection/protects from new infections

2924/2923: Genetic Diseases of Liver Metabolism


 Dubin-Johnson syndrome – conjugated bilirubinemia; jaundice with body stressors (illness, pregnancy,
OCP use) but otherwise no/minor symptoms (fatigue, abd. pain, weakness) and no hemolysis
o More common in Sephardic Jews
o Clinically normal aside from reactionary jaundice and black liver (epinephrine metabolites)
o Labs: Bilirubin (20-25 mg/dL), normal LFTs, normal coproporphyrin (predominantly Copro I)
 Rotor syndrome – Dubin-Johnson syndrome without the black liver
 Crigler-Najar syndrome – autosomal recessive unconjugated bilirubinemia;
o Type 1: significant mental retardation/death; Phototherapy/plasmapheresis can help
conjugate the bilirubin to buy time for curative liver transplant
o Type 2: fairly benign unconjugated bilirubinemia with jaundice; often asymptomatic, but
symptoms (if occuring) can be treated with phenobarbital or clofibrate
 Gilbert syndrome – mild unconjugated bilirubinemia triggered by bodily stressors; very similar to C-N
syndrome II, difference is in the enzyme defect

Malignant Obstruction of the Hepatobiliary Tree


 A classic reason to get bilirubinemia, typically due to pancreatic adenocarcinoma/cholangiocarinoma
 Presentation: conjugated hyperbilirubinemia with predominantly elevated Alk.phos, painless jaundice,
and fatigue/weakness/weight loss
 Dx: Abdominal ultrasound or CT; retrograde CPG endoscopy if imaging is negative
 Tx: Resection or chemo/radiation therapy

2209: Pancreatic Cancer


 Can occur in the head or the body/tail of the pancreas causing different symptoms!
 Head: painless jaundice, “double duct” sign, Courvorsier sign (non-tender, distended gallbladder at
right costal margin), jaundice/pruritis/pale stools/dark urine
 Body/Tail: abdominal pain WITHOUT jaundice/obstruction

3585: Diagnosis of Pancreatic Adenocarcinoma


 Common symptoms: Weight loss, gnawing abdominal pain that radiates to the back, worsens with
eating, laying down, or at night
 Pancreatic Head symptoms: jaundice and steatorrhea (obstruction of hepatobiliary secretion)
o Abdominal ultrasound is the preferred technique for diagnosis
 Pancreatic Body/Tail symptoms: NO JAUNDICE
o Abdominal CT is preferred technique for diagnosis

2978: Acute Cholangitis


 Biliary stasis due to any number of reasons (commonly gallstone obstruction, malignancy, bile duct
stenosis) but all of them cause backup. The fear is that disruption of the blood-bile barrier could allow
for dissemination of bacteria/toxins
 Presentation: [fever, jaundice, RUQ pain], [confusion, hypotension] (Charcot triad, Reynold’s pentad)
o Labs: neutrophilia, elevated alk.phos/GGT/AST/ALT/direct bilirubin
 Dx: presentation/labs with biliary dilatation (ultrasound FIRST, then CT if not diagnostic)
 Tx: [ERCP with sphincterotomy or percutaneous transhepatic cholangiography] +[ broad spectrum
antibiotics]

2898/2950: Primary Biliary Cholangitis


 Chronic liver disease characterized by intrahepatic biliary duct destruction by anti-mitochondial
antibodies resulting in cholestasis
 Presentation: fatigue, pruritus, jaundice, hepatomegaly, steatorrhea, portal HTN, xanthalasma, and
metabolic bone disease
o Labs: elevated Alk.phos, hyperlipidemia, normal AST/ALT
o Classically in middle-aged women and associated with other autoimmune diseases
 Dx: ultrasound to confirm intrahepatic cholestasis  anti-mitochondrial antibodies are diagnostic
o Liver biopsy would show ductopenia
 Tx: Ursodeoxycholic acid (delays histologic progression via increased bile transit and possible anti-
inflammatory effects
o Liver transplant is the ultimate treatment

4331/3609/4347: G6PD deficiency (glucose-6-phosphate dehydrogenase)


 X-linked recessive disorder which can result in acute anemic crises in response to certain situations.
G6PD typically used to generate glutathione to minimize oxidation damage to RBCs. Should a
oxidizing stressor come on, G6PD deficiency pts are not able to compensate.
o Classically triggered by infection, oxidant drugs (anti-malarials, sulfa drugs, etc.), and fava bean
 Presentation: acute hemolytic crisis (fever, jaundice, abdominal pain, dark urine) due to hemoglobin
denaturation and disruption/lysis of RBCs.
o Labs: normal G6PD levels are typical (as the deficient cells were destroyed and new
reticulocytes have normal G6PD levels)  re-test in 3 months after crisis
o Smear: degmocytes (“bite cells”) and Heinz bodies (denatured basophilic hemoglobin)
o Urine may stain with Prussian Blue (+ is presence for iron, thus hemolysis is occuring
 Dx: clinical scenario/blood smear
 Tx: supportive care during crisis with avoidance of precipitating substances

3062/4333: Hereditary Spherocytosis


 Autosomal dominant defect in spectrin or ankyrin RBC structural proteins resulting in RBC membrane
blebbing and eventually deformation into fragile spheres (instead of bi-concave disks)
 Presentation: hemolytic anemia, jaundice, splenomegaly classically in European-descent folks
o Labs: anemia, inc. MCHC/dec. MCV (small RBCs)
o Smear: abundant spherocytes with some shearing of RBCs
o Complications: aplastic anemia in Parvovirus B19 infection OR pigmented gallstones (increased
hemolysis causing increased bilirubin), overwhelming sepsis from encapsulated organisms if
spleen has been removed.
 Dx: Osmotic fragility test (increased fragility), acidified glycerol lysis test (increased lysis), or abnormal
eosin-5-maleimide binding test, with negative Coomb’s test and blood smear
 Tx: splenectomy (stops RBC destruction), folate supplementation, blood transfusions
o Anti-pneumococcus, Haemophilius, and meningococcus vaccines before splenectomy and daily
oral penicillin for 3-5yrs after surgery due to risk of sepsis post-splenectomy, which apparently
lasts for >30yrs even though our prophylaxis only goes on for about 5 years. [HGH YIELD]

4437/2248: Autoimmune Hemolytic Anemia (AIHA)


 Warm AIHA
o Hemolysis triggered by a stressor (abx, infection, etc.) causing IgG/Anti-C3 antibody formation
o Presentation: normocytic anemia, hemolysis (jaundice, indirect bilirubinemia, inc. serum lactate
dehydrogenase, decreased serum haptoglobin), splenomegaly
 Smear: reticulocytes, spherocytes, elliptocytes, and microspherocytes
 Concern of venous thromboembolic disease
o Dx: direct Coombs test demonstrating specific antibodies bound to RBCs
o Tx: high-dose corticosteroids or splenectomy (if refractory)
 Cold AIHA
o Hemolysis triggered by infection (HCV, EBV mono, or Mycoplasma) or in lymphoproliferative
disorders causing IgM/Anti-C3 antibody formation
o Presentation: Warm AIHA symptoms + livedo reticularis/acral cyanosis with cold exposure that
disappear with warming
 Concern of gangrene from cold agglutinin aggregates
o Dx: direct Coombs test
o Tx: avoid cold temps + rituximab +/- fludaribine

Medication induced – acetaminophen and alcohol (>3 drinks) can lead to toxic NAPQI accumulation
 Herbals like Kava, Ma Huang, etc can all cause liver damage, thus should be investigated
 Milk thistle might aid in helping resolve with alcoholic cirrhosis, but likely does nothing

Case 47 – Dyspepsia/Peptic Ulcer Disease (PUD)


Helicobacter pylori
• Slender/curved gram(-) rods with polar flagella
• Associated with:
• Gastritis
• Peptic ulcer disease (95% duodenal, 75% gastric)
• Gastric adenocarcinoma and lymphoma
• Microaerophile (requires O2, but only needs a low level of it)
• Virulence factors:
• Vaculolating cytotoxin (VacA) – toxic to cells, not well studied
• Cytotoxin Associated gene A (CagA) - PAI encoding Type III secretion system allowing for
infection of VacA toxin into cells
• Urease – hydrolyses urea to make ammonia and CO2 à neutralizes stomach acid for protection
• Diagnosis
• Endoscopy with biopsy/culture – can be useful, although invasive. The bacterium will sit on top
of the mucosa in little clumps in the gut lumen
• Stool antigen – enzyme immunoassay used to detect antigen in the infected patient’s stool.
Fairly inexpensive and non-invasive.
• Urea breath test – qualitative test using 14C- and 13C-labelled urea can be given orally and when
degraded by the bacterial urease, labelled CO2 will equilibrate with body fluids and ultimately
be expired from the lungs into the breath. Can be used to prove absence of bacterium and
follow treatment! Somewhat expensive but non-invasive.
• Serology – useful for confirming an infection at some point by the bacterium. Titers stay high
for years, even with eradicated infection, so it’s not as useful as you may think
• Tx
• Triple therapy – Amoxicillin (mitronidazole if allergic to penicillin) +Clarithromycin + PPI
• Quadruple therapy – PPI + Bismuth + Metronidazole + Tetracycline (Pylori Being Mean? Use
tetra [4])

NSAIDs – non-specific COX inhibitors can screw up the mucus barrier in the stomach leading to ulcers

Gastroesophageal Reflux Disease (GERD)


 Reflux of acid from stomach into esophagus due to decreased Lower Esophageal Sphincter (LES) tone
o Often heartburn/regurgitation upon lying down – may mimic cardiac chest pain
o Nocturnal cough/dyspnea or adult-onset asthma – airway irritation from HCl moving up the
esophagus/into common space and down into the airway
o Enamel erosion of the teeth – HCl getting into mouth
o Ulceration/stricture – total destruction of mucosa knocking out stem cells  healing with
fibrosis resulting in narrowing of lumen
o Barrett’s esophagus – discussed below
 Risk factors
o Alcohol/tobacco use, fatty diet, obesity, caffeine
o Hiatal hernia (“sliding hernia”) – herniation of gastric cardia above the diaphragm = HCl
production above the diaphragm means easier entry of HCl into the esophagus (often more
severe GERD)
o Hour-glass sign – LES is pushed higher into thoracic cavity/retains some tone; so you have
esophagusLEScardia above the diaphragm
Clinical Points
 Often gastroenterologist referral prudent when ‘alarm feature’ are present (changes in chronic
disease, unresponsive symptoms, dysphagia, vomiting, cancer concerns, anemia, etc.)
 Endoscopy may be useful for diagnosing GERD (good specificity, poor sensitivity) and often findings
will clinic diagnosis (lack of findings doesn’t tell you much)

2200: Endoscopy’s role in Evaluation of GERD


 Basically a person with GERD isn’t too much of a worry as long as they’re doing OK. However there are
certain situations where further investigation is required
 Let’s say you have a patient with GERD, if:
o Male >50, symptoms >5yrs, cancer risk (smoking, alcoholism, etc.), alarm symptoms present
(dysphagia, odynophagia, weight loss, anemia, GI bleeding, recurrent vomiting)  endoscopy
o They don’t have any of the above characteristics/symptoms  PPI trial 2 months
 If symptoms are well controlled  stay on that PPI
 If symptoms are refractory  consider switching PPI or increasing dose
 If symptoms STILL refractory  endoscopy

4335: Asthma exacerbation due to GERD


 Presentation: recent worsening of asthma symptoms with normal lung exam. Often sore throat,
morning hoarseness, worsening cough when laying down (at night is common), need for inhaler
following meals, dysphagia, chest pain/heartburn, regurgitation sensation
o Result of microaspirations causing irritation and vagal hypersensitivity resulting in proclivity to
bronchospasm
o Often coincides with weight gain (possibly due to a stomach ulcer!)
 Tx: PPI trial

4346: Note that chronic GERD patients may have chronic cough (stomach HCl into lungs) or hoarseness (HCl
into the larynx) as part of their clinical picture.

11126: Remember how we made fun of people for misdiagnosing heart attacks for GERD? Well that shit is real
because GERD can sound a whole lot like coronary artery disease! A patient with GERD might come
complaining of chest pain with radiation!! Note the characteristic GERD signs:
 Prolonged pain >1hr, post-prandial symptoms, heartburn/dysphagia, relief with anti-reflux drugs
 Often cardiovascular workup is necessary in these patients, but will remain unremarkable

3588: Dyspepsia
 Uncomfortable feeling of fullness or discomfort after eating; more of a symptom than a disease
 Three major routes to consider based on pt presentation:
o Typical GERD symptoms: PPI or H2 blocker
o NSAID/COX-2 inh. use: stop using the NSAID/COX-2 inh.
o Neither of the two above:
 Alarm symptoms (weight loss, GI bleed, anemia, dysphagia, vomiting, early satiety):
upper/lower GI endoscopy
 No alarm symptoms:
 Age >55: Scope
 Age <55:
o H.Pylori testing (Breath test or stool assay)
 Treat if positive; PPI trial if negative
 If all of that fails, then have the patient undergo endoscopy

Peptic Ulcer Disease


 Presentation: epigastric abdominal pain that changes (improve or worsen) with eating, often with
symptoms appearing at night spontaneously. Onset gradual over weeks-months
 Stool occult blood/anemia workup should be done to ensure no active bleeding ulcers
 Testing for H.pylori with PPI/triple or quad therapy trial should be undertaken. Endoscopy should only
be sought if these do NOT resolve symptoms or alarm symptoms are present

3178: Perforated Peptic Ulcer Disease


 Presentation: chronic epigastric pain that suddenly worsens, with rebound tenderness/guarding
(peritonitis due to gastric contents in peritoneal cavity), air under the diaphragm on CXR
 Dx: immediate CXR, as diagnosis and prompt treatment (<12hr) is critical to limit mortality
 Tx: immediate surgical repair

2596: MALT lymphoma (MALToma)


 Gastric cancer often the result of H.pylori infection; looks like a heaped up ulcer
 Dx: endoscopy with biopsy showing MALT-lymphoma
 Tx:
o Eradication of H.pylori ([omeprazole, clarithromycin, amoxicillin] or [bismuth, omeprazole, a
tetracycline, an aminoglycoside]
o Chemotherapy ([CHOP] or [CHOP + bleomycin]) can still play a role should H.pylori eradication
fail to cure the cancer

3591: Gastrinoma (Zollinger-Ellison Syndrome)


 Gastrin secreting tumor  overwhelming stimulation of parietal cells to secrete HCl/intrinsic factor
 Presentation: dyspepsia, GERD, abdominal pain, weight loss, diarrhea, frank GI bleeding
o Endoscopy: thickened gastric folds, multiple gastric ulcers, duodenal/jejunal ulcers which are
refractory to PPI use
o Often sporadic (ages 20-50) but can be associated with MEN-1 syndrome
o May result in steatosis due to acidic inactivation of pancreatic enzymes
 Dx: if characteristic findings clinically/on upper endoscopy follow this testing algorithm
o Serum gastin level: check 1 week after stopping PPI
 <110 pg/mL = not gastrinoma
 110-1000 pg/mL = indeterminate, further studies are needed
 >1000 pg/mL = gastrinoma , confirm with stomach pH testing
o Secretin secretion test: a test for indeterminate levels of gastrin; secretin inhibits gastrin
producing G-cells, thus if gastrin levels drop, you don’t have a gastrinoma. Gastinomas are
stimulated by secretin, thus a rise means you have gastinoma.
 Calcium infusion study: if a secretin test is negative but clinical suspicion is still high, the
calcium infusion that illicits a rise in serum gastrin points to gastrinoma
 Chromogranin A: a neuroendocrine tumor marker that’s elevated in many situations but
can be elevated in gastrinoma and helpful if clinical suspicion is high
o Stomach pH testing: important confirmatory test with a diagnostic gastrin level. Gastrin may be
physiologically elevated due to achlorhydria (failure of HCl secretion).
 pH< 4 = gastrinoma
 ph > 4 = not gastrinoma
 Tx: surgical resection

Case 48 – Fever and Rash in a Child

Case 49 – Diseases of the Breast


[Chapter 33: Disorders of The Breast – Topic 40]
Anatomy
 Breasts are essentially modified sebaceous glands that have gained the ability to produce milk.
Younger women have a high amount of glandular/ductal tissue and as life progresses, the tissue is
slowly replaced by fat, a process accelerated by menopause
 The largest amounts glandular tissue is in the upper, outer quadrant of the breast; this also explains
why the majority of breast cancers first appear in this region
 The breast has a rich blood supply and lymphatic system, which is great for support, but poses
problems when cancer arises. The ipsilateral lymph nodes and internal mammary lymph nodes are
the most common sites of metastases.
 Breast tissue development is primarily the result of sex hormones:
o Estrogen – stimulates growth of adipose tissue/lactiferous ducts
o Progesterone – stimulates lobular growth and alveolar budding
Evaluating Breast Signs/Symptoms
 Breast Cancer Risk Factors – increasing age, Hx of breast cancer/atypical hyperplasia, BRCA1 or BRCA2
mutation, high density breast tissue, increased estrogen exposure (early menarche, late menopause,
nulliparity, no breast feeding), long-term OCP use, alcohol consumption, tall height, high SES, and
Ashkenazi Jewish ancestry
 History/Physical – history is always important. The breast exam is best done during the follicular
phase of the menstrual cycle as breast tissue will be maximally stimulated by hormonal changes
 Diagnostic Testing
o Mammography – basically an X-ray of the breast. It’s a standard screening procedure for
women age >50. Useful as it can detect lesions approx. 2yr before palpable. Also note that
lobular carcinoma is the hardest to detect. If a mass is found, scan the other breast as well as
the lymph nodes to search for other abnormalities.
o Ultrasonography – the most common modality to evaluate a breast mass in women <40yrs
(esp. adolescents / those with dense breast tissue)
o MRI – may be used in women who are high risk, but not a typical modality due to cost, no
standard imaging protocol, and inability to detect microcalcifications.
o Fine-Needle Aspiration (FNA) – if clear fluid and reduction of mass occurs, pt may be scheduled
for follow up in 4-6wk. If bloody, send to cytology and initiate a mammogram/ultrasound. If
mass re-appears upon follow-up, initiate mammorgram/ultrasound.
o Core-Needle Biopsy – often used in larger, solid breast masses to assess cellular architecture
 Algorithm
o If it’s cystic  do an FNA
 Clear/mass disappears  no worries, follow up and reassess
 Clear/mass persists  mammogram/ultrasound
 Bloody  send for cytology/perform mammogram/ultrasound
 Nothing aspirated  perform mammorgram/ultrasound
o If you’ve done a mammogram/ultrasound
 Complex cyst  FNA and consult surgery/rads
 Simple cyst  aspirate or just watch depending on symptoms
 Suspicious/intermediate/solid mass  excisional biopsy
 Normal  follow-up in 2-3mo
Benign Breast Diseases
 Mastalgia – breast pain, characterized by it’s type of onset
o Cyclic – begins in luteal phase and resolves with menses. Bilateral/upper outer quadrants. The
result of proliferation of breast tissue during normal hormonal changes
o Noncyclic – not associated with menstruation and worrisome. May be idiopathic but must be
worked up for other causes. May be the result of hormone therapy (OCPs, ERT, etc.)
o Extramammary – chest wall trauma, rib fractures, shingles, you name it. Lots of things can
happen to the chest wall, so just because it’s there, doesn’t mean it’s the breast.
o Treatment can include:
 Hormone suppression with danazol, bromocriptine, GnRH agonists, Lisuride maleate
(DA agonist), progestin-only contraception, or SERMs (tamoxifen/raloxifene; only used
in severe mastalgia that’s unresponsive to conventional therapy.)
 Assessment for better fitting bra/use of a sports bra
 Nipple Discharge – it’s all about the color, consistency, and bilaterallity
o Spontaneous, non-bloody, bilateral – fibrocystic changes or ductal ectasia
o Green, yellow, or brown-sticky bilateral – ductal ectasia or fibrocystic changes in the breast
o Milky bilateral – childbearing or endocrine abnormality
o Purulent – infection of duct
o Bloody/unilateral – intraductal papilloma, invasive ductal carcinoma, intraductal carcinoma
 Will need workup with ultrasound, mammography, and possibly ductograpy
 Benign Breast Mass
o Signs of malignancy - >2cm, immobile, poorly defined margins, firm, skin dimpling, skin color
changes, retraction or changes to nipple, blood discharge, or ipsilateral lymphadenopathy.
o If these signs are not present, then you’re more likely to have a benign mass
o Non-proliferative lesions
 Fibrocystic changes – dilation and scarring/inflammation that occurs normally in women
that can may be breast diffusely more dense/firm
 Adenosis – increase in number of glands/lobular growth that can produce small masses
 Simple Fibroadenoma – tumor of young women; round, rubbery, mobile; do NOT have
malignant potential but because they’re made of breast tissue they can enlarge with
normal cycles or pregnancy and cause discomfort
o Proliferative Lesions without Atypia – often do NOT cause a palapable mass, but are
proliferations of benign cells of either ductal or lobular epithelium
 Epithelial hyperplasia – expansion of the basement membrane making for more than 2
layers of cells along the ducts
 Sclerosing adenosis – increased fibrosis/distortion and compression of the epithelium
 Radial scar – nidus of tubules trapped within a hyalized stroma; mimicking invasive
carcinoma as the arms of sclerosis radiate outward.
 Papilloma – intraductal growths with abundant stroma and normal lining. These are the
most common cause for blood in nipple discharge (sero-sanguinous)
o Proliferative Lesions with Atypia – proliferation of malignant epithelium lining the ducts
resulting in a carcinoma in-situ. This has malignant potential, but it not cancer yet due to an
intact basement membrane by definition.
 Lobualr (LCIS) – involves the lobules, often bilateral and multicentric (multiple spots).
Biggest risk for malignancy is either breast.
 Ductal (DCIS) – involves the ducts, often unilateral and unicentric (in one spot). Biggest
risk for malignancy is in the breast that it’s appeared in.
 Both should be evaluated with a core-needle biopsy and treated accordingly; note that
SERM usage after treatment has been shown to reduce the risk of invasive breast cancer
Breast Cancer
 #2 cancer in women (behind skin cancers), with a steadily increasing rate attributed to increase of
mammographic screening, finding small disease which would have previously gone undetected.
 Risk Factors
o Age – the #1 risk factor for developing breast cancer. Older women get breast cancer (>age 50)
which is why we begin regular mammograms at age 50.
o Race – White women have higher rates (go figure!)
o Genetics – if a woman is diagnosed with breast cancer before age 40, she should have
screening for genetic mutations (most notably BRCA1/BRCA2)
o Estrogen exposure – increased estrogen exposure for the classic reasons are associated
o Breast changes – women with dense breast tissue are at higher risk
o Others – overweight/obesity (^estrogen) and alcohol use (unknown) are both assocaited
 Staging – TNM staging (Tumor characteristics, Node involvement, and Metastases) as well as estrogen
receptor testing (Her2/neu) can help predict prognosis
 Treatments
o Surgical
 Mastectomy – removal of all breast tissue with preservation of pectoralis muscles
 Lumpectomy + radiation – removal of breast tumor only. Radiation is essential and the
combination of the two approaches similar outcome rates as mastectomy
 Breast re-construction should always be offered to patients after these procedures
o Medical
 Classic chemo drugs used as adjuvant therapy are often used with widespread disease
 Aromatase inhibitors can be used to limit estrogen in post-menopausal women
 SERMs (tamoxifen/raloxifene) can be used to limit estrogen exposure only in the breast.
Unfortunately, these can be agonists at other sites increasing risk of endometrial cancer.
 Trastuzumab – useful if cancer is Her2-neu(+) as it targets these cells for destruction;
serious side effects of heart failure, respiratory problems, allergic rxns can occur
Screening Guidelines
 Breast Self Exams are NOT recommended any longer, as they were found to be ineffective and lead to
excess concern/testing for benign breast diseases
 Breast self-awareness is now recommended (being aware of normal breast architecture/look and
voicing concern if this changes)
 Mammography value increases with increasing age, thus we begin at age 50 for regular screening
 Guidelines
o Women age 20-39 – clinical breast exam every 1-3yr
o Women age 40-49 – clinical breast exam every 1-3yr +/- mammography
o Women age 50+ – clinical breast exam and mammography every year
o Women with BRCA mutations – monthly breast self-exams starting at age 18, annual clinical
breast exams at age 18, and annual mammography +/- MRI at age 25

[Chapter 33: Disorders of The Breast – Topic 40]


 Stimulation of the breast during the physical examination may give rise to an elevated prolactin level
and promote milky discharge. Post partum women may continue to produce milk for up to two years
after cessation of breastfeeding
o Accurate prolactin levels are best obtained in the fasting state.
o If still elevated, then a TSH level and brain MRI are indicated to rule out a pituitary tumor.
o Although pathologic factors such as hypothyroidism, hypothalamic disorders, pituitary
disorders (adenomas, empty sella syndrome), chest lesions (breast implants, thoracotomy
scars, and herpes zoster) and renal failure can elevate prolactin levels, a non-significant benign
elevation needs to be ruled out first.
 If there’s a dominant breast mass, but no obvious signs of malignancy an FNA is a great first option. If
it’s got signs of malignancy, then an excisional biopsy may be indicated.
 Fibrocystic breast changes are the most common type of benign breast conditions and occur most
often during the reproductive years. Fibrocystic disease is often associated with cyclic mastalgia,
possibly related to a pronounced hormonal response. Caffeine intake can increase the pain associated
with fibrocystic breast changes, so recommending decreased caffeine intake may be helpful.
 The first noticeable symptom of breast cancer is typically a lump that feels different from the rest of
the breast tissue. More breast cancer cases are discovered when the woman feels a lump.
o Even though the mass decreased in size after aspiration, the bloody discharge obtained
obligates an excisional biopsy be performed to rule out breast cancer.
o If clear discharge is obtained on aspiration and the mass resolves, reexamination in two
months is appropriate to check that the cyst has not recurred.
 Most postpartum mastitis is caused by staphylococcus aureus, so a penicillin-type drug is the first line
of treatment. Dicloxacillin is used due to the large prevalence of penicillin resistant staphylococci.
Erythromycin may be used in penicillin allergic patients.
 If you have a dominant breast mass and FNA fails to reduce it, get an excisional biopsy.

2345: Evaluation of a Breast Mass


 Step 1: Breast Mass found/reported; do a breast exam to confirm report
 Step 2: Ultrasound & mammography for evaluation of the found mass
o If <30  ultrasound +/- mammography
o If >30  mammography +/- ultrasound
 Step 3: Evaluate the findings
o Simple cyst  FNA to asses fluid; if normal, come back in 2-4 months for evaluation
o Complex cyst or solid mass  core biopsy with possible excision of mass depending on results

2607: Paget’s Disease of the Breast/Nipple


 Presentation: persistent eczematous/ulcerating rash around the nipple/areola with pain/itching
without relief from topic corticosteroid cream
o 85% of these patients have underlying adenocarcinoma
 Dx: ultrasound/mammography with biopsy
 Tx: excision

Case 50 – Irregular Menstrual Cycle


Never forget, the #1 reason for an abnormal menstrual period is pregnancy.

Chapter 37: Reproductive Cycles; Ch 39: Amenorrhea/Abnormal Uterine Bleeding – Topic 43/45 QUESTIONS
 Most women resume normal menstrual cycles after discontinuing oral contraceptive pills (OCPs), they
are not usually considered the cause of the amenorrhea. A history of irregular cycles prior to pill use
may increase the risk of amenorrhea upon discontinuation. This is sometimes referred to as “post pill
amenorrhea.”
 Disorders of clotting may present with menstrual symptoms in young women, with Von Willeberand
disease being most common.
 Management of an endometrial polyp includes the following: observation, medical management with
progestin, curettage, surgical removal (polypectomy) via hysteroscopy, and hysterectomy.
o Observation is not recommended if the polyp is > 1.5 cm.
o In women with infertility polypectomy is the treatment of choice. While her inability to get
pregnant may be more complicated than just her polyp, removal of the polyp should occur prior
to infertility treatments.
 Abnormal uterine bleeding is a term used to describe uterine bleeding abnormalities.  This term can
encompass both structural causes (polyp, adenomyosis. Leiomyoma, or malignancy [or hyperplasia]) as
well as non-structural causes (coagulopathies, ovulatory dysfunction, endometrial, iatrogenic or not
classified). The acronym PALM-COEIN is a means for this classification.
 Hysteroscopic myomectomy preserves the uterus, while removing the pathology causing the patient’s
symptoms. A laparoscopic approach is not indicated as the myoma is submucosal and not accessible
using a laparoscopic approach

4769: Pts who are in their first year of ovulation will experience 90% annovulatory cycles in that year. This
means that their cycles will be irregular with breakthrough bleeds and often heavy because the lack of
ovulation means a lack of endometrial sloughing. Thus the endometrium gets built up until the woman finally
does ovulate, making for very heavy periods.
 If periods become more normal (1/month) and are still very heavy, bleeding disorders and
abnormalities of the uterus should then be considered, but not in the first 1-2 years of cycling.

[Chapter 48: Uterine Leiomyoma and Neoplasia – Topic 53]


Uterine Leiyomyoma (“Fibroids”)
 Presentation: vaginal bleeding with progressively heavier menstrual flow, which is longer in duration
with progressive pain that ultimately persists beyond menstrual bleeding; iron deficiency anemia,
subjective pelvic pressure/fullness, and necrosis of the fibroid can also occur
o Abnormal uterine distortion often present in large fibroids; sometimes palpable externally as a
large, solid, midline, irregular-contoured, mobile pelvic mass.
o Compression of the ureters can cause hydrouteter/hydronephrosis
 Etiology: abnormal growth of the uterine smooth muscle for poorly defined reasons
 Dx: presentation with ultrasound/MRI confirmation of uterine distortion with masses
o Surgical evaluation may be used in pts where imaging does not confirm diagnosis but diagnosis
is strongly clinically suspected
 Tx
o Most commonly pts are asymptomatic and do NOT require treatment
o Medical treatment: progestin OCPs or local hormonal contraceptives with NSAIDs to minimize
symptoms until menopause; often GnRH analogous are used to reduce fibroid size (40-60%) if
surgical treatment is sought
o Surgical treatment: typically used in rapidly enlarging masses, masses unresponsive to medical
treatment, and masses causing complication (infertility, anemia, hydronephrosis)
 Myomectomy: removal of only the fibroid tissue. Not a definitive treatment, but good
for retention of fertility
 Hysterectomy: definitive treatment for women who are done with childbearing or in
dire situations. Fibroids are the most common cause of hysterectomy
 Myolysis or Uterine Artery Embolization: both seek to shrink/destroy fibroid tissue, but
rarely can take out all fibroids. May be useful in patients who are poor candidates for
surgery and only seek relief for symptoms. Not currently recommended for women
trying to continue to have children.
 Effects on Pregnancy: associated with infertility due to distortion of the uterine cavity with large
leiomyomas
o Typically, medical treatment is strictly indicated, however discrete pedunculated myomas may
be surgically removed if easily clamped/ligated.
o Otherwise myomas should be removed at time of delivery.
o Rarely, myomas can distort the lower portion of the uterus, contributing to dystocia. If this
occurs, c-section is warranted.
8943: Uterine Leiomyoma (Fibroids)
 Presentation: progressive menorrhagia with dysmenorrhea with abnormally shaped, irregularly
enlarged uterus on exam. May cause pressure on bladder/bowels causing painful urinartion/defecation
o May prolapse through the cervical os if present submucosally, showing a firm, whitish, smooth
mass protruding through the cervical os causing labor-like pain and cervical dilation.
 Dx: pelvic ultrasound (don’t do a biopsy or a CT/MRI!)
 Tx: If asymptomatic  observation; if perimenopausal  contraception till menopause; if definitive
treatment if sought  embolization, myomectomy, or hysterectomy are all options!

2408/2606: When considering SERMs there’s really two:


 Tamoxifen: antagonist at the breast, but agonist everywhere else
o Used as adjunctive therapy for breast cancer
o Inc risk of DVT/hot flashes/endometrial cancer and hyerplasia
 Raloxifene: antagonist at the uterus and breast, but agonist every where else
o Used to treat osteopososis when pt cannot tolerate bisphosphonate
o Inc risk of DVT/hot flashes
 Note that hot flashes are the most common side effect of SERMs

Chapter 48: Uterine Leiomyoma and Neoplasia – Topic 53 QUESTIONS


 The #1 symptom in uterine fibroids is menorrhagia (think of your lady in clinic!) due to:
o Increase in the uterine cavity size that leads to greater surface area for endometrial sloughing
o Obstructive effect on uterine vasculature that leads to endometrial venule ectasia and proximal
congestion in the myometrium/endometrium resulting in hypermenorrhea.
 Fibroids can be detected in as high as 80% of women. Most uterine fibroids are asymptomatic and do
not require any treatment.
o Fibroids may grow or become symptomatic in pregnancy due to hemorrhagic changes
associated with rapid growth, known as red or carneous degeneration. However, this is
uncommon for smaller fibroids.
o Fibroids can be located below the fetus, in the lower uterine segment, or cervix, causing a soft
tissue dystocia, necessitating delivery by Cesarean section.
o Myomectomy at the time of Cesarean section should be avoided, if possible, secondary to the
risk for increased blood loss
o It is not necessary to follow the growth of fibroids during pregnancy, except for the rare cases
when the fibroid is causing symptoms (primarily pain) or appear to be located in a position
likely to cause dystocia.
 Leiomyomas are an infrequent cause of miscarriages and subfertility either by mechanical obstruction
or distortion (and interference with implantation).
o Submucosal or intracavitary myomas cause lower pregnancy/implantation rates via:
 Focal endometrial vascular disturbance
 Endometrial inflammation
 Secretion of vasoactive substances.
o Submucosal fibroids are best treated by hysteroscopic resection.
 Medical management of fibroids includes 1NSAIDs and progestin OCPs/IUDs; but if that doesn’t work
2
GnRH analogues can be used to shrink fibroids for either symptomatic tx OR easier surgical resection.
o Don’t forget! Fibroids feed off estrogen! Thus, just giving estrogen would be a poor choice.
o Often GnRH analogues are used in older women (this lady was 49) until they reach menopause,
where the fibroids will shrink due to loss of endogenous estrogen.
 This question had a woman who was worried her big belly (BMI 37) was caused by a large fibroid.
Sadly, she’s just obese and should be counselled on diet and exercise.
o Bariatric surgery may be considered when BMI is greater than 40, or is between 35 to 39.9
accompanied by a serious weight-related health problem, such as type 2 diabetes, high blood
pressure or severe sleep apnea. This lady had no other problems.
 If patients present with menstrual abnormalities, the endometrial cavity should be sampled to rule
out endometrial hyperplasia or cancer, even if it really sounds like fibroids. This is most important in
patients in their late reproductive years or postmenopausal years.
o If the patient’s bleeding is not heavy enough to cause iron deficiency anemia, reassurance and
observation may be all that are necessary.
o Treatment with GnRH analogues to inhibit estrogen secretion may be used as a temporizing
measure. Can be used for three to six months prior to a hysterectomy to decrease the uterine
size and increase a patient’s hematocrit; either leading to easier surgery (smaller fibroid/less
blood loss) or tide a patient over into menopause.
 Mean uterine size decreases 30-64% after three to six months of GnRH agonist treatment.
o In a patient who desires future fertility, this may be a good short-term option to decrease
pain/blood loss; esp. if hematocrit is decreasing.
o Maximal response is usually achieved by three months of GnRH agonist treatment. The
reduction in size correlates with the estradiol level and with body weight.
 When treating with a GnRH agonist:
o Hot flashes are experienced by >75% of patients (3-4wk) but shouldn’t persist for more than
one to two months.
o After cessation of treatment, menses return in 4-10wk, and myoma/uterine size return to
pretreatment levels in 3-4mo.
 Fibroids will regain their normal growth after cessation of treatment; the myth that
they’ll grow faster due to hormone withdrawal is unsubstantiated.

[Chapter 49: Cancer of the Uterine Corpus – Topic 54]


Endometrial Hyperplasia
 Types:
o Simple Hyperplasia (1% risk progression) – proliferation and stromal cell proliferation together.
Does NOT typically progress to endometrial carcinoma
o Complex Hyperplasia (3% risk progression) – abnormal proliferation of mainly glandular
elements without accompaniment with stromal elements giving a “crowded” appearance.
o Hyperplasia with Atypia (8% if simple, 29% if complex) – either type of hyperplasia where
glandular elements show cytologic atypia and disordered maturation (loss of polarity, nuclear
enlargement, increased nuclear-to-cytoplasmic ratio, dense chromatin, and prominent nucleoli)
 Pathophysiology: overgrowth of the endometrium due to unopposed estrogen exposure
o Endogenous sources: ovaries, fat, or granulosa cell tumor of the ovary
o Exogenous sources: exogenous estrogen of any kind (OCPs, patches, creams, etc.)
 Risk Factors: all have to do with estrogen exposure, arranged from highest to lowest risk increase:
o High-dose menopausal estrogen (10-20x), Living in N.America or N.Europe (3-18x), Tamoxifen
use (3-7x), estrogen producing tumor (5x), Obesity (2-5x), Increased estrogen exposure from
menstrual cycles (nulliparity, infertility, early menarche, late menopause)
 Presentation: abnormal uterine bleeding should prompt further evaluation in women >35yrs or
women <35yrs with risk factors
 Dx:
o Endometrial evaluation: endometrial biopsy (90-98% accuracy) should be initial evaluation.
D&C and pap smear are not as accurate at diagnosis
o Transvaginal Ultrasound: if you see an endometrial thickness >4mm in post-menopause, a
polypoid mass, or fluid collection you are prompted for further evaluation
 Tx:
o Medical: synthetic progesterones (medroxyprogesterone acetate; decrease estrogen receptors
and thin the endometrium) are used typically in pts without atypia
o Other Tx: may be therapeutic D&C or definitive treatment with hysterectomy

Endometrial Polyps
 The result of focal, benign hyperplasic growth with abnormal bleeding being the primary presentation
 Less than 5% of polyps are malignant; although may be removed
 More often malignancy in post-menopausal women, esp. if taking tamoxifen therapy

Endometrial Cancer
 Presentation: abnormal uterine bleeding in a post-menopausal woman
 Pathogenesis:
o Estrogen dependent (90%; Type I): often from estrogen exposure; good prognosis
o Estrogen independent (Type II): arise spontaneously; poorly differentiated; poor prognosis
o If invasive, cancer typically spreads through endometrial cavity, into myometrium, then
endocervix, then into the lymphatics
 Risk Factors: identical to those for endometrial hyperplasia
 Dx: endometrial biopsy showing cancerous change with pre-operative CA-125 level
o Prognosis: Higher grade and deeper depth of invasion yield a worse prognosis
 Tx: hysterectomy with complete surgical staging and assessment of regional/retroperitoneal lymph
nodes is both therapeutic and improves survival
o If lymph nodes are positive, post-surgical radiation/chemotherapy is critical for treatment

Recurrent Endometrial Carcinoma


 If pt has not received radiation: evaluate with speculum/recto-vaginal exam every 3-4mo for 2-3yr
 If radiation therapy: less frequent surveillance is used
 If present, recurrent estrogen or progestin (+) cancer can respond well to high-dose progestin therapy

Hormone Therapy After Endometrial Carcinoma Tx


 Post-menopausal estrogen therapy MAY be used in women who have be previously treated for
endometrial carcinoma, but good candidates must be selected based on certain prognostic indicators
and women must know of the risk they have.

Uterine Sarcoma
 Presentation: progressive uterine growth in a post-menopausal woman, post-menopausal bleeding,
pelvic pain, increased vaginal discharge
o Even if they’re on estrogen therapy, this type of growth should always be considered pathologic
o Often these tumors are extremely aggressive, thus treatment must be prompt
 Dx: surgical evaluation, resection, and histologic evaluation
 Tx: hysterectomy
o Radiation, chemotherapy, and and hormonal therapy do not produce good results
2391: Abnormal Uterine Bleeding (AUB) is always concerning for endometrial cancer, thus an endometrial
biopsy is typically warranted to rule this out. Criteria indicating an endometrial biopsy include:
 Age >45 – AUB or post-menopause bleeding  do an endometrial biopsy!
 Age <45 – AUB + [unopposed estrogen, failed medical management of AUB, or Lynch syndrome]
 Remember that a Pap smear is ONLY A SCREENING test and should never be used to confirm suspected
pathologic changes!

Chapter 49: Cancer of the Uterine Corpus – Topic 54 QUESTIONS


 In a post-menopausal patient having abnormal uterine bleeding, risk factors for endometrial carcinoma
can help determine how likely they’ve developed a cancer/possible severity. In order of highest
conferred risk:
o Finding of complex endometrial hyperplasia on endometrial biopsy – 28% of pts have cancer
o Obesity (esp. if >50lbs over ideal body weight) – 10x normal risk
o Nulliparity, early menarche, late menopause, hypertension, diabetes and exposure to
unopposed estrogens (HRT or SERMs) – all increase to different degrees
 Less than 5% of women diagnosed with endometrial cancer are asymptomatic. Approximately 80-90%
of women with endometrial carcinoma present with vaginal bleeding or discharge as their only
presenting symptom. Thus, if a patient is asymptomatic and lacks many risk factors (or all in this case)
yearly regular exams are the best course of action.
 The most common symptom is abnormal postmenopausal bleeding. This may be accompanied by
abnormal vaginal discharge, lower abdominal discomfort, and increased size of the uterus, but these
are less common findings.
o Note that combination ERT does NOT increase the risk of endometrial cancer
 This question has a 65y/o G2P1 with homogenous thickening of the endometrium to 5mm. Biopsy
revealed rare atypical cells. D&C is the correct next step. In a woman with persistent symptoms
suggestive of endometrial cancer, AND with a negative biopsy, further investigation with D&C to
sample the entire endometrium is the best course of action.
 This question has a lady that had FIGO Grade 1 endometreoid adenocarcinoma on endometrial biopsy.
This is low-grade and the next best step is to confirm that there’s no spread of the cancer outside the
uterus, so we can possibly have less intense treatment thus:
o Routine evaluation in this setting should include a chest x-ray as the lungs are the most
common site of distant spread.
o CA-125 may be helpful in predicting those patients that may have extrauterine spread, but is
not absolutely necessary
o A pelvic ultrasound is not indicated once a pathologic diagnosis has been established, although
one may have been obtained as part of the initial evaluation of postmenopausal bleeding.
o When there is a low suspicion for advanced disease, a CT scan, MRI, PET scan, and other
invasive and costly tests are not indicated.
 Recommended components of the surgical approach to an early endometrial cancer are the
extrafascial total abdominal hysterectomy, bilateral salpingo-oophorectomy, and pelvic and para-
aortic lymphadenectomy; best done in high grade (>3) or aggressive types (clear cell, papillary serious)
o Vaginal or laproscopic approaches can also be done as alternatives
o Chemotherapy, radiation, and hormonal therapy may be utilized, but in an adjuvant setting.
 Graunlosa cell tumor (Granulosa-Theca tumor) is a functional, estrogen secreting tumor that may
cause increase bleeding, “speeding up” of periods, and complex endometrial hyperplasia w/out atypia
 Theca-lutein cyst – typically bilateral cysts that occurs due to markedly increased B-hCG production
(most often during a molar pregnancy), causing uniformly complex, fluid filled ovarian cysts. Often
these resolved after B-hCG level return to normal.
 Postmenopausal bleeding or discharge accounts for the presenting symptom in 80-90% of women
with endometrial cancer, thus if these occur, an H&P, pelvic examination, and endometrial biopsy
should occur. However, the causes for post-menopausal bleeding are as follows:
o Atrophy of the endometrium (60-80%), hormone replacement therapy (15-25%), endometrial
cancer (10-15%), polyps (2-12%), and hyperplasia (5-10%).
o Transvaginal ultrasound can be a useful tool as it can visualize polyps, myomas, or endometrial
thickening. Also, if the endometrial stripe is <5mm, there’s a low chance for endometrial cancer
to be present
 This question had a lady with remote hx (15yr ago) of endometrial cancer now presenting with cough,
dyspnea, and bilateral lower extremity edema.
o The most appropriate next step is to refer her to a pulmonologist (or cardiologist) for a
thorough work-up. Pleural effusions and lower extremity edema point towards a
cardiopulmonary etiology and you need to rule this out before going for cancer.
o However, the finding of a solitary lung nodule in a patient exposed to second hand smoke
certainly suggests the possibility of a primary lung cancer.
 While Taxomifen is known to increase risk of endometrial cancer, any woman on therapy with it should
have regular yearly exams from their gynecologist. If they’re asymptomatic, no ultrasound/biopsy
studies are needed.

Case 51 – Diabetes Mellitus


[Type I Diabetes]
Autoimmune destruction of pancreatic B-cells (produce insulin)
 Juvenile onset & pathogenesis poorly understood
o 95% of pts with T1DM have HLA Haplotype DR3/DR4
o Viral infections (coxsackie virs & rubella) have been linked with development
o Girls with protracted monilial (Candidia) vulvovaginitis may have early Type 1 DM
o Autoantibodies formed – islet cell antibodies (ICA; 85%), anti-insulin antibodies, or angi
glutamic acid dearcoxylase antibodies; often can be detected well before clinical manifestations
o Adolescents may present with DM1 during puberty growth spurt (GH and sex steroids are
antagonistic to to insulin action…resulting in setting of phenotype)
 Prone to diabetic ketoacidosis (DKA)
o Result of nearly no uptake of sugar from the blood into cells
o Because cells can’t get sugar, they start to metabolize fat, producing ketones at a large scale
o This is the basis for a ketogenic diet (burns fat cuz no carbs!)
[Type II Diabetes]
Resistance to the Effects of Insulin
 Adult onset & pathogenesis involves first the resistance to uptake causing a constant hyper-insulin-
emic state, eventually giving way to “burnout” of the pancreatic B-cells
 Patients will be hyperinsulinemic for years until symptoms occur, then will be diabetic for many more
years until burnout occurs (the “insulin dependent type II diabetic”)
 Prone to Hyperosmolar Hyperglycemic State (HHS)
o Result of poor uptake of sugar
o However, the sugar CAN (to some degree) enter the cells, making fat burning not necessary,
thus the build-up of ketones doesn’t occur
o Extreme serum osmolarity results in diuresis & dehydration causing problems
[Gestational Diabetes]
4793/12179: Gestational Diabetes Mellitus
 Presentation: abnormal fasting glucose with onset during pregnancy
o The results of B-cell hyperplasia and physiologic peripheral insulin resistance due to hPL
production by the placenta (increase sugar availability for the fetus/placenta)
 Dx:
o At 24-28wk: 1-Hr 50g glucose challenge  target if <140 mg/dL
o If >140 mg/dL: 100g glucose challenge; check levels each hour for 3 hrs; if 2 parts are abnormal
you’ve made your diagnosis
 Fasting – <95 mg/dL is normal
 Hr 1 – 180 mg/dL is normal
 Hr 2 – 155 mg/dL is normal
 Hr 3 – 140 mg/dL is normal
 Tx: 1. Dietary modifications (balanced diet with glucose control); if not controlled then 2. Insulin
(intermediate + short-acting BID; best choice) 3.metformin or glyburide can be used as well
o Weight loss is NOT recommended
o Goals: fasting glucose <100 mg/dL and 1hr post-prandial <140 mg/dL
o Strong evidence that fasting levels <95 mg/dL provide better neonatal outcomes

A history of gestational diabetes mellitus (GDM) is the greatest risk factor for future development of diabetes
mellitus. It is thought that GDM unmasks an underlying propensity to diabetes. While a healthy pregnancy is a
diabetogenic state, it is not thought to lead to future diabetes. About 50% of women with a history of GDM
will develop type 2 diabetes within 10 years of the affected pregnancy. Women should be screened for DMII
in the post-partum period because of this.

[Diabetes Insipidus]
3729/3899: Diabetes Insipidus
 Presentation: increased thirst, need to urinate, normal fasting glucose
 Dx:
o Water deprivation test – serial measure of urine volume/osmolality until levels are stable on
approximately 2-3hrs of testing
 Diabetes Insipidus – urine will continue to be dilute, while serum will be concentrated
 Primary Polydipsia – urine will concentrate (>600mOsm)
o Vasopressin Challenge – give vasopressin (ADH) to see if urine concentrates
 Central DI – urine successfully becomes concentrated (kidneys can respond)
 Nephrogenic DI – urine fails to concentrate
 Tx:
o Central DI – intranasal demopressin (ADH analogue)
o Nephrogenic DI – supportive care and HCTZ or other diuretics
 Discontinue offending drugs lithium, demeclocycline, foscarnate, cidofovir
amphotericin B

[“Bronze Diabetes”]
Hereditary Hemochromatosis
 Presentation: Skin (Hyperpigmentation, “bronze diabetes”), MSK
(arthalgias/arthropathy/chondrocalcinois), GI (elevated LFTs, with hepatomegaly  cirrhosis,
increased risk of hepatocellular carcinoma), endocrine (diabetes, hypogonadism/hypothyroidism,
diminished libido/erectile dysfunction), cardio (restrictive cardiomyopathy/abnormal condution), and
infections (Listeria, Vibrio, and Yersinia increased susectibility)
 Dx: HFE genetic mutation and elevate iron studies
 Tx: serial phlebotomy (decrease iron store)

[Screening and Diagnosis of Diabetes]


3807: Screening for Diabetes Mellitus
 Available Tests: Fasting Plasma glucose (>126), Random glucose (>200), HbA1c (>6.5), or oral glucose
tolerance test (>200) – the values indicate a diagnosis of diabetes mellitus
 All tests should be repeated to confirm diagnosis
 USPSTF – screen all patients with sustained BP >135/80
 ADA – screen all patients >45y/o with risk factors for DM

Ways to Diagnosis Diabetes Mellitus (all must be confirmed with repeat testing)
 Hemoglobin A1c level ≥6.5%
 Fasting plasma glucose level ≥126 mg/dL (no caloric intake for 8hr prior)
 Random glucose level ≥200 mg/dL in a patient with symptoms of diabetes
 2-hour oral glucose tolerance test value ≥200 mg/dL.
 While a urine dipstick may be used to screen for diabetes, it is not a diagnostic test.

Some Fast Facts in Making the Diagnosis


 HbA1c of 6-6.5% is called “pre-diabetes” and patients should undergo lifestyle changes to stop it.
 Fructosamine levels can indicate glucose control over the past 2-3wk; used in place of HbA1c in those
with hemoglobinopathies or recent intense blood loss

Once the Diagnosis is Made


 Fasting lipid panels (annually) – DM has a huge association with metabolic syndrome
 Serum creatinine/urinalysis/urine microalbuminuria (annually) – diabetic nephropathy is a big killer
 Dilated eye examinations (annually) – diabetic retinopathy is a huge blow to life quality
 Regular foot exams – diabetic foot ulcers/neuropathy are a huge killer/blow to quality of life
 Thyroid Screening (Type 1 only) – due to autoimmune nature of disease, autoimmune damage to or
activation of the thyroid gland is not uncommon

[Management of Diabetes]
Type I Diabetics rely on Insulin therapy. Typically a 1.short-acting at meal time with 2. Intermediate or long-
acting once a day for baseline coverage.

Type II Diabetics should initially attempt control with lifestyle changes (it’s ALWAYS the first intervention).
However, diabetes drugs should be used to aid glucose control. The following are considered in descending
order for control:
 Metformin (1-2% drop) – typically first agent in DM control.
o Mec: decreases gluconeogenesis primarily; sensitizes peripheral tissue to insulin aiding in
uptake & decreases intestinal uptake of glucose secondarily
o Weight neutral/decrease in TAGs & cholesterol
o Low risk of hypoglycemia
o Most commonly causes GI upset and diarrhea
o Life threatening lactic acidosis with renal insufficiency (Cr of >1.5 in men, >1.4 in women),
hepatic failure or heart failure
 Sulfonylureas (glipizide; 1-2% drop) – typically added with metformin monotherapy failure.
o Mec: stimulate B-cells of pancreas to enhance insulin secretion
o Weight gain and hypoglycemia are major side effects.
 Thiazoliadones (“-glitazone“; 1-1.5% drop)
o Mec: improves insulin sensitivity peripherally for enhanced uptake of glucose
o Low risk of hypoglycemia with slow onset of action (12wk of therapy for effect)
o Metabolized by the liver, thus Can be used in renal insufficiency
o Weight gain, edema, CHF, bone fracture are all possible.
 Meglinitides (“-glinide”; 0.5% drop)
o Mec: rapid onset stimulation of insulin secretion from the pancreas
o Onset within an hour; but high cost and risk of hypoglycemia
 a-Glucosidase inhibitors (Acarbose)
o Mec: inhibit the enzyme a-glucosidase, decreasing absorption of glucose from the intestines
o Best in limiting post-prandial sugar spikes, and useful in patients with sporatic eating habits
o Side effects of retention of sugar in gut causing flatulence/diarrhea
 GLP-1 agonists (exenatide; 0.5-1% drop) – low hypoglycemia risk, may aid in weight loss. Can cause
acute pancreatitis and hypoglycemia when added to a sulfonylurea
o Mec: stimulates GLP-1 receptors to enhance release of insulin from pancreas
o Upper respiratory symptoms & hypersensitivity are the major side effects
 DPP-4 inhibitors (“-gliptin”; 0.5% drop) – low risk of hypoglycemia, weight neural, can be used in renal
insufficiency
o Mec: stops inactivation of GLP-1/GIP which are release from the gut to stimulate the pancreas

11367: Tight glycemic control in Diabetes Mellitus


 While glycemic control goals are between 6-7%, it doesn’t decrease all the complications of DM:
o Macrovascular complications (stroke, atherosclerosis, etc.) don’t change with tight control
o Microvascular complications (nephropathy, retinopathy) decrease with tight control
o All-case Mortality is not changed with tight control, but goes up with poor control

2910/3902: Tight Blood Pressure Control in Diabetes Mellitus


 All diabetes patients should be treated to keep blood pressure around 140/90
 Pts with diabetic nephropathy should have more intese blood pressure control of 130/80 to limit the
effects of nephrotic syndrome. Typically, careful administration of an ACE or ARB are warranted. This
has been proven to decrase end-organ damage and should be initiated at the first signs of diabetic
nephropathy (microalbuminuria)

3823/4336: Guidelines for Lipid-Lowering therapy (typically a statin)


 Symptomatic atherosclerotic disease  high or low intensity based on age (> or < than 75yr)
 LDL >190mg/dL  high intensity statin therapy
 Age 40-75 with diabetes  high or low depending on 10-year ASCDV risk
 Estimated 10-year ASCVD risk >7.5%  moderate to high intensity statin therapy

3822: (refer to 2698) A patient with Type II Diabetes Mellitus over the age of 40 should always be on a statin
and initiate positive lifestyle changes according to lipid lowering guidelines. Refer to 2698 for the complete
guidelines.
4171: Side effects of Thiazide diuretics (HCTZ or chlorthalidone)
 Glucose intolerance – worse insulin release from pancreas AND tissue resistance to glucose uptake.
Worse in pt with diabetes or metabolic syndrome
 Poor lipid metabolism with increased LDL/triglycerides – similar to glucose intolerance. Worse in
diabetes or metablic syndrome
 Hyponatremia/kalemia/magnesemia and hypercalcemia – based on mechanism of action. Actually
beneficial in staving off osteoporosis
 Hyperureciemia and worsening gout – reduces renal uric acid excretion

[Complications of Diabetes]
2171/2184: Diabetic Ketoacidosis (DKA)
 Often can be the initial presentation of a young person with Type I diabetes mellitus. Often pt will
have weight loss/polydipsia and be able to compensate by simply drinking more water. However,
anything disrupting oral intake (recent illness) can throw this off and send them into DKA
 Presentation: polydipsia/polyuria, burred vision, weight loss, altered mentation, hyperventilation,
abdominal pain
o Labs: hyperglycemia (250-500s), bicarb <18 (acidosis), elevated anion gap, (+)serum ketones
 Dx: presentation with specific labs
 Tx: High-flow IV fluids, IV insulin, close watch/replacement of potassium
o Note the ketone level (B-hydroxybutryate) or anion gap are the best methods by which you
track response to treatment

4247: Hyperkalemia in DKA [HIGH YIELD]


 When DKA occurs, pts often show paradoxical hyperkalemia due to 1extra-cellular shift of K in
exchange for H+ ions to limit the acidosis and 2impaired K+ entry into cells due to low insulin levels
(insulin helps K+ enter into cells)
o Called “paradoxical” because the body is actually wasting K+, and overall levels are low,
however serum concerntration is high due to extracellular shift
 Thus pts in DKA should be given supplemental K+, insulin, and IV fluids because once insulin is on
board, the K+ level can tank quickly, unless it’s very high

2186: Hyperosmolar Hyperglycemic State (HHS)


 Presentation: old person with type II diabetes; confusion/lethargy, fever, dehydration, gradual onset of
hyperglycemia symptoms (polyuria, polydipsia, etc.)
o Labs: electrolyte abnormalities, glucose >1000, pH around normal (7.4ish), normal bicarb,
negative serum ketones, serum osmolality >320
o Pts often have paradoxical hyperkalemia driven by insulin deficiency/hyperosmolality resulting
in severe diuresis. The K+ moves out and is peed out.
 Dx: clinical picture with normal pH and severely high glucose (>1000)
 Tx: aggressive normal saline hydration (switch to 0.45% if sodium replete), potassium
supplementation, IV insulin NOT SUB-Q INSULIN DAMMIT and close moitoring
o Use ketone level or anion gap to track response to treatment

3800/3795: Diabetic Neuropathy


 Typically, this occurs in a stocking-glove pattern. Two major manifestations of diabetic neuropathy:
o Small fiber disease: pain/allodynia predominate with less sensory/reflexive loss
o Large fiber disease: numbness and loss of sensation, often with loss of distal reflexes
 Tx: many options available and are listed in terms of desire:
o First line: TCAs (amitriptyline, desipramine, nortriptyline); good but may cause worsening
urinary symptoms and autonomic hypotension
o Second line: Gabapentin; best used when TCA side effects cannot be tolerated
o Third line: NSAIDs (ketorolac, etc.); may worsen renal functions, thus not as desired

Neuropathy Associated with Diabetes Mellitus


 Femoral neuropathy – weakness of the lower left leg, giving way of the knee, and discomfort in the
anterior thigh. Associated, but caused by conditions common to diabetics, not the diabetes itself.
 Meralgia Parasthetica – lateral femoral cutaneous nerve neuropathy, secondary to diabetes.
Numbness and paresthesia over the anterolateral thigh with no motor dysfunction
 Ileofemoral Atherosclerosis - common complication of diabetes mellitus, may produce intermittent
claudication involving one or both calf muscles but would not produce the motor weakness noted in
this patient.
 Diabetic polyneuropathy - symmetric and distal limb sensory and motor deficits with gradual onset

3797: Gastroparesis due to Diabetic Autonomic Neuropathy


 Presentation: decreased appetite, nausea, abdominal bloating, and early satiety after meals in the
setting of long-standing diabetes mellitus II
 Dx: clinical presentation, possibly gastric emptying study showing gastroparesis
 Tx: drugs (below) with small, more frequent meals
o Metoclopramide: prokinetic/antiemetic; risk of tardive dyskinesia
o Erythromycin: helps with GI kinetic activation; best used IV in acute exacerbations
o Cisapride: typically, not used for risk of cardiac arrhythmias/death

4750: Pathophys of Diabetic Nephropathy


 Caused by advanced glycation of the glomerular basement membrane, resulting in dysfunction 
microalbuminuria  frank albuminuria
 Histologic changes: diffuse nodular glomerulosclerosis (Kimmelstiel-Wilson nodules)

2810: Renal Tubular Acidosis in Diabetes Mellitus


 Classically Type 4 RTA (retention of K+/H+ by kidney resorption) occurs in poorly controlled DM
 This results in preserved kidney function, with non-anion gap metabolic acidosis
o Labs: mild hyperkalemia, mild acidosis, diminished GFR, no anion gap

[AAFP Questions]
Intensive management of hyperglycemia, with a goal of achieving nondiabetic glucose levels, helps reduce
microvascular complications such as retinopathy, nephropathy, and neuropathy (which subsequently
decreases foot infections).
 Intensive management of hyperglycemia also has a beneficial effect on cardiovascular disease in
patients with type 1 but not type 2 diabetes mellitus.
 In fact, there is data to suggest 1c that intensive glycemic control (hemoglobin A <6.5) may be
detrimental in certain populations, such as the elderly and those with cardiovascular disease.

Diabetes mellitus and cigarette smoking are significant risk factors for intermittent claudication, as are
hypertension and dyslipidemia.
Metformin and insulin are the only agents approved for treatment of type 2 diabetes mellitus in children.

The recommended time to screen for gestational diabetes is 24–28 weeks gestation. The patient may be
given a 50-g oral glucose load followed by a glucose determination 1 hour later.

Advisory Committee on Immunization Practices of the Centers for Disease Control andPrevention
recommended hepatitis B vaccine for all previously unvaccinated adults between the ages of19 and 59 with
diabetes mellitus, as soon as possible after the diagnosis of diabetes is made.

Onset of a lot of skin tags (acrochordons) are associated with diabetes mellitus and obesity. Typically occurs in
early adulthood, and the most common locations are the neck and axillae.

Testing for diabetes mellitus should be considered in all asymptomatic adults who have a [BMI ≥25 kg/m2 +
one or more additional risk factors]:
 Physical inactivity, a first degree relative with diabetes, a high-risk ethnicity, hypertension,
hyperlipidemia, or polycystic ovary syndrome. In asymptomatic patients with no risk factors
 Screening should begin at age 45.

Treating Diabetes in Pt with renal disease


 Glipizide is a good option as it is not eliminated renally OR has metabolites eliminated renally
 Metformin (avoid with Cr >1.5) is eliminated renally and glyburide has an active metabolite that’s
eliminated renally

The black box warning for thiazolidinediones (like pioglitazone) specifically addresses heart failure. These
agents are also contraindicated in patients with type 1 diabetes mellitus or hepatic disease, and in
premenopausal anovulatory women.

Case 52 – Adverse Drug Reactions


Adverse drug reactions happen all the time. There are a TON of reasons for them. It’s important to remember:
 More drugs = more problems: taking >5 meds gives about a 33% chance of a drug rxn
 Old people: changes in liver function, kidney function, water volume distribution, and simply taking
more medications puts old people a much increased risk
 Be aware: always ask, “might I have caused this?”, when a person on polypharmacy presents. Herbal
supplements and OTCs can often play a huge role, always ask!
 If unsure about problem drugs, used the Beers Criteria list or the STOPP list to screen for drugs that are
often found to cause adverse reactions in older folks
Liver
 Often drugs undergo first-pass metabolism, thus altering the liver’s ability to metabolize drugs can
hugely affect how much drug gets into the system
 Liver metabolism of medications is largely via CYP enzymes; alteration of these enzymes (alcohol,
grapefruit juice, medications, etc.) is often the culprit when investigating these problems
 Albumin levels may also drop with liver function decline. Drugs that are albumin-bound may
dramatically increase serum levels if now unbound!
Kidney
 Many drugs are excreted by the kidney and function may be affected by drugs! Thus it’s important to
monitor kidney function (via Creatinine) when assessing adverse drug reactions
 Pt with chronic diseases that affect the kidney (diabetes, HTN, etc.) should be closely monitored when
using renal eliminated drugs

Case 53 – Low Back Pain


Lower back pain can range from mild and benign to severe and life threatening. As the primary care doc, it’s
your job to differentiate between the two so interventions are prompt and appropriate.

Cauda Equina Syndrome


 Compression of the cauda equina as they exit the lower spine; may result in permanent nerve damage
 Presentation: low back pain often radiating down the legs (“bilateral sciatica”); leg weakness,
bladder/bowel dysfunction, anesthesia/paresthesia in “saddle distribution”, decreased anal sphincter
tone, decreased lower extremity reflexes
o Straight leg test produces pain/worsens symptoms
 Dx: clinical presentation with immediate lumbar MRI
 Tx: immediate surgical decompression of lumbar spine

Infection In/Around the Spine


 Osteomylitis/Pott’s Disease, paraspinous abscess, epidural abcess, or septic discitis
 Presentation: typically, low back pain with specific area of discomfort and possibly signs of underlying
infection (fever, redness, fluctant mass, etc.)
o Labs: CBC, ESR
o Imaging: Osteolytic lesions in spine or abscess formation (often plain films  CT/MRI)
o Classically IV drug users could get this disease
 Dx: signs/symptoms with biopsy or strong clinical suspicion
 Tx: surgical debridement/drainage with IV antibiotcs

Metastatic Cancer to the Spine


 Presentation: low back pain with insidious onset. Weight loss, worsening at night, failure to improve
after 1 month, and age >50 are all signs.
o Typically, osteoblastic lesions present in lumbar spine
o MRI/radioactive bone scan should be done to look/ID lesions
o Multiple myeloma or metastatic prostate/breast/lung cancers are most common
 Dx: made with biopsy
 Tx: resection/chemotherapy/radiation therapy

Herniated Disc
 Squeezing out of the spinal disc contents to compress the surrounding spinal cord
 Presentation: sharp/burning pain that radiates down the back & side of the leg.
o Improves with lying down/resting
o Worsens with Valsalva, sneezing, coughing, and activity & straight leg raise
o Specific signs of radioculopathy (anesthesia, paresthesia, etc.) should only be confined to one or
two dermatomes corresponding with area of compression
 Dx: clinical presentation
o Most occurs at the L4/L5 spinal outlet
 Tx: NSAIDs (pain control), physical therapy, or corticosteroids  surgical resolution
Spinal Stenosis
 Narrowed canals for passage of the spinal nerves causing symptoms
 Presentation: back pain, LE weakness/parasthesia, pseudoclaudication with activity
o Improves with bending over/squatting/sitting/lying down (opening up those canals)
o Most common in pts over 60yrs
 Dx: clinical presentation
 Tx: NSAIDs (pain control), physical therapy, or corticosteroids  surgical resolution

Vertebral Compression Fracture


 Acute damage to spine causing compression due to altered outlets
 Presentation: acute onset back pain, classically after impact, sudden movement, lifting/bending or
coughing. Pain often moves from back to abdomen (“girdle of pain”)
o Most common at T12-L2 spinal outlet
 Dx: plain X-ray of spine (shows fracture)
 Tx: rest, analgesia, treatment of underlying osteoporosis (calcitonin + bisphosphonates)

Lumbar Strain – lower back pain due to incomplete tear of the annulus fibrosis and leakage of fluid into
surrounding tissue causing inflammation, but no major damage. Often this is the diagnosis of low back pain
with a negative workup.

Case 54 – Intellectual Disability/Developmental Disorders

Case 55 – Movement Disorders


Idiopathic Parkinson’s Disease
Spontaneous neurodegenerative disorder, but has some genetic risk factors: SNCA (a-synuclein), PARK2 (E3
ubiquitin ligase), UCHL1 (ubiquitin carboxy-terminal esterase L1), PINK1 (PTEN-induced putative kinase 1),
LRRK2 (leucine0rich repeat kinase 2) are prominent ones

Pathology: brain-wide neurodegeneration with specific concern of dopaminergic neuron loss in the vental
substantia nigra pars compacta being a main driver of disease motor manifestations

Symptoms: Four major symptoms with one less common symptom are characteristic of PD
 Pill rolling tremor: slow (3-5 Hz); evident at rest
 Cogwheel rigidity: arm passively moves in a “chunking” cogwheel fashion
 Bradykinesia (slow movement)/bradyphrenia (slow thinking)
 Postural instability: failed postural muscle reflexes needed to maintain balance
 Dementia: less common (20-30%) but recognized as a result of primary disease
Tx:
 Levodopa-carbidopa (carbidopa is a COMT inhibitor to keep L-dopa levels high) – may cause dyskinesia
 DA agonists (pramipexole, ropinirole, bromocriptine) – may decrease incidence of dyskinesia if used 1st
 MAO-B inhibitor (selegiline, rasagiline) – augments L-dopa and primarily helps disease symptoms
 Amantadine (NMDA antagonist; useful for eliminating L-dopa induced dyskinesia)
 Deep Brain Stimulation of the subthalamic nucleus
Note: Anti-psychotics and anti-emetics (prochlorperazine/metoclopramide) can induce Parkinsonism

Parkinson Syndromes: some syndromes share “Parkinsonism” as a feature but are not PD.
 Progressive supranuclear palsy: parkinsonism with characteristic ophthalmoplegia (limited vertical
gaze > limited horizontal gaze); difficulty looking down makes it hard to walk
 Cortico-basal ganglionic degeneration: parkinsonism with apraxia (difficulty in executing motor/other
actions despite normal strength/sensory function) and alien-hand syndrome
 Diffuse Lewy Body disease: parkinsonism with early dementia, hallucinations, and extreme sensitivity
to neuroleptic drugs
 Vascular Parkinsonism: parkinsonism that affects mainly the lower limb accompanied by vascular
disease
 Multiple systems Atrophy: parkinsonism with early autonomic/corticospinal/cerebellar dysfunction
and sometimes myoclonus or vocal cord paresis

Drug-Induced Movement Disorders


Often treatment of psychotic disorders incudes D2 DA-receptor blockage, thus it should be no surprise that
these drugs may result in movement disorders. Some big ones are below:
 Akathisia – inability to sit still (ants in the pants); Tx: anti-cholinergics, beta-blockers, decrease
neuroleptic
 Neuroleptic Malignant Syndrome – muscle rigidity (^CK levels), fever, leukocytosis, autonomic
instability, and altered consciousness. Tx: discontinue neuroleptic, antipyretics, rehydration,
bromocriptine/dantrolene, physical cooling
 Tardive Dyskinesia – uncontrollable lip smacking/tongue rolling/chewing in the context of neuroleptic
use (often typical antipsychotics); Tx: discontinue drug, DA depletion may aid in limiting symptoms but
often this is irreversible

Stiff-Person Syndrome
Rare autoimmune/paraneoplastic disorder of progressive muscle rigidity with painful spasm
 Tin man gait: Stiffness in trunk/axial muscles, lumbar hyperlordosis, and restricted hip/spine mobility
 Painful spasms occurring suddenly and sometimes in response to startle
Dx: clinical symptoms and continuous NMJ stimulation without evidence of other disease
 Anti-glutamic acid decarboxylase (GAD) or Anti-amphiphysin antibodies may be found
 CSF is often normal
Tx: benzodiazepine and baclofen for muscle relaxation to stop spasms
 If autoantibodies present, immunosuppressive therapy (steroids, plasmapheresis, IVIG) are useful

Tremor
Involuntary rhythmic movement of the body. Resting (when pt at rest), postural (when pt maintains a posture
for a prolonged time), action (appears with voluntary movement), or intention (appears when action has
nearly completed)
 Action/intension tremors are a feature of cerebellar disease
 Resting tremor is characteristic of Parkinsonism

Essential Tremor
Defined: the most common movement disorder; a simple, isolated tremor with no serious sequelae.
 May be a “benign familial tremor” due to autosomal dominant gene mutation
Signs/Symptoms:
 Tremor: bilateral onset, arms>head>legs>larynx>trunk, occurs with purposeful movement (“task
specific tremor”), latency is immediate onset with outstretched arms, “jerky” quality
 Froment sign: some rigidity will be present in the arms; NOT COGWHEEL
 Small amounts of alcohol improve tremor
 PET scan shows increased activity in the thalamus
Dx: mainly clinical with ruling out other causes of tremor
Tx:
 Propranolol – B-blocker, aids in decreasing the tremor; concern with asthma/diabetes/heart problems
 Primidone – anti-seizure med; used in low doses for those who don’t respond to B-blockers
 Tranquilizers/Botox may be used to paralyze the tremor if refractory
 Deep Brain stimulation in the thalamus to limit its increased activity is a final effort

Chorea
Involuntary dance-like movements, often with clumsiness and discoordination. Some other features include:
 Motor Impersistence: failure to sustain a motor contraction
 Serpentine Tongue: cannot keep the tongue protruded, thus it writhes like a snake
 Milkmaid grip: cannot maintain grip thus the pt hand slides down like it’s milking what their gripping
Causes: There’s a huge list of causes of chorea but some major ones include Huntington’s Disease, Post-
streptococcal infection (Syndenham chorea), SLE, thyrotoxicosis, and pregnancy
Tx: haloperidol (D2 antagonist) has had success

Ballism
Poorly controlled flinging/flailing movements of the limb (arm goes ballis-tic)
 Hemiballismus: unilateral ballism; typically, from contralateral basal-ganglia lesions or hyperglycemia
 Tx: DA depletion/blockaid is typical therapy | thalamotomy/pallidotomy may help in severe cases

Dystonia
Defined: sustained muscle contraction; often contorting the pt into uncomfortable positions
 Primary Dystonia: no discernable cause is found; diagnosed only after secondary ruled out; several
different types, but the two most common are:
o DYT-1: caused by autosomal dominant glutamate deletion in torsin A (9q34); difficult to treat;
most common primary dyst.
o DYT-5: dystonia responds to dopamine agonists
 Secondary Dystonia: caused by something else! Classically medications, Wilson’s Disease, and a
handful of other things will cause this
Signs/Symptoms:
 Often childhood onset with worsening until a plateau in early adulthood
 Sustained muscle contractions causing twisting, abnormal posturing, or repetitive motions
Dx:
 Extensive History and Physical searching for secondary causes; classically from anti-psychotic drugs or
metoclopramide (anti-emetic)
 If secondary causes ruled out, consider genetic testing to confirm primary dystonia
Tx:
 Levodopa/DA agonists: stimulate basal ganglia to stop aberrant firing of GPi and SNpr
 Anti-cholinergic drugs (trihexiphendyl): limit the aberrant ACh firing stimulated by dysregulated basal
ganglia activation
 Benzos/baclofen (sedative/muscle relaxant): may help relax the muscles
 Deep brain stimulation of the GPi to inhibit basal ganglia activity
 Benadryl IV may help with dystonia from DA blockade as it can stimulate cholinergic activity

Myoclonus
Sudden “lightning-like” movements from brief, intense muscle contraction (positive) or inhibition (negative)
that can occur in 4 different contexts:
 Physiologic (hiccups, hypnic jerks) – occur due to come physiologic process
 Essential – no underlying etiology can be found; often improves with alcohol
 Epileptic – occurs in context of epileptic seizures
 Symptomatic – occurs as a symptom of primary pathologic process
Tx: clonazepam or valproic acid (both anti-consultants) are used successfully

Tics
Abrupt, stereotyped, coordinated movements or vocalizations as a result of an “inner urge” or tension that is
relieved by engaging in the tic

Tourette’s syndrome is the classic tic disorder characterized by at least 1 motor and 1 vocal tic for >6 months
causing significant distress/impairment to the patient
 Often prominent in teenage years but can diminish into adulthood
 Associated with Obsessive-Compulsive Disorder
Pediatric Autoimmune Neurologic Disorders Associated with Streptococcal-Infection (PANDAS)
 Tic/OCD exacerbation following a Group A B-hemolytic streptococcal infection
 Strep-autoantibody formation after the infection affects the basal ganglia (unproved theory)
Tx: DA-antagonists are most effective but Clonazepam/clonidine have more favorable side-effect profiles, and
are often first-line

Wilson’s Disease
Autosomal recessive disorder due to copper-binding protein mutation
 Mutation  impaired copper-ceruloplasmin conjugation  copper entry to biliary excretory pathway
 Copper accumulates in the liver (liver damage) and spills into the blood (deposition into tissue)
Symptoms
 Parkinsonism: due to basal ganglia deposition; rigidity, tremor, dyskinesia
 Cognitive Dysfunction: due to cortical deposition; cognitive slowing, difficulty with processing
 Mood/Personality changes from cortical frontal-lobe deposition
 Kayser-Fleischer ring: golden/brown/greenish discoloration in the cornea due to deposition in
Descemet’s membrane
Dx:
 Increased serum copper, decreased ceruloplasmin are expected, not 100%
 Increased 24-hr urine copper excretion is most sensitive screening
 Kayser-fleshcer rings in the eye are helpful clue
 Liver biopsy with copper deposition is diagnostic
Tx: lifelong copper chelation with D-penicillimine or newer zinc chelators; screening for family members

Paroxysmal Dyskinesias
Rare disorders causing recurrent attacks of hyperkinesis with preserved consciousness
 Kinesigenic: short and abrupt; often triggered by sudden movements
 Non-kinesigenic: longer lasting; often triggered by alcohol, fatigue, and stress
 Exercise-induced dystonia (occurs with exercise) may last for hours
Tx: carbamazepine (anti-convulsants)

Case 56 – Wheezing and Asthma


4617/3027: Asthma Classification and Treatment
 Intermittent: daytime symptoms <2x week | nighttime symptoms <2x month | short-acting B-agonist
use <2x week with normal baseline PFTs and no impairment of function
o Tx: PRN albuterol
 Mild Persistent: symptoms >2x week but NOT daily | nighttime symptoms 3-4x month | normal PFTs
baseline with minor limitations of activitities
o Tx: PRN albuterol, inhaled corticosteroid
 Moderate Persistent: daily symptoms, weekly nighttime awakenings, moderate limitation of activites,
with FEV1 60-80% predicted value
o Tx: PRN albuterol, inhaled corticosteroid, inhaled LABA
 Severe Persistent: symptoms throughout day, frequent nighttime awakenings, very limited activity,
FEV1 <60% predicted value
o Tx: PRN albuterol, high-dose inhaled corticosteroid, inhaled LABA, +/- oral prednisone

3026: B-2 agonist inhalers (albuterol) are the first line treatment to induce bronchodilation in an acute asthma
attack, however they are not without side effects:
 Low Potassium  Muscle weakness, arrhythmias, EKG changes (B-agonism drives K+ into cells by
activating the Na-K-ATPase)
 Tremor, headache, and palpitations may also occur
 When this complication occurs  check serum electrolytes to confirm K+ levels

4105/4706: Acute Asthma Exacerbation


 Presentation: dyspnea, cough, accessory respiratory muscle use, difficulty speaking,
hyperventilation/tachypnea (respiratory alkalosis, low PaCO2), lung hyperinflation, tachycardia
 Dx: clinical signs/symptoms with history of asthma
o Note that a normal PaCO2 on arterial blood gas is a sign of impending respiratory collapse.
Normally the person will hyperventilate, BUT if they’re so fatigued from the attack that they’re
starting to wear out, they can no longer hyperventilate, PaCO2 will go back to normal and
they’re getting towards the point where they’ll stop breathing from fatigue.
 Tx: SABA (albuterol)  +systemic corticosteroid  +ipratropium nebulizer  +one-time infusion of
magnesium sulfate (after 1-hr without response to other therapies)  intubation/mechanical
ventilation

4863: Differentiating between Major Lung Diseases with PFTs


 Asthma – low FEV1/FVC (obstructive) with resolution on bronchodilator challenge
 COPD – low FEV1/FVC (obstructive) with no resolution on bronchodilator challenge
 Chest Wall Weakness – normal to high FEV1/FVC (restrictive) with low Vital Capacity + normal DLCO
(no difficulty with air diffusion across lung tissue)
 Interstitial Lung Disease – normal to high FEV1/FVC (restrictive) with low Vital Capacity + low DLCO
(trouble with getting gas across the lung tissue)
o Classic is long-tome asbestos exposure with lung calcifications  pneumoconiosis

8905: Distinguishing Asthma from COPD


 Clinically they can both look similar, especially in an adult patient, but there are two major hallmarks
that can set asthma off from COPD:
o Reversibility: FEV1 increase by >12% demonstrates reversibility  asthma. COPD may be
partially reversible in early stages, but never to this degree.
o DLCO: normal or increased DLCO  asthma. COPD may be normal in early stages, but never
increased.
 Note that adult onset asthma is harder to treat (more limited response), but is usually the situation
where you’re trying to distinguish it from COPD
3050: Exercise Induced Bronchoconstriction
 Bronchoconstriction/difficulty breathing often triggered by high volumes of dry, cold air. Classically
occurs in athletes with current or previous diagnosis of asthma!
 Tx:
o First line: Albuterol (short acting B-agonist) inhaler 10-20 before exercise
o Second line: Anti-leukotriene inhaler 15-20 minutes before exercise (cannot tolerate albuterol!)
o Can be combined in high-performance athletes

4335: Asthma exacerbation due to GERD


 Presentation: recent worsening of asthma symptoms with normal lung exam. Often sore throat,
morning hoarseness, worsening cough when laying down (at night is common), need for inhaler
following meals, dysphagia, chest pain/heartburn, regurgitation sensation
o Result of microaspirations causing irritation and vagal hypersensitivity resulting in proclivity to
bronchospasm
o Often coincides with weight gain (possibly due to a stomach ulcer!)
 Tx: PPI trial

2842/4065: Aspirin-Exacerbated Respiratory Disease (AERD)


 Presentation: asthmatic symptoms (cough, wheezing, chest tightness), allergic symptoms (chest
congestion, rhinorrhea, periorbital edema), and facial flushing within 30 min – 3hr of NSAID use
o Associated with Nasal polyp, asthma, and chronic rhinosinusitis
o Very similar to allergic/asthmatic flare-up, thus symptoms may not be attributed to NSAID
o Nasal polyp development can cause bland tasting food (caused by anosmia), recurrent nasal
discharge with grey-glistening mucoid mass in the nasal canal.
o Considered a pseudo-allergic reaction that is not IgE mediated. Instead the aspirin diverts the
arachidonic acid pathway to produce high amounts of leukotrienes = inflammation
 Dx: Clinical scenario
 Tx: Surgical removal of nasal polyp, management of underlying conditions, avoidance/desensitization
of NSAIDs, and sometimes leukotriene inhibitors (ziletuon/montelukast)

4742: Oddly enough this question painted a picture of a guy with asthma via telling us that he had chronic
rhinitis, nasal polyps, and eczema. He likely had unstable angina and treatment may have caused him to have
a dry cough!
 Aspirin = common trigger for bronchoconstriction for asthmatics
 Non-selective B-blockers = limit ability to bronchodilate, thus exacerbating asthmatics
This guy was also on an ACE inhibitor…which has a classic side effect of dry cough

4771: There are only two major mechanisms for pulsus paradoxus:
 Cardiac tamponade (we’ve discussed this one!)
 Severe asthma/COPD: typically, there’s a small variation in intrathoracic pressure with inspiration (gets
air to flow in); but in asthma/COPD, air trapping raises pressure in the lungs, resulting in a much more
intense decrease in lung pressure during inspiration to allow for inward flow (nearly 20x greater
pressure drop). This results in blood pooling in the lung vasculature (decrease preload), and inflation
causes impingement on the heart (decrease outflow). Both of these things result in less blood coming
out of the heart upon inspiration, resulting in a drop in BP with inspiration!

Case 57 – Obstructive Sleep Apnea


3033/4448: Obstructive Sleep Apnea
 Presentation: daytime sleepiness, morning headaches, poor sleep with multiple awakenings
o Labs: erythrocytosis with increased EPO (reaction to hypoxemia)
o Risk factors: Obesity, tonsillar hypertrophy, excessive oropharyngeal tissue, or short mandible
o Sequlae: cor-pulmonale, right-heart failure, pulmonary/systemic HTN, depression, daytime
accidents due to sleepiness, impotence
 Dx: nocturnal polysomnigraphy  >15 apneic or hypopnea events in a night
 Tx: directed at underlying cause (CPAP, BiPAP, repositioning device, surgical therapy)

4489/4663/4413: Obesity Hypoventilation Syndrome (Pickwickian Syndrome)


 Basically these people are so fat that it’s making chest wall expansion difficult, thus their lungs are OK
but they cannot ventilate properly  respiratory acidosis
o Long-standing with metabolic alkalosis compensation blunts the body’s response to increase
respiratory drive, leading to worsening O2 saturation
 Presentation: obese patient with difficulty breathing (SOB, orthopnea) but clear lungs and no overt
signs of heart failure but cor pulmonary/pulmonary HTN can be features
o Labs: hypoxemia, hypercapnia, polycythemia, increased bicarbonate with decreased Cl -
o Obstructive Sleep Apnea (apneic events, difficulty sleeping, excessive daytime sleepiness) may
co-exist due to decreased nighttime respiratory drive AND poor chest wall expansion
 Dx: clinical picture
 Tx: nocturnal PEEP/BIPaP (first line) + weight loss + avoidance of sedatives.
o Acetazolamide to increase respiratory drive is a last resort

4593/4297: Cor pulmonale: process of increased lung-vascular pressure. Untreated, causes right heart failure.
 Etiology: COPD (most common), idiopathic pulmonary HTN, interstitial lung disease, obstructive sleep
apnea. Note that LVF causing RVF is NOT cor pulmonale.
 Dx:
o Sym: Dyspnea, syncope, or angina on exertion
o Sign: Peripheral edema, increased JVD, hepatojugular reflex, pulsatile liver, edema, ascities, etc.
o Imaging: EKG or echo can be used but right heart catheterization with elevated pulmonary
artery systolic pressure >25mmHg is confirmatory

2788/4677: Respiratory Acidosis from Alveolar Hypoventilation


 For any reason, if a person is not ventilating enough they’re not going to be able to blow off enough
CO2 to keep blood pH at a normal level. This will result in a respiratory acidosis.
 Some causes of hypoventilation include:
o Pulmonary/thoracic disease: COPD, sleep apnea, obesity hypoventilation, scoliosis
o Neuromuscular disease: Myasthenia gravis, Lambert-Eaton syndrome, Guillian Barre syndrome
o Drugs: illegal narcotics (heroin), sedatives, anesthesia
o Primary CNS hypoventilation: brainstem damage/stroke, herniation, post-ictal state seizure
 Classically this presents with acidosis (pH <7.4), hypoxemia (PaO2 <75), and hypercapnia (PaCO2 >45),
especially within the first 48hr before the kidneys can help compensate

Case 58 – Osteoporosis
Osteoporosis – low bone mass which puts pts at risk for fractures. Defined as a Bone Mineral Density of
<2.5SD below what’s considered normal for a healthy adult
 Osteopenia – same, but BMD is from 1.0-2.5SD below a healthy adult

Risk Factors
 Classics: Low body weight, previous fracture, a family history of osteoporosis with fracture, a history of
falls, physical inactivity, low vitamin D or calcium intake
 Chronic systemic diseases: COPD, HIV, severe liver disease, renal failure, systemic lupus
erythematosus, and rheumatoid arthritis.
 Endocrine disorders: type 1 diabetes mellitus, hyperparathyroidism, hyperthyroidism, Cushing’s
syndrome, and others.
 Medications: anticonvulsants, corticosteroids, and immunosuppressants.
 Nutritional risks: celiac disease, vitamin D deficiency, anorexia nervosa, gastric bypass, and increased
alcohol or caffeine intake.
 Note that obesity can be protective (you’re constantly moving all that weight!)

Prevention
 Because this typically a disease of the elderly, the normal rate of bone mineral loss can be staved off in
the hopes that a person may complete their life without symptoms of osteoporosis
 Men & Women >50yr – at risk; should have Ca2+ + Vit.D supplementation & perform weight bearing
exercise to reduce risk
 Smoking cessation & alcohol reduction are also recommended

Screening
 Women >65yr should have a dual-energy X-ray Assessment (DEXA) of hip/lumbar spine scan to assess
BMD; if qualifying for osteoporosis based on this test, then treatment may be initiated
 Women <65yr with risk factors of a woman >65yr based on the WHO’s FRAX 10-yr risk calculator
o Things increasing risk are glucocorticoid use, low body weight, previous fractures, rheumatoid
arthritis, and some other risk factors
 According to USPSTF, men of any age are not recommended to have screening
o Some societies recommend screening of men >70yr of age or men 50+ with risk factors

Diagnosis
 Made with DEXA scanning showing T-score -2.5 of greater
 Secondary causes should be sought (hyperthyroid, hyperparathyroid, anorexia, tobacco/alcohol abuse)
Treatment
 Those who should be treated for osteoporosis
o DEXA less than -2.5 at femoral neck or spine
o DEXA between -2.5 and -1.0 with FRAX 10-yr risk >20%
o Aged >50 with previous Hip or Vertebral fracture
 Non-pharmacologic – fall prevention, smoking/alcohol cessation, Ca2+ 1200mg daily, VitD loading dose
50,000 IU with maintenance of 1000IU daily (goal of >30 ng/mL of 25-OH VitD on random test)
 1st line - Bisphosphnates (alendronate, risendronate, etc.) – inhibit/kill osteoclasts to minimize bone
breakdown and promote increasing bone density. Must be taken on empty stomach with full glass of
water. Large risk of erosive esophagitis. Rare risk of osteonecrosis of the jaw after dental procedures.
 Hormone Replacement (raloxifene only) – due to risk of increasing breast/endometrial cancer risk,
these drugs are somewhat limited as a class. Raloxifene can be used as it works as an estrogen agonist
at the bone, but not the breast or endometrium. Used in post-menopausal woman who cannot
tolerate bisphosphonates.
 Calcitonin – nasal spray used as second line. Works by promoting uptake of calcium from the blood
into the bones and has a modest analgesic effect.
 Teriparatide – recombinant human parathyroid hormone (promote Ca resorption and uptake). Will
stimulate osteoblast activity, thus contraindicated in patients with osteosarcoma, Paget’s Disease of
the bone, Hx of bone radiation, or elevated Alk.Phos levels
 Denosumab – prevents osteoclast differentiation and limits bone turnover. Works well, but risk of
serious infection is very real.

Case 59 – Chronic Pain Management


Chronic pain can be frustrating for both doctor and patient. On one hand the patient may truly have pain that
they cannot control or understand well. The doctor may have difficulty with diagnosis and managing these
patients takes a keep suspicion due to concerns for narcotic abuse.

When assessing someone with chronic pain:


 Search past medical history of clues/prior testing
 Provide full Hx/Physical/Psychiatric evaluation to search for underlying targets for treatment
o Duration/type/characterization of pain is important to establish
 Provide objective assessment to establish a baseline so further treatment may be stacked against it

Non-Pharmacologic Tx
 Physical therapy, psychiatric therapy, complimentary/alternative modalities of treatment can all be
part of the plan. The most important thing is patient buy in/participation. If the patient wants to try
acupuncture or whatever, it’s likely not going to hurt them and even may help (so why not!)
 It’s important to always consider cost to the patient, as insurances may not cover many modalities of
alternative therapy.

Pharmacologic Tx
 NSAIDs – non-steroidals are often a good place to start. May not help but may, who knows. Best to use
a more COX-2 specific drug to minimize GI symptoms (celecoxib [Celebrex])
 Anticonvulsants (Neurontin) – often used for neuropathic pain and a classic for folks with poorly
controlled diabetes and neuropathic pain. It’s almost like you’re depressing PNS activity!
 Muscle relaxants – often helpful with musculoskeletal pain. Not a bad one to try.
 Opioids – kill pain but at significant risk for side effects/addiction. Best to consider long-acting drugs as
the “high” is considered less addiction forming, although short-acting may be considered for break-
through pain.
o Constipation is a major side effect and should be asked about at any followup. Typically stool
softenered (Dulcolax) may be used to aid in pooping.
o Overdose/Addiction should always be on the radar in pts on long-term opioids. Typically,
patients will enter into a “contract” with the physician specifying the exact terms of their
treatment. Should they deviate, the drugs provided will be terminated

Case 60 – Lower Extremity Swelling


Clinical Assessment
 Most common reason for edema in >50yr – chronic venous insufficiency
 Most common reason for edema in <50yr – idiopathic edema
 Typically, assessment focuses around finding underlying cause

Making the Diagnosis


 Hx – characterize duration/pain/current medications/onset & improvement or worsening. Basically
you should always do a thorough history to see if something can explain the edema
 Physical – obesity, bilateral or unilateral limb swelling, generalized, how far does it go up (to the knee,
to the sacrum, etc.), pitting or no?, varicosities, skin changes are all good specific to note. A general
heart/lung/abdominal exam should ALWAYS be done.
 Diagnostic Studies –
o Unclear or heart failure suspected – CBC, CMP, urinalysis, thyroid functions, albumin, and BNP
(if CHF suspected) are all classics in a patient >50yr to help characterize systemic disease
o Idiopathic suspected – typically a young person; water load test may be useful. Often edema is
generalized and associated with obesity/depression.
o DVT – d-dimer and examination of leg with comparison with unaffected leg are important
o Sleep apnea – sleep study
o Liver disease – LFTs, albumin, and coagulation studies can be useful
o Malignancy – CT chest/abdomen; often prostate, ovarian, and lymphoma are associated with
edema
Tx
 Idiopathic Edema – typically lifestyle modifications (intermittent laying down, heat avoidance, low-Na
diet, decreased fluid intake, weight loss) with mild diuretics (spironolactone  thiazide +
spironolactone; loop diuretics should be avoided if possible)
 Venous Insufficiency – compression leg stockings/leg elevation; lower extremity Doppler studies are
often suggested for baseline; loop diuretics/surgical intervention may be necessary
 Lymphedema – exercise, elevation, manual lymphatic drainage, and sometimes surgical procedure to
aid in lymph drainage. Diuretics NOT helpful. Cellulitis may be recurrent due to swelling and should be
treated aggresivly.
 Deep Vein Thrombosis (DVT) – anticoagulation with INR goal of 2.0-3.0 should be sought. Warfarin or
equivalent NOVAC may be used. IVC filter may also be placed with anticoagulation therapy is
contraindicated to stop catastrophic clot formation.

Das könnte Ihnen auch gefallen